Cardiac Electrophysiology
Andrea Natale • Amin Al-Ahmad Paul J. Wang • John P. DiMarco (Editors)
Cardiac Electrophysiology Clinical Case Review
Editors Andrea Natale Texas Cardiac Arrhythmia Institute St. David’s Medical Center 1015 East 32nd Street Suite 516, Austin, TX 78705 USA Amin Al-Ahmad School of Medicine Stanford University 300 Pasteur Drive H-2146, Stanford, CA 94305 USA
Paul J. Wang School of Medicine Stanford University 300 Pasteur Drive H-2146, Stanford, CA 94305 USA John P. DiMarco Cardiovascular Division University of Virginia Health System 1215 Lee Street Charlottesville, VA 22908 USA
ISBN 978-1-84996-389-3 e-ISBN 978-1-84996-390-9 DOI 10.1007/978-1-84996-390-9 Springer London Dordrecht Heidelberg New York British Library Cataloguing in Publication Data A catalogue record for this book is available from the British Library Library of Congress Control Number: 2010937972 © Springer-Verlag London Limited 2011 Apart from any fair dealing for the purposes of research or private study, or criticism or review, as permitted under the Copyright, Designs and Patents Act 1988, this publication may only be reproduced, stored or transmitted, in any form or by any means, with the prior permission in writing of the publishers, or in the case of reprographic reproduction in accordance with the terms of licenses issued by the Copyright Licensing Agency. Enquiries concerning reproduction outside those terms should be sent to the publishers. The use of registered names, trademarks, etc., in this publication does not imply, even in the absence of a specific statement, that such names are exempt from the relevant laws and regulations and therefore free for general use. Product liability: The publisher can give no guarantee for information about drug dosage and application thereof contained in this book. In every individual case the respective user must check its accuracy by consulting other pharmaceutical literature. Cover design: eStudioCalamar, Figueres/Berlin Printed on acid-free paper Springer is part of Springer Science+Business Media (www.springer.com)
To our wives and families, for the constant support and patience.
Preface
As the field of cardiac electrophysiology continues to evolve and advance, we continue to be challenged to educate new generations of cardiac electrophysiologists with the basics as well as the latest advances in the field. While there are many outstanding books that provide an in-depth review of topics in electrophysiology, there are few case-based books that comprehensively cover clinical electrophysiology, devices, and ablation. Case reviews offer a simple, yet effective way in teaching important concepts, offering insight into both the basic pathophysiology of a problem as well as the clinical reasoning that leads to a solution. In “Cardiac Electrophysiology: Clinical Case Review” we sought to put together the most comprehensive case-based review of electrophysiology that would appeal to all students of the field whether they are fellows, allied professionals, or practicing electrophysiologists. In “Cardiac Electrophysiology: Clinical Case Review” we are fortunate to have many true experts in the field contributing cases that they have encountered and summarizing the important learning objectives in a succinct way. The book covers clinical electrophysiology, device troubleshooting and analysis, as well as intracardiac electrogram analysis and ablation. It is our sincere hope that the readers of “Cardiac Electrophysiology: Clinical Case Review” will find the cases useful as a review of electrophysiology or in their day-to-day interactions with patients. Stanford CA, USA Charlottesville VA, USA Cleveland OH, USA
Amin Al-Ahmad Paul J. Wang John P. DiMarco
vii
Contents
Section 1 Ablation Case 1 . . . . . . . . . . . . . . . . . . . . . . . . . . . . . . . . . . . . . . . . . . . . . . . . . . . . . . . . . . . . . . . Amin Al-Ahmad
3
Case 2 . . . . . . . . . . . . . . . . . . . . . . . . . . . . . . . . . . . . . . . . . . . . . . . . . . . . . . . . . . . . . . . Michel Haissaguerre
7
Case 3 . . . . . . . . . . . . . . . . . . . . . . . . . . . . . . . . . . . . . . . . . . . . . . . . . . . . . . . . . . . . . . . Amin Al-Ahmad
11
Case 4 . . . . . . . . . . . . . . . . . . . . . . . . . . . . . . . . . . . . . . . . . . . . . . . . . . . . . . . . . . . . . . . Michel Haissaguerre
15
Case 5 . . . . . . . . . . . . . . . . . . . . . . . . . . . . . . . . . . . . . . . . . . . . . . . . . . . . . . . . . . . . . . . Anurag Gupta and Amin Al-Ahmad
19
Case 6 . . . . . . . . . . . . . . . . . . . . . . . . . . . . . . . . . . . . . . . . . . . . . . . . . . . . . . . . . . . . . . . Michel Haissaguerre
23
Case 7 . . . . . . . . . . . . . . . . . . . . . . . . . . . . . . . . . . . . . . . . . . . . . . . . . . . . . . . . . . . . . . . Michel Haissaguerre
29
Case 8 . . . . . . . . . . . . . . . . . . . . . . . . . . . . . . . . . . . . . . . . . . . . . . . . . . . . . . . . . . . . . . . Michel Haissaguerre
35
Case 9 . . . . . . . . . . . . . . . . . . . . . . . . . . . . . . . . . . . . . . . . . . . . . . . . . . . . . . . . . . . . . . . Alberto Diaz, Dimpi Patel, William R. Lewis, and Andrea Natale
39
Case 10 . . . . . . . . . . . . . . . . . . . . . . . . . . . . . . . . . . . . . . . . . . . . . . . . . . . . . . . . . . . . . . Antonio Rossillo, Sakis Themistoclakis, Aldo Bonso, Andrea Corrado, and Antonio Raviele
41
Case 11 . . . . . . . . . . . . . . . . . . . . . . . . . . . . . . . . . . . . . . . . . . . . . . . . . . . . . . . . . . . . . . Yash Y. Lokhandwala, Anoop K. Gupta, and Ranjan K. Thakur
47
ix
x
Contents
Case 12 . . . . . . . . . . . . . . . . . . . . . . . . . . . . . . . . . . . . . . . . . . . . . . . . . . . . . . . . . . . . . . Eric Buch, Shiro Nakahara, Marmar Vaseghi, Noel G. Boyle, and Kalyanam Shivkumar
51
Case 13 . . . . . . . . . . . . . . . . . . . . . . . . . . . . . . . . . . . . . . . . . . . . . . . . . . . . . . . . . . . . . . Bradley P. Knight
53
Case 14 . . . . . . . . . . . . . . . . . . . . . . . . . . . . . . . . . . . . . . . . . . . . . . . . . . . . . . . . . . . . . . M. Eyman Mortada, Jasbir S. Sra, and Masood Akhtar
57
Case 15 . . . . . . . . . . . . . . . . . . . . . . . . . . . . . . . . . . . . . . . . . . . . . . . . . . . . . . . . . . . . . . Eric Buch, Shiro Nakahara, Marmar Vaseghi, Noel G. Boyle, and Kalyanam Shivkumar
63
Case 16 . . . . . . . . . . . . . . . . . . . . . . . . . . . . . . . . . . . . . . . . . . . . . . . . . . . . . . . . . . . . . . Antonio Rossillo, Sakis Themistoclakis, Aldo Bonso, Andrea Corrado, and Antonio Raviele
65
Case 17 . . . . . . . . . . . . . . . . . . . . . . . . . . . . . . . . . . . . . . . . . . . . . . . . . . . . . . . . . . . . . . Bradley P. Knight
69
Case 18 . . . . . . . . . . . . . . . . . . . . . . . . . . . . . . . . . . . . . . . . . . . . . . . . . . . . . . . . . . . . . . Antonio Rossillo, Sakis Themistoclakis, Aldo Bonso, Andrea Corrado, and Antonio Raviele
71
Case 19 . . . . . . . . . . . . . . . . . . . . . . . . . . . . . . . . . . . . . . . . . . . . . . . . . . . . . . . . . . . . . . Yash Y. Lokhandwala, Anoop K. Gupta, and Ranjan K. Thakur
81
Case 20 . . . . . . . . . . . . . . . . . . . . . . . . . . . . . . . . . . . . . . . . . . . . . . . . . . . . . . . . . . . . . . Marmar Vaseghi, Noel G. Boyle, and Kalyanam Shivkumar
83
Case 21 . . . . . . . . . . . . . . . . . . . . . . . . . . . . . . . . . . . . . . . . . . . . . . . . . . . . . . . . . . . . . . Bradley P. Knight
85
Case 22 . . . . . . . . . . . . . . . . . . . . . . . . . . . . . . . . . . . . . . . . . . . . . . . . . . . . . . . . . . . . . . Antonio Rossillo, Sakis Themistoclakis, Aldo Bonso, Andrea Corrado, and Antonio Raviele
87
Case 23 . . . . . . . . . . . . . . . . . . . . . . . . . . . . . . . . . . . . . . . . . . . . . . . . . . . . . . . . . . . . . . Yash Y. Lokhandwala, Anoop K. Gupta, and Ranjan K. Thakur
95
Case 24 . . . . . . . . . . . . . . . . . . . . . . . . . . . . . . . . . . . . . . . . . . . . . . . . . . . . . . . . . . . . . . 101 Marmar Vaseghi, Noel G. Boyle, and Kalyanam Shivkumar Case 25 . . . . . . . . . . . . . . . . . . . . . . . . . . . . . . . . . . . . . . . . . . . . . . . . . . . . . . . . . . . . . . 103 Bradley P. Knight Case 26 . . . . . . . . . . . . . . . . . . . . . . . . . . . . . . . . . . . . . . . . . . . . . . . . . . . . . . . . . . . . . . 107 Richard H. Hongo and Andrea Natale
Contents
xi
Case 27 . . . . . . . . . . . . . . . . . . . . . . . . . . . . . . . . . . . . . . . . . . . . . . . . . . . . . . . . . . . . . . 111 M. Eyman Mortada, Jasbir S. Sra, and Masood Akhtar Case 28 . . . . . . . . . . . . . . . . . . . . . . . . . . . . . . . . . . . . . . . . . . . . . . . . . . . . . . . . . . . . . . 115 Yash Y. Lokhandwala, Anoop K. Gupta, and Ranjan K. Thakur Case 29 . . . . . . . . . . . . . . . . . . . . . . . . . . . . . . . . . . . . . . . . . . . . . . . . . . . . . . . . . . . . . . 123 Eric Buch, Shiro Nakahara, Marmar Vaseghi, Noel G. Boyle, and Kalyanam Shivkumar Case 30 . . . . . . . . . . . . . . . . . . . . . . . . . . . . . . . . . . . . . . . . . . . . . . . . . . . . . . . . . . . . . . 125 Luis C. Sáenz and Miguel A. Vacca Case 31 . . . . . . . . . . . . . . . . . . . . . . . . . . . . . . . . . . . . . . . . . . . . . . . . . . . . . . . . . . . . . . 133 Bradley P. Knight Case 32 . . . . . . . . . . . . . . . . . . . . . . . . . . . . . . . . . . . . . . . . . . . . . . . . . . . . . . . . . . . . . . 135 Luigi Di Biase, Rodney P. Horton, and Andrea Natale Case 33 . . . . . . . . . . . . . . . . . . . . . . . . . . . . . . . . . . . . . . . . . . . . . . . . . . . . . . . . . . . . . . 139 M. Eyman Mortada, Jasbir S. Sra, and Masood Akhtar Case 34 . . . . . . . . . . . . . . . . . . . . . . . . . . . . . . . . . . . . . . . . . . . . . . . . . . . . . . . . . . . . . . 143 Antonio Rossillo, Sakis Themistoclakis, Aldo Bonso, Andrea Corrado, and Antonio Raviele Case 35 . . . . . . . . . . . . . . . . . . . . . . . . . . . . . . . . . . . . . . . . . . . . . . . . . . . . . . . . . . . . . . 147 Yash Y. Lokhandwala, Anoop K. Gupta, and Ranjan K. Thakur Case 36 . . . . . . . . . . . . . . . . . . . . . . . . . . . . . . . . . . . . . . . . . . . . . . . . . . . . . . . . . . . . . . 151 Eric Buch, Shiro Nakahara, Marmar Vaseghi, Noel G. Boyle, and Kalyanam Shivkumar Case 37 . . . . . . . . . . . . . . . . . . . . . . . . . . . . . . . . . . . . . . . . . . . . . . . . . . . . . . . . . . . . . . 153 Bradley P. Knight Case 38 . . . . . . . . . . . . . . . . . . . . . . . . . . . . . . . . . . . . . . . . . . . . . . . . . . . . . . . . . . . . . . 157 Luigi Di Biase, Rodney P. Horton, and Andrea Natale Case 39 . . . . . . . . . . . . . . . . . . . . . . . . . . . . . . . . . . . . . . . . . . . . . . . . . . . . . . . . . . . . . . 159 M. Eyman Mortada, Jasbir S. Sra, and Masood Akhtar Case 40 . . . . . . . . . . . . . . . . . . . . . . . . . . . . . . . . . . . . . . . . . . . . . . . . . . . . . . . . . . . . . . 163 Antonio Rossillo, Sakis Themistoclakis, Aldo Bonso, Andrea Corrado, and Antonio Raviele Case 41 . . . . . . . . . . . . . . . . . . . . . . . . . . . . . . . . . . . . . . . . . . . . . . . . . . . . . . . . . . . . . . 181 Yash Y. Lokhandwala, Anoop K. Gupta, and Ranjan K. Thakur
xii
Case 42 . . . . . . . . . . . . . . . . . . . . . . . . . . . . . . . . . . . . . . . . . . . . . . . . . . . . . . . . . . . . . . 185 Eric Buch, Shiro Nakahara, Marmar Vaseghi, Noel G. Boyle, and Kalyanam Shivkumar Case 43 . . . . . . . . . . . . . . . . . . . . . . . . . . . . . . . . . . . . . . . . . . . . . . . . . . . . . . . . . . . . . . 189 Bradley P. Knight Case 44 . . . . . . . . . . . . . . . . . . . . . . . . . . . . . . . . . . . . . . . . . . . . . . . . . . . . . . . . . . . . . . 191 Antonio Rossillo, Sakis Themistoclakis, Aldo Bonso, Andrea Corrado, and Antonio Raviele Case 45 . . . . . . . . . . . . . . . . . . . . . . . . . . . . . . . . . . . . . . . . . . . . . . . . . . . . . . . . . . . . . . 195 Yash Y. Lokhandwala, Anoop K. Gupta, and Ranjan K. Thakur Case 46 . . . . . . . . . . . . . . . . . . . . . . . . . . . . . . . . . . . . . . . . . . . . . . . . . . . . . . . . . . . . . . 199 Eric Buch, Shiro Nakahara, Marmar Vaseghi, Noel G. Boyle, and Kalyanam Shivkumar Case 47 . . . . . . . . . . . . . . . . . . . . . . . . . . . . . . . . . . . . . . . . . . . . . . . . . . . . . . . . . . . . . . 201 Bradley P. Knight Case 48 . . . . . . . . . . . . . . . . . . . . . . . . . . . . . . . . . . . . . . . . . . . . . . . . . . . . . . . . . . . . . . 203 Antonio Rossillo, Sakis Themistoclakis, Aldo Bonso, Andrea Corrado, and Antonio Raviele Case 49 . . . . . . . . . . . . . . . . . . . . . . . . . . . . . . . . . . . . . . . . . . . . . . . . . . . . . . . . . . . . . . 217 Eric Buch, Shiro Nakahara, Marmar Vaseghi, Noel G. Boyle, and Kalyanam Shivkumar Case 50 . . . . . . . . . . . . . . . . . . . . . . . . . . . . . . . . . . . . . . . . . . . . . . . . . . . . . . . . . . . . . . 221 Bradley P. Knight Case 51 . . . . . . . . . . . . . . . . . . . . . . . . . . . . . . . . . . . . . . . . . . . . . . . . . . . . . . . . . . . . . . 223 Yash Y. Lokhandwala, Anoop K. Gupta, and Ranjan K. Thakur Case 52 . . . . . . . . . . . . . . . . . . . . . . . . . . . . . . . . . . . . . . . . . . . . . . . . . . . . . . . . . . . . . . 229 Eric Buch, Shiro Nakahara, Marmar Vaseghi, Noel G. Boyle, and Kalyanam Shivkumar Case 53 . . . . . . . . . . . . . . . . . . . . . . . . . . . . . . . . . . . . . . . . . . . . . . . . . . . . . . . . . . . . . . 233 Bradley P. Knight Case 54 . . . . . . . . . . . . . . . . . . . . . . . . . . . . . . . . . . . . . . . . . . . . . . . . . . . . . . . . . . . . . . 235 Yash Y. Lokhandwala, Anoop K. Gupta, and Ranjan K. Thakur Case 55 . . . . . . . . . . . . . . . . . . . . . . . . . . . . . . . . . . . . . . . . . . . . . . . . . . . . . . . . . . . . . . 241 Eric Buch, Shiro Nakahara, Marmar Vaseghi, Noel G. Boyle, and Kalyanam Shivkumar Case 56 . . . . . . . . . . . . . . . . . . . . . . . . . . . . . . . . . . . . . . . . . . . . . . . . . . . . . . . . . . . . . . 245 Bradley P. Knight
Contents
Contents
xiii
Case 57 . . . . . . . . . . . . . . . . . . . . . . . . . . . . . . . . . . . . . . . . . . . . . . . . . . . . . . . . . . . . . . 247 Yash Y. Lokhandwala, Anoop K. Gupta, and Ranjan K. Thakur Case 58 . . . . . . . . . . . . . . . . . . . . . . . . . . . . . . . . . . . . . . . . . . . . . . . . . . . . . . . . . . . . . . 253 Roopinder Sandhu, Dimpi Patel, William R. Lewis, and Andrea Natale Case 59 . . . . . . . . . . . . . . . . . . . . . . . . . . . . . . . . . . . . . . . . . . . . . . . . . . . . . . . . . . . . . . 263 M. Eyman Mortada, Jasbir S. Sra, and Masood Akhtar Case 60 . . . . . . . . . . . . . . . . . . . . . . . . . . . . . . . . . . . . . . . . . . . . . . . . . . . . . . . . . . . . . . 267 Matthew D. Hutchinson Case 61 . . . . . . . . . . . . . . . . . . . . . . . . . . . . . . . . . . . . . . . . . . . . . . . . . . . . . . . . . . . . . . 271 Ronald Lo, Henry H. Hsia, and Amin Al-Ahmad Case 62 . . . . . . . . . . . . . . . . . . . . . . . . . . . . . . . . . . . . . . . . . . . . . . . . . . . . . . . . . . . . . . 275 Richard H. Hongo and Andrea Natale Case 63 . . . . . . . . . . . . . . . . . . . . . . . . . . . . . . . . . . . . . . . . . . . . . . . . . . . . . . . . . . . . . . 279 David J. Callans Case 64 . . . . . . . . . . . . . . . . . . . . . . . . . . . . . . . . . . . . . . . . . . . . . . . . . . . . . . . . . . . . . . 283 J. David Burkhardt, Dimpi Patel, and Andrea Natale Case 65 . . . . . . . . . . . . . . . . . . . . . . . . . . . . . . . . . . . . . . . . . . . . . . . . . . . . . . . . . . . . . . 287 Matthew D. Hutchinson Case 66 . . . . . . . . . . . . . . . . . . . . . . . . . . . . . . . . . . . . . . . . . . . . . . . . . . . . . . . . . . . . . . 291 Matthew D. Hutchinson Case 67 . . . . . . . . . . . . . . . . . . . . . . . . . . . . . . . . . . . . . . . . . . . . . . . . . . . . . . . . . . . . . . 295 Matthew D. Hutchinson Case 68 . . . . . . . . . . . . . . . . . . . . . . . . . . . . . . . . . . . . . . . . . . . . . . . . . . . . . . . . . . . . . . 299 Matthew D. Hutchinson Case 69 . . . . . . . . . . . . . . . . . . . . . . . . . . . . . . . . . . . . . . . . . . . . . . . . . . . . . . . . . . . . . . 303 Matthew D. Hutchinson Case 70 . . . . . . . . . . . . . . . . . . . . . . . . . . . . . . . . . . . . . . . . . . . . . . . . . . . . . . . . . . . . . . 307 Matthew D. Hutchinson Case 71 . . . . . . . . . . . . . . . . . . . . . . . . . . . . . . . . . . . . . . . . . . . . . . . . . . . . . . . . . . . . . . 311 Matthew D. Hutchinson Section 2 Devices Case 72 . . . . . . . . . . . . . . . . . . . . . . . . . . . . . . . . . . . . . . . . . . . . . . . . . . . . . . . . . . . . . . 317 Amin Al-Ahmad and Paul J. Wang
xiv
Case 73 . . . . . . . . . . . . . . . . . . . . . . . . . . . . . . . . . . . . . . . . . . . . . . . . . . . . . . . . . . . . . . 319 Amin Al-Ahmad and Paul J. Wang Case 74 . . . . . . . . . . . . . . . . . . . . . . . . . . . . . . . . . . . . . . . . . . . . . . . . . . . . . . . . . . . . . . 321 Kenneth A. Ellenbogen Case 75 . . . . . . . . . . . . . . . . . . . . . . . . . . . . . . . . . . . . . . . . . . . . . . . . . . . . . . . . . . . . . . 325 Nora Goldschlager Case 76 . . . . . . . . . . . . . . . . . . . . . . . . . . . . . . . . . . . . . . . . . . . . . . . . . . . . . . . . . . . . . . 327 Fred M. Kusumoto, Jennifer Crain, and Nora Goldschlager Case 77 . . . . . . . . . . . . . . . . . . . . . . . . . . . . . . . . . . . . . . . . . . . . . . . . . . . . . . . . . . . . . . 331 Gregory M. Marcus and Nora Goldschlager Case 78 . . . . . . . . . . . . . . . . . . . . . . . . . . . . . . . . . . . . . . . . . . . . . . . . . . . . . . . . . . . . . . 335 Amin Al-Ahmad and Paul J. Wang Case 79 . . . . . . . . . . . . . . . . . . . . . . . . . . . . . . . . . . . . . . . . . . . . . . . . . . . . . . . . . . . . . . 337 Kenneth A. Ellenbogen and Rod Bolanos Case 80 . . . . . . . . . . . . . . . . . . . . . . . . . . . . . . . . . . . . . . . . . . . . . . . . . . . . . . . . . . . . . . 341 Nora Goldschlager Case 81 . . . . . . . . . . . . . . . . . . . . . . . . . . . . . . . . . . . . . . . . . . . . . . . . . . . . . . . . . . . . . . 343 Fred M. Kusumoto, Jennifer Crain, and Nora Goldschlager Case 82 . . . . . . . . . . . . . . . . . . . . . . . . . . . . . . . . . . . . . . . . . . . . . . . . . . . . . . . . . . . . . . 345 Amin Al-Ahmad and Paul J. Wang Case 83 . . . . . . . . . . . . . . . . . . . . . . . . . . . . . . . . . . . . . . . . . . . . . . . . . . . . . . . . . . . . . . 347 Kenneth A. Ellenbogen Case 84 . . . . . . . . . . . . . . . . . . . . . . . . . . . . . . . . . . . . . . . . . . . . . . . . . . . . . . . . . . . . . . 349 Nora Goldschlager Case 85 . . . . . . . . . . . . . . . . . . . . . . . . . . . . . . . . . . . . . . . . . . . . . . . . . . . . . . . . . . . . . . 351 Anurag Gupta and Amin Al-Ahmad Case 86 . . . . . . . . . . . . . . . . . . . . . . . . . . . . . . . . . . . . . . . . . . . . . . . . . . . . . . . . . . . . . . 353 Fred M. Kusumoto, Jennifer Crain, and Nora Goldschlager Case 87 . . . . . . . . . . . . . . . . . . . . . . . . . . . . . . . . . . . . . . . . . . . . . . . . . . . . . . . . . . . . . . 357 Kenneth A. Ellenbogen Case 88 . . . . . . . . . . . . . . . . . . . . . . . . . . . . . . . . . . . . . . . . . . . . . . . . . . . . . . . . . . . . . . 361 Nora Goldschlager Case 89 . . . . . . . . . . . . . . . . . . . . . . . . . . . . . . . . . . . . . . . . . . . . . . . . . . . . . . . . . . . . . . 363 Kenneth A. Ellenbogen, Rod Bolanos, and Mark A. Wood
Contents
Contents
xv
Case 90 . . . . . . . . . . . . . . . . . . . . . . . . . . . . . . . . . . . . . . . . . . . . . . . . . . . . . . . . . . . . . . 369 Fred M. Kusumoto, Jennifer Crain, and Nora Goldschlager Case 91 . . . . . . . . . . . . . . . . . . . . . . . . . . . . . . . . . . . . . . . . . . . . . . . . . . . . . . . . . . . . . . 371 M. Eyman Mortada, Jasbir S. Sra, and Masood Akhtar Case 92 . . . . . . . . . . . . . . . . . . . . . . . . . . . . . . . . . . . . . . . . . . . . . . . . . . . . . . . . . . . . . . 373 Amin Al-Ahmad Case 93 . . . . . . . . . . . . . . . . . . . . . . . . . . . . . . . . . . . . . . . . . . . . . . . . . . . . . . . . . . . . . . 375 M. Eyman Mortada, Jasbir S. Sra, and Masood Akhtar Case 94 . . . . . . . . . . . . . . . . . . . . . . . . . . . . . . . . . . . . . . . . . . . . . . . . . . . . . . . . . . . . . . 379 Kenneth A. Ellenbogen and Rod Bolanos Case 95 . . . . . . . . . . . . . . . . . . . . . . . . . . . . . . . . . . . . . . . . . . . . . . . . . . . . . . . . . . . . . . 383 Byron K. Lee Case 96 . . . . . . . . . . . . . . . . . . . . . . . . . . . . . . . . . . . . . . . . . . . . . . . . . . . . . . . . . . . . . . 387 Amin Al-Ahmad and Paul J. Wang Case 97 . . . . . . . . . . . . . . . . . . . . . . . . . . . . . . . . . . . . . . . . . . . . . . . . . . . . . . . . . . . . . . 389 Kenneth A. Ellenbogen and Rod Bolanos Case 98 . . . . . . . . . . . . . . . . . . . . . . . . . . . . . . . . . . . . . . . . . . . . . . . . . . . . . . . . . . . . . . 391 Kenneth A. Ellenbogen Case 99 . . . . . . . . . . . . . . . . . . . . . . . . . . . . . . . . . . . . . . . . . . . . . . . . . . . . . . . . . . . . . . 395 Amin Al-Ahmad and Paul J. Wang Case 100 . . . . . . . . . . . . . . . . . . . . . . . . . . . . . . . . . . . . . . . . . . . . . . . . . . . . . . . . . . . . . 399 Kenneth A. Ellenbogen Case 101 . . . . . . . . . . . . . . . . . . . . . . . . . . . . . . . . . . . . . . . . . . . . . . . . . . . . . . . . . . . . . 403 Amin Al-Ahmad and Paul J. Wang Case 102 . . . . . . . . . . . . . . . . . . . . . . . . . . . . . . . . . . . . . . . . . . . . . . . . . . . . . . . . . . . . . 405 Kenneth A. Ellenbogen Case 103 . . . . . . . . . . . . . . . . . . . . . . . . . . . . . . . . . . . . . . . . . . . . . . . . . . . . . . . . . . . . . 407 Amin Al-Ahmad and Paul J. Wang Case 104 . . . . . . . . . . . . . . . . . . . . . . . . . . . . . . . . . . . . . . . . . . . . . . . . . . . . . . . . . . . . . 411 Kenneth A. Ellenbogen Case 105 . . . . . . . . . . . . . . . . . . . . . . . . . . . . . . . . . . . . . . . . . . . . . . . . . . . . . . . . . . . . . 415 Kenneth A. Ellenbogen and Rod Bolanos Case 106 . . . . . . . . . . . . . . . . . . . . . . . . . . . . . . . . . . . . . . . . . . . . . . . . . . . . . . . . . . . . . 419 Kenneth A. Ellenbogen
xvi
Case 107 . . . . . . . . . . . . . . . . . . . . . . . . . . . . . . . . . . . . . . . . . . . . . . . . . . . . . . . . . . . . . 423 Paul A. Friedman and Charles D. Swerdlow Case 108 . . . . . . . . . . . . . . . . . . . . . . . . . . . . . . . . . . . . . . . . . . . . . . . . . . . . . . . . . . . . . 427 Paul A. Friedman and Charles D. Swerdlow Case 109 . . . . . . . . . . . . . . . . . . . . . . . . . . . . . . . . . . . . . . . . . . . . . . . . . . . . . . . . . . . . . 431 Paul A. Friedman and Charles D. Swerdlow Case 110 . . . . . . . . . . . . . . . . . . . . . . . . . . . . . . . . . . . . . . . . . . . . . . . . . . . . . . . . . . . . . 433 Paul A. Friedman and Charles D. Swerdlow Case 111 . . . . . . . . . . . . . . . . . . . . . . . . . . . . . . . . . . . . . . . . . . . . . . . . . . . . . . . . . . . . . 439 Paul A. Friedman and Charles D. Swerdlow Case 112 . . . . . . . . . . . . . . . . . . . . . . . . . . . . . . . . . . . . . . . . . . . . . . . . . . . . . . . . . . . . . 441 Paul A. Friedman and Charles D. Swerdlow Case 113 . . . . . . . . . . . . . . . . . . . . . . . . . . . . . . . . . . . . . . . . . . . . . . . . . . . . . . . . . . . . . 443 Paul A. Friedman and Charles D. Swerdlow Case 114 . . . . . . . . . . . . . . . . . . . . . . . . . . . . . . . . . . . . . . . . . . . . . . . . . . . . . . . . . . . . . 447 Paul A. Friedman and Charles D. Swerdlow Case 115 . . . . . . . . . . . . . . . . . . . . . . . . . . . . . . . . . . . . . . . . . . . . . . . . . . . . . . . . . . . . . 451 Paul A. Friedman and Charles D. Swerdlow Case 116 . . . . . . . . . . . . . . . . . . . . . . . . . . . . . . . . . . . . . . . . . . . . . . . . . . . . . . . . . . . . . 453 Paul A. Friedman and Charles D. Swerdlow Case 117 . . . . . . . . . . . . . . . . . . . . . . . . . . . . . . . . . . . . . . . . . . . . . . . . . . . . . . . . . . . . . 455 Paul A. Friedman and Charles D. Swerdlow Case 118 . . . . . . . . . . . . . . . . . . . . . . . . . . . . . . . . . . . . . . . . . . . . . . . . . . . . . . . . . . . . . 459 Paul A. Friedman and Charles D. Swerdlow Case 119 . . . . . . . . . . . . . . . . . . . . . . . . . . . . . . . . . . . . . . . . . . . . . . . . . . . . . . . . . . . . . 465 M. Eyman Mortada, Jasbir S. Sra, and Masood Akhtar Case 120 . . . . . . . . . . . . . . . . . . . . . . . . . . . . . . . . . . . . . . . . . . . . . . . . . . . . . . . . . . . . . 469 Paul A. Friedman and Charles D. Swerdlow Case 121 . . . . . . . . . . . . . . . . . . . . . . . . . . . . . . . . . . . . . . . . . . . . . . . . . . . . . . . . . . . . . 471 Kenneth A. Ellenbogen Case 122 . . . . . . . . . . . . . . . . . . . . . . . . . . . . . . . . . . . . . . . . . . . . . . . . . . . . . . . . . . . . . 473 Anurag Gupta Case 123 . . . . . . . . . . . . . . . . . . . . . . . . . . . . . . . . . . . . . . . . . . . . . . . . . . . . . . . . . . . . . 477 Ronald Lo, Amin Al-Ahmad, and Paul J. Wang
Contents
Contents
xvii
Case 124 . . . . . . . . . . . . . . . . . . . . . . . . . . . . . . . . . . . . . . . . . . . . . . . . . . . . . . . . . . . . . 481 Ronald Lo, Amin Al-Ahmad, and Paul J. Wang Case 125 . . . . . . . . . . . . . . . . . . . . . . . . . . . . . . . . . . . . . . . . . . . . . . . . . . . . . . . . . . . . . 483 Ronald Lo, Amin Al-Ahmad, and Paul J. Wang Section 3 Clinical Cases Case 126 . . . . . . . . . . . . . . . . . . . . . . . . . . . . . . . . . . . . . . . . . . . . . . . . . . . . . . . . . . . . . 487 Mehmet K. Aktas, Abrar H. Shah, and James P. Daubert Case 127 . . . . . . . . . . . . . . . . . . . . . . . . . . . . . . . . . . . . . . . . . . . . . . . . . . . . . . . . . . . . . 491 Loren P. Budge and John P. DiMarco Case 128 . . . . . . . . . . . . . . . . . . . . . . . . . . . . . . . . . . . . . . . . . . . . . . . . . . . . . . . . . . . . . 495 David J. Callans Case 129 . . . . . . . . . . . . . . . . . . . . . . . . . . . . . . . . . . . . . . . . . . . . . . . . . . . . . . . . . . . . . 497 Andrew E. Darby and John P. DiMarco Case 130 . . . . . . . . . . . . . . . . . . . . . . . . . . . . . . . . . . . . . . . . . . . . . . . . . . . . . . . . . . . . . 501 Thomas J. Sawyer, Burr W. Hall, and James P. Daubert Case 131 . . . . . . . . . . . . . . . . . . . . . . . . . . . . . . . . . . . . . . . . . . . . . . . . . . . . . . . . . . . . . 505 John P. DiMarco Case 132 . . . . . . . . . . . . . . . . . . . . . . . . . . . . . . . . . . . . . . . . . . . . . . . . . . . . . . . . . . . . . 509 John P. DiMarco Case 133 . . . . . . . . . . . . . . . . . . . . . . . . . . . . . . . . . . . . . . . . . . . . . . . . . . . . . . . . . . . . . 511 John P. DiMarco Case 134 . . . . . . . . . . . . . . . . . . . . . . . . . . . . . . . . . . . . . . . . . . . . . . . . . . . . . . . . . . . . . 513 John P. DiMarco Case 135 . . . . . . . . . . . . . . . . . . . . . . . . . . . . . . . . . . . . . . . . . . . . . . . . . . . . . . . . . . . . . 517 John P. DiMarco Case 136 . . . . . . . . . . . . . . . . . . . . . . . . . . . . . . . . . . . . . . . . . . . . . . . . . . . . . . . . . . . . . 521 John P. DiMarco Case 137 . . . . . . . . . . . . . . . . . . . . . . . . . . . . . . . . . . . . . . . . . . . . . . . . . . . . . . . . . . . . . 523 John P. DiMarco Case 138 . . . . . . . . . . . . . . . . . . . . . . . . . . . . . . . . . . . . . . . . . . . . . . . . . . . . . . . . . . . . . 527 John P. DiMarco Case 139 . . . . . . . . . . . . . . . . . . . . . . . . . . . . . . . . . . . . . . . . . . . . . . . . . . . . . . . . . . . . . 529 John P. DiMarco
xviii
Case 140 . . . . . . . . . . . . . . . . . . . . . . . . . . . . . . . . . . . . . . . . . . . . . . . . . . . . . . . . . . . . . 531 John P. DiMarco Case 141 . . . . . . . . . . . . . . . . . . . . . . . . . . . . . . . . . . . . . . . . . . . . . . . . . . . . . . . . . . . . . 535 John P. DiMarco Case 142 . . . . . . . . . . . . . . . . . . . . . . . . . . . . . . . . . . . . . . . . . . . . . . . . . . . . . . . . . . . . . 539 John P. DiMarco Case 143 . . . . . . . . . . . . . . . . . . . . . . . . . . . . . . . . . . . . . . . . . . . . . . . . . . . . . . . . . . . . . 545 John P. DiMarco Case 144 . . . . . . . . . . . . . . . . . . . . . . . . . . . . . . . . . . . . . . . . . . . . . . . . . . . . . . . . . . . . . 551 John P. DiMarco Case 145 . . . . . . . . . . . . . . . . . . . . . . . . . . . . . . . . . . . . . . . . . . . . . . . . . . . . . . . . . . . . . 555 John P. DiMarco Case 146 . . . . . . . . . . . . . . . . . . . . . . . . . . . . . . . . . . . . . . . . . . . . . . . . . . . . . . . . . . . . . 559 Brett A. Faulknier, David T. Huang, and James P. Daubert Case 147 . . . . . . . . . . . . . . . . . . . . . . . . . . . . . . . . . . . . . . . . . . . . . . . . . . . . . . . . . . . . . 565 Stefan H. Hohnloser and Joachim R. Ehrlich Case 148 . . . . . . . . . . . . . . . . . . . . . . . . . . . . . . . . . . . . . . . . . . . . . . . . . . . . . . . . . . . . . 567 Joachim Ehrlich and Stefan H. Hohnloser Case 149 . . . . . . . . . . . . . . . . . . . . . . . . . . . . . . . . . . . . . . . . . . . . . . . . . . . . . . . . . . . . . 571 Bradley P. Knight Case 150 . . . . . . . . . . . . . . . . . . . . . . . . . . . . . . . . . . . . . . . . . . . . . . . . . . . . . . . . . . . . . 575 Bradley P. Knight Case 151 . . . . . . . . . . . . . . . . . . . . . . . . . . . . . . . . . . . . . . . . . . . . . . . . . . . . . . . . . . . . . 577 Bradley P. Knight Case 152 . . . . . . . . . . . . . . . . . . . . . . . . . . . . . . . . . . . . . . . . . . . . . . . . . . . . . . . . . . . . . 579 Andrew D. Krahn Case 153 . . . . . . . . . . . . . . . . . . . . . . . . . . . . . . . . . . . . . . . . . . . . . . . . . . . . . . . . . . . . . 583 Byron K. Lee, Melvin M. Scheinman, and Zian H. Tseng Case 154 . . . . . . . . . . . . . . . . . . . . . . . . . . . . . . . . . . . . . . . . . . . . . . . . . . . . . . . . . . . . . 585 Srijoy Mahapatra Case 155 . . . . . . . . . . . . . . . . . . . . . . . . . . . . . . . . . . . . . . . . . . . . . . . . . . . . . . . . . . . . . 589 Pamela K. Mason Case 156 . . . . . . . . . . . . . . . . . . . . . . . . . . . . . . . . . . . . . . . . . . . . . . . . . . . . . . . . . . . . . 593 Pamela K. Mason
Contents
Contents
xix
Case 157 . . . . . . . . . . . . . . . . . . . . . . . . . . . . . . . . . . . . . . . . . . . . . . . . . . . . . . . . . . . . . 595 Pamela K. Mason Case 158 . . . . . . . . . . . . . . . . . . . . . . . . . . . . . . . . . . . . . . . . . . . . . . . . . . . . . . . . . . . . . 597 Pamela K. Mason Case 159 . . . . . . . . . . . . . . . . . . . . . . . . . . . . . . . . . . . . . . . . . . . . . . . . . . . . . . . . . . . . . 599 Lisa G. Umphrey and John Paul Mounsey Case 160 . . . . . . . . . . . . . . . . . . . . . . . . . . . . . . . . . . . . . . . . . . . . . . . . . . . . . . . . . . . . . 603 James A. Reiffel Case 161 . . . . . . . . . . . . . . . . . . . . . . . . . . . . . . . . . . . . . . . . . . . . . . . . . . . . . . . . . . . . . 607 James A. Reiffel Case 162 . . . . . . . . . . . . . . . . . . . . . . . . . . . . . . . . . . . . . . . . . . . . . . . . . . . . . . . . . . . . . 611 Jens Seiler and William G. Stevenson Case 163 . . . . . . . . . . . . . . . . . . . . . . . . . . . . . . . . . . . . . . . . . . . . . . . . . . . . . . . . . . . . . 617 Jens Seiler and William G. Stevenson Case 164 . . . . . . . . . . . . . . . . . . . . . . . . . . . . . . . . . . . . . . . . . . . . . . . . . . . . . . . . . . . . . 625 Jens Seiler and William G. Stevenson Case 165 . . . . . . . . . . . . . . . . . . . . . . . . . . . . . . . . . . . . . . . . . . . . . . . . . . . . . . . . . . . . . 631 J. Jason West and John Paul Mounsey Case 166 . . . . . . . . . . . . . . . . . . . . . . . . . . . . . . . . . . . . . . . . . . . . . . . . . . . . . . . . . . . . . 635 Darren Traub, James P. Daubert, and Spencer Rosero Case 167 . . . . . . . . . . . . . . . . . . . . . . . . . . . . . . . . . . . . . . . . . . . . . . . . . . . . . . . . . . . . . 639 Vikas P. Kuriachan and George D. Veenhuyzen Case 168 . . . . . . . . . . . . . . . . . . . . . . . . . . . . . . . . . . . . . . . . . . . . . . . . . . . . . . . . . . . . . 643 Vikas P. Kuriachan and George D. Veenhuyzen Case 169 . . . . . . . . . . . . . . . . . . . . . . . . . . . . . . . . . . . . . . . . . . . . . . . . . . . . . . . . . . . . . 647 Jeffrey D. Booker and George D. Veenhuyzen Case 170 . . . . . . . . . . . . . . . . . . . . . . . . . . . . . . . . . . . . . . . . . . . . . . . . . . . . . . . . . . . . . 649 M. Eyman Mortada, Jasbir S. Sra, and Masood Akhtar Case 171 . . . . . . . . . . . . . . . . . . . . . . . . . . . . . . . . . . . . . . . . . . . . . . . . . . . . . . . . . . . . . 653 M. Eyman Mortada, Jasbir S. Sra, and Masood Akhtar Index . . . . . . . . . . . . . . . . . . . . . . . . . . . . . . . . . . . . . . . . . . . . . . . . . . . . . . . . . . . . . . . . 655
Section Ablation
I
Case 1 Amin Al-Ahmad
Case Summary A 72-year-old male with a history of aortic sinus of valsalva rupture and repair 30 years prior presents with palpitations. The patient reports having symptoms of shortness of breath and fatigue with palpitations. He is currently taking only aspirin. He has had cardioversions in the past for this arrhythmia and would like to consider ablation. He does not want to take anti-arrhythmic medications. His 12-lead ECG (Fig. 1.1) shows a 2:1 atrial tachyarrhythmia at a rate of approximately 180 bpm. The P-waves in the inferior leads are negative; however there is an isoelectric segment between P-waves.
What maneuvers would be important to elucidate the diagnosis during electrophysiology study and ablation?
Case Discussion This tachycardia may represent a focal atrial tachycardia given the isoelectric segment on the 12-lead ECG. However, in patients with prior cardiac surgery, atrial flutters should be considered. The possibility that this tachycardia is typical isthmus-dependent atrial flutter should be explored using entrainment pacing at the tricuspid valve (TV) to inferior
Fig. 1.1 Twelve-lead ECG showing atrial arrhythmia. Note the negative atrial deflections in the inferior leads and the isoelectric segment between them
A. Al-Ahmad School of Medicine, Stanford University, 300 Pasteur Drive, H-2146, Stanford, CA 94305 A. Natale et al. (eds.), Cardiac Electrophysiology, DOI: 10.1007/978-1-84996-390-9_1, © Springer-Verlag London Limited 2011
3
4
A. Al-Ahmad
vena cava (IVC) isthmus. Reverse typical atrial flutter would be unlikely given the morphology of the flutter waves. Scarrelated atrial flutters should also be considered. Ablation of all potential atrial flutter circuits in patients post cardiac surgery may reduce the likelihood of recurrence.1
In this case, entrainment at the TV-IVC isthmus demonstrated a near perfect PPI and ablation at the TV-IVC isthmus terminated the atrial flutter (Figs. 1.2 and 1.3). The isoelectric segment between flutter waves likely represents slow atrial conduction in atrial flutter rather than atrial tachycardia.
200ms
I aVF V6 hRA p HIS p HIS d CS 9,10 CS 7,8 CS 5,6 CS 3,4 CS 1,2 RV p ABL
PPI
TCL
ABL d Stim 4
12:22:16 PM
12:22:17 PM
Fig. 1.2 Pacing using the ablation catheter positioned at the tricuspid valve-inferior vena cava (TV-IVC) isthmus. Concealed entrainment and a near perfect post-pacing interval are noted
Case 1
5 500ms
I
aVF hRA p HIS p HIS d CS 9,10 CS 7,8 CS 5,6 CS 3,4 CS 1,2 RV p ABL ABL d Stim 4
1:01:47 PM
1:01:48 PM
1:01:49 PM
Fig. 1.3 Application of RF energy in the TV-IVC area leads to termination of the arrhythmia
Reference 1. Verma A, Marrouche NF, Seshadri N, Schweikert RA, Bhargava M, Burkhardt JD, Kilicaslan F, Cummings J, Saliba W, Natale A. Importance of ablating all potential right atrial flutter circuits in postcardiac surgery patients. J Am Coll Cardiol. July 2004;44(2): 409–414.
1:01:50 PM
1:01:51 PM
Case 2 Michel Haissaguerre
Case Summary A 47-year-old male with a 4-year history of symptomatic, drug-resistant lone paroxysmal atrial fibrillation (AF) was referred for a first ablation procedure. He suffered from daily AF episodes that lasted a maximum of 8 h. Episodes of
AF always started following monomorphic atrial ectopy (Fig. 2.1). A decapolar catheter (Xtrem, ELA Medical, Le-PlessisRobinson, France) was inserted inside the coronary sinus (CS) while the ablation catheter (Thermocool Biosense Webster, Diamond Bar, CA) and a decapolar circumferential
I II III aVR aVL aVF V1 V2 V3 V4 V5 V6
Fig. 2.1 Twelve-lead ECG. Sinus rhythm and short coupling atrial ectopies (with functional left bundle branch block [LBBB])
M. Haissaguerre Department of Cardiology and Electrophysiology, CHU de Bordeaux, Hospital Cardiologique de Haut Leveque, Avenue de Magellan, Pessac 33604, France A. Natale et al. (eds.), Cardiac Electrophysiology, DOI: 10.1007/978-1-84996-390-9_2, © Springer-Verlag London Limited 2011
7
8
M. Haissaguerre
catheter (Lasso, Biosense Webster, Diamond Bar, CA) were introduced through a long sheath in the left atrium (LA). The circumferential catheter was placed in the left superior pulmonary vein (LSPV) and recorded venous and atrial potentials (Fig. 2.2). What mechanism is illustrated and what action is required?
Case Discussion Figure 2.1 shows a recording of atrial ectopics with a short coupling interval (also note the functional left bundle branch block [LBBB]). The P-wave morphology during ectopy is flat in the lateral leads, positive in the inferior leads and in lead V1, suggesting they originate from the LSPV. Endocardial tracings from the LSPV (Fig. 2.2) show two separated potentials during sinus rhythm (Fig. 2.2, first complex). The first potential (white star) represents activation of
the adjacent LA and is synchronous with the second half of the P-wave (in the right PVs it should be the first part of the P-wave). The second potential reflects local activity from the PV striated musculature (black star). When ectopy occurs in the PV (Fig. 2.2, second complex), there is a reversal of the described activation sequence, with the PV potential preceding the atrial potential. This pattern of reverse activation in a dead-end structure during ectopic triggered AF evidence for the arrhythmogenic potential of that PV. Mapping of the earliest site of activity during ectopy allows identification of discrete sites inside the vein, while the atrial exit site is dependent on the anatomy of the PV-LA connecting fascicles. Given that arrhythmia recurrence can occur from either the pulmonary vein that is active at the time of the procedure or any other PV, complete electrical isolation of all PVs has to be carried out with a series of coalescent RF applications using a dedicated PV circumferential catheter to help with mapping. Ablation is performed outside the vein (within 1–2 cm of the PV ostia) for right PVs and for the posterior part
I II V1
PV 1-2 PV 2-3 PV 3-4 PV 4-5 PV 5-6 PV 6-7 PV 7-8 PV 8-9 PV 9-10 CS 1-2 CS 3-4 CS 5-6 CS 7-8 CS 9-10
Fig. 2.2 Endocardial tracings from the left superior pulmonary vein (LSPV). Recording from a decapolar circumferential catheter inserted inside the LSPV (PV 1–2 to PV 9–10) and a decapolar catheter inserted into the coronary sinus (CS 1–2 to CS 9–10). During sinus rhythm (first
complex), there existed the presence of two separated potentials (one atrium, white star followed by one venous, black star). During ectopy (second complex), reversal of the described activation sequence with the PV potential preceding the atrial potential occurred
Case 2
9
of the left PVs; however, due to the ridge between the left pulmonary veins and the left atrial appendage (LAA), catheter stability is an issue for ablation of the anterior aspect of the left PV’s, and ablation is often within 1 mm of the veins. In this case, ablation was started at the low anterior LSPV (pole 5) where the earliest activity was located and where a reverse in PV polarity was observed, both criteria pointing at a local anatomical connection (Fig. 2.3, panel A). Ablation at this point delayed the venous potentials (Fig. 2.3, panel B), and a second anatomical breakthrough was subsequently ablated at the upper part of the vein (pole 1). The ectopic beats stopped (Fig. 2.3, panel C) and the venous potentials became dissociated (Fig. 2.4).Ablation of the left inferior PV (LIPV) was performed in the same way. For the right veins,
A
segmental or circumferential ablation with a continuous circular lesion can be performed depending on the operator’s preference. When doing continuous circumferential lesions, it is unusual to achieve PV isolation without further ablation targeting the earliest PV activity or sites of reverse PV polarity as recorded on the circumferential mapping catheter, as in Fig. 2.2, indicating a residual anatomical connection on the line of ablation. After ablation, nine attempts at induction (with bursts up to a cycle length of 200 ms) at three different places (CS and both appendages) could not induce sustained arrhythmia, predicting a favorable clinical outcome. This case illustrates a typical ablation of paroxysmal AF where it was clearly demonstrated that the arrhythmogenic ectopic beats triggering AF originated from the LSPV.
B
C
I II
V1
PV 1-2 PV 2-3 PV 3-4 PV 4-5 PV 5-6 PV 6-7 PV 7-8 PV 8-9 PV 9-10 CS 1-2 CS 3-4 CS 5-6 CS 7-8 CS 9-10
Fig. 2.3 Endocardial tracings recorded during sinus rhythm from the left superior pulmonary (LSPV) vein. Recording from a decapolar circumferential catheter inserted inside the LSPV (PV 1–2 to PV 9–10) and a decapolar catheter inserted into the coronary sinus (CS 1–2 to CS
9–10). During ablation targeting the earliest venous potential (black star), progressive slowing of the conduction to the vein (panel A and B) to the complete block (panel C)
10
M. Haissaguerre
I
17
18
19
20
21
22
23
24
25
26
II
V1
PV 1-2 PV 2-3 PV 3-4 PV 4-5 PV 5-6 PV 6-7 PV 7-8 PV 8-9 PV 9-10 CS 1-2 CS 3-4 CS 5-6 CS 7-8 CS 9-10
Fig. 2.4 Endocardial tracings recorded during sinus rhythm from the left superior pulmonary vein (LSPV). Recording from a decapolar circumferential catheter inserted inside the LSPV (PV 1–2 to PV 9–10)
and a decapolar catheter inserted into the coronary sinus (CS 1–2 to CS 9–10). Dissociation of the venous potential (black star) with a slow automatic activity
Bibliography
Haïssaguerre M, Hocini M, Sanders P, et al. Localized sources maintaining atrial fibrillation organized by prior ablation. Circulation. 2006;113(5):616-625. Sanders P, Hocini M, Jais P, et al. Characterization of focal atrial tachycardia using high-density mapping. J Am Coll Cardiol. 2005 December 6;46(11):2088-2099. Mohamed U, Skanes AC, Gula LJ, et al. A novel pacing maneuver to localize focal atrial tachycardia. J Cardiovasc Electrophys. 2007;18(1):1-6.
Haïssaguerre M, Shah DC, Jaïs P, et al. Electrophysiological breakthroughs from the left atrium to the pulmonary veins. Circulation. 2000;102:2463-2465. Haissaguerre M, Sanders P, Hocini M, et al. Catheter ablation of longlasting persistent atrial fibrillation: critical structures for termination. J Cardiovasc Electrophys. 2005;16(11):1125-1137.
Case 3 Amin Al-Ahmad
Case Summary A 51-year-old male with a history of pulmonary fibrosis and a single lung transplant is found to have atrial flutter on routine follow-up. Initially he is unaware of this rhythm, but later recalls an increase in fatigue over the previous week. He is referred for evaluation by an electrophysiologist. A Holter monitor shows his heart rate to be approximately 90 on average. He has periods of normal sinus rhythm and periods of atrial flutter. A 12-lead ECG during atrial flutter is shown in Fig. 3.1. Over the next few months his atrial flutter becomes persistent and he continues to complain of fatigue. He is talking multiple medications post transplant to prevent rejection and has some renal insufficiency. Drug therapy with anti-
arrhythmic medications does not seem to be an attractive option given his multiple co-morbid conditions. And despite better rate control he remains symptomatic and is taken to the electrophysiology suite. Where is this tachycardia origin, and what pacing maneuvers may help determine the best area to ablate?
Case Discussion Atrial flutter can be common immediately after lung transplantation. Canine models have suggested a substrate for atrial flutter in the left atrium due to suture lines. In addition,
Fig. 3.1 Twelve-lead ECG of atrial flutter. Note the peaked and strongly positive flutter waves in lead V1
A. Al-Ahmad School of Medicine, Stanford University, 300 Pasteur Drive, H-2146, Stanford, CA 94305 A. Natale et al. (eds.), Cardiac Electrophysiology, DOI: 10.1007/978-1-84996-390-9_3, © Springer-Verlag London Limited 2011
11
12
A. Al-Ahmad
right-sided atrial flutters may also develop as these patients can commonly have right atrial dilation due to potentially long-standing increased right-sided pressures. To differentiate right- and left-sided atrial flutter, the flutter wave morphology can be useful. In this case a strongly positive flutter wave in the anterior precordial leads suggests that the flutter is left-sided. In addition, entrainment from areas on the right side can be done quickly and can help
determine if the flutter is left-sided or right-sided. Entrainment can further distinguish the critical isthmus for the atrial flutter and help guide ablation. In this case, entrainment on the right side clearly demonstrated that the flutter was not right-sided (Fig. 3.2). Entrainment in the left atrium near the right inferior pulmonary vein was near perfect. Ablation in that area to create a line to the mitral valve caused the flutter to terminate (Figs. 3.3 and 3.4).
200ms
I II V1 V6
S
RA 9,10 RA 7,8 RA 5,6
PPI
RA 3,4
TCL
RA 1,2 CS 9,10 CS 7,8 CS 5,6 CS 3,4 CS 1,2 HIS p HIS m
C
HIS d ABL ABL d Stim 3
S1
S1
S1 9:42:27 AM
S1
S1 9:42:28 AM
Fig. 3.2 Pacing in the right atrium shows a long post-pacing interval. This was the case from multiple areas in the right atrium
Case 3
13 200ms
I II V1 V6 RA 9,10 RA 7,8 RA 5,6 RA 3,4 RA 1,2 CS 9,10 CS 7,8 CS 5,6
T
CS 3,4 CS 1,2
ABL ABL d Stim 4
PPI
S1 S1 12:55:47 PM
S1
TCL 12:55:48 PM
12:55:49 PM
Fig. 3.3 Pacing in the left atrium near the right inferior pulmonary vein yields a near-perfect PPI
Bibliography Nielsen TD, Bahnson T, Davis RD, Palmer SM. Atrial fibrillation after pulmonary transplant. Chest. August 2004;126(2):496-500.
Fig. 3.4 Right anterior oblique view of the ablation catheter in the left atrium near the right inferior pulmonary vein. CS coronary sinus, ICE intracardiac echo, ABL ablation catheter
Case 4 Michel Haissaguerre
Case Summary A 56-year-old male was referred for ablation of persistent drug-resistant atrial fibrillation (AF). He had a 17-year history of AF, which had been persistent for the last 12 months. The atrial fibrillation cycle length (AFCL) measured during electrophysiological study was 154, 158, and 152 ms on surface ECG, left and right atrial appendages, respectively (Fig. 4.1). Our approach in persistent AF is to first isolate the pulmonary veins (PVs), before targeting complex fractionated atrial
electrograms (CFAEs) and finally performing linear lesions. Any resulting atrial tachycardias (ATs) are mapped and ablated. In this case, following PV isolation there was prolongation of the AFCL to 165 and 159 ms at left and right appendages, respectively. A NavX fractionation map of the LA was performed using a 20-pole high-density mapping catheter. CFAEs were distributed throughout the LA and the CS, as shown in Fig. 4.2. In cases like this, which CFAE should be targeted, and what is the value in measuring AFCL?
I II V1
LAA
Fig. 4.1 Recording from left (LAA) and right (RAA) atrial appendages, and the coronary sinus (CS 1–2 to 9–10). While fractionation is present in the coronary sinus, preventing an easy measurement of the cycle length, potentials at the top of both appendages are almost always of high voltage and unambiguous
RAA CSd
CSp
M. Haissaguerre Department of Cardiology and Electrophysiology, CHU de Bordeaux, Hospital Cardiologique de Haut Leveque, Avenue de Magellan, Pessac 33604, France A. Natale et al. (eds.), Cardiac Electrophysiology, DOI: 10.1007/978-1-84996-390-9_4, © Springer-Verlag London Limited 2011
15
16
M. Haissaguerre 7CFE Mean Map
120 mS 65 mS
0 mS
Fig. 4.2 NavX (EnSite System, St. Jude Medical, MN, USA) fractionation map of the left atrium and coronary sinus after isolation of the four pulmonary veins showing ubiquitous distribution of complex fractionated electrograms
Case Discussion Atrial electrograms in persistent AF are complex and cannot be reliably evaluated in terms of CL except in either right or left appendage. We routinely place a catheter in the right atrial appendage (RAA) and LAA to measure simultaneously the left and right AFCLs at the start of the procedure (Fig. 4.1). Using custom analysis software (Bard EP, Lowell, MA, USA), the mean AFCL for the selected window is calculated. The electrogram annotation is then verified manually online to ensure accuracy (Fig. 4.1). The electrograms measured at the top of both appendages are almost always discrete and of high amplitude, thereby facilitating unambiguous automatic annotation. In the absence of this software, the mean AFCL can be easily estimated by measuring the total duration required for 10–30 cycles (or longer) and then dividing that number by the number of cycles. Often, the AFCL is also reasonably estimated from V1 on the 12-lead ECG. Following PV isolation and throughout the procedure, the circumferential mapping catheter is placed within the RAA, and the ablation catheter in the LAA, and the AFCL may then be assessed simultaneously to measure the impact of ablation in both chambers. Initial AFCL has been shown to be the strongest predictor of success for AF ablation (AF of less than 5 years
continuous duration). AFCL of less than 140 ms is associated with AF termination in less than 69%, while a higher AFCL like in our case is associated with more than 89% of AF termination. Furthermore, the impact of ablation of each region during electrogram-based ablation can be quantified in both atria by AFCL monitoring. After each step of ablation, a gradual prolongation of AFCL is usually observed (a change in AFCL greater than 6 ms is considered significant). Conversion to sinus rhythm or AT occurs when AFCL reaches between 180 and 200 ms and, conversely, rarely occurs when the AFCL is shorter than this. Concerning the appropriate sequence of ablation during the electrogram-based part of the procedure, the main issue is to distinguish active from passive patterns, which still remains one of the major obstacles to minimal effective ablation of persistent AF. This is particularly striking when there are multiple CFAEs distributed throughout the atrium, as in this case. From a purely anatomic perspective, in addition to PVs, ablation at structures annexed to the LA, namely the interface between the inferior LA and the CS and the base of the LAA, have been shown to have the greatest impact on the AF, as measured by AFCL.1 The endpoint of ablation during electrogram-based ablation remains imprecise; however, the organization and slowing of local potentials by ablation seems for us to be preferable to complete elimination of local potentials. In this case, following PV isolation, ablation started along the inferior LA where continuous activity was recorded (Fig. 4.3, panel A). Ablation at the endocardial interface of the CS aims at interrupting the muscular fascicles connecting the LA and the CS and organizing the chaotic activity recorded within the CS. This resulted in a prolongation of the AFCL to 171 ms in the LAA and 168 ms in the RAA. The following step consisted of the ablation at the posterior part of the LAA, where consistent distal-to-proximal electrograms suggesting centrifugal activation were recorded (Fig. 4.3, panel B); this resulted in a prolongation of both AFCLs to 176 ms. Ablation was performed along the roof of the LA where almost continuous electrograms were recorded (Fig. 4.3, panel C), and the AFCL was prolonged to 188 and 183 ms in the right and left appendages, respectively. Finally, ablation of continuous electrical activity at the anteroseptal LA resulted in restoration of sinus rhythm (Fig. 4.4). During sinus rhythm, PV isolation was confirmed and the roofline was completed.
Case 4
Fig. 4.3 Recording of the ablation catheter (RF) and the coronary sinus (CS 1–2 to 9–10). Panel A: Ablation along the inferior left atrium targeting continuous electrical activity. Panel B: Ablation at the posterior part of the left atrial appendage targeting centrifugal activation with
17
consistent distal-to-proximal fractionated electrograms. Panel C: Ablation was then performed along the roof targeting almost continuous fractionated electrograms
18
M. Haissaguerre
I
I
II II V1
V1
RFp CS 1-2
RFd CS 3-4
CS 1-2
CS 5-6
CS 3-4 CS 7-8 CS 5-6 CS 7-8 CS 9-10 CS 9-10
Fig. 4.4 Recording of the ablation catheter (RF) and the coronary sinus (CS 1–2 to 9–10). Ablation of fractionated electrical activity at the anterseptal LA (left panel) resulted in direct restoration of sinus rhythm (right panel, black star)
Bibliography Haïssaguerre M, Shah DC, Jaïs P, et al. Electrophysiological breakthroughs from the left atrium to the pulmonary veins. Circulation. 2000;102:2463-2465. Haissaguerre M, Sanders P, Hocini M, et al. Catheter ablation of longlasting persistent atrial fibrillation: critical structures for termination. J Cardiovasc Electrophys. 2005;16(11):1125-1137.
Haïssaguerre M, Hocini M, Sanders P, et al. Localized sources maintaining atrial fibrillation organized by prior ablation. Circulation. 2006;113(5):616-625. Mohamed U, Skanes AC, Gula LJ, et al. A novel pacing maneuver to localize focal atrial tachycardia. J Cardiovasc Electrophys. 2007; 18(1):1-6. Sanders P, Hocini M, Jais P, et al. Characterization of focal atrial tachycardia using high-density mapping. J Am Coll Cardiol. 6 December 2005;46(11):2088-2099.
Case 5 Anurag Gupta and Amin Al-Ahmad
Case Summary A 21-year-old male with Wolf–Parkinson–White syndrome is referred for electrophysiology study given worsening episodes of palpitations. A 12-lead electrocardiogram (ECG) during sinus rhythm shows pre-excitation (Fig. 5.1). Diagnostic electrode catheters are then placed for recording in the His bundle area and right ventricle, as well as a 20-pole Halo catheter for recording in the right atrium adjacent to the tricuspid valve annulus. Ventricular pacing shows the retrograde atrial
activation sequence (Fig. 5.2). Can this patient be treated with a single application of radiofrequency energy? What maneuvers may be helpful in determining this?
Case Discussion Analysis of the delta waves) on the 12-lead ECG during sinus rhythm (Fig. 5.1), including R/S 15% of the CL (inset ECG) suggesting, in fact, focal AT. Furthermore, post pacing interval (PPI) at the posterior wall was short (+20 ms), but the anterior PPI was very long (+200 ms), ruling out macro re-entrant roof-dependant AT. Since the roofline was previously blocked, it suggested a focal origin from the high posterior LA. Mapping this area close to the roof line revealed very low voltage local activity spanning most of the CL at three different spots very close to each other (Fig. 6.3, POST 1–3). This voltage was only 0.04 mV, which can be easily hidden in case of electrical noise. Ablation of a very low voltage continuous activity site located in between the three described spots terminated the tachycardia (Fig. 6.4). This case emphasizes the importance of localized re-entrant AT in the context of prior AF ablation. In this context, 53% of AT are focal, and localized re-entries are the
most frequent AT mechanism, representing 71% of focal ATs (and 37% of total ATs). The preferential regions for focal ATs are the PV–LA junction, left septum, and the mouth of the LAA. Focal ATs are generally localized to sites targeted during electrogram-based AF ablation. Injury or oedema to the atrial tissue induced by RF applications could generate the substrate for further arrhythmia by creating an anchoring point potentially able to maintain re-entry. Therefore the mechanism of AT after AF ablation may be different to that of spontaneous AT. The presence of pre-existing LA scar (which may be idiopathic or related to underlying structural heart disease) may also result in local slow conduction areas predisposing to re-entry. Mapping and ablation of focal or macro re-entrant AT is a crucial step in the AF ablation process, and often represents the difference between procedural success and failure.
26
M. Haissaguerre
Fig. 6.4 Same graphic as Fig. 6.2. Ablation catheter positioned a nterior and septal to the roofline
LAA LPV
Septum
RPV
CS
V1
RFd
0, 100 mV RFd RFp
CSd 100 ms
V1
0, 100 mV RFd
RFd1
LAA
POST RFd2
pt um Se
Fig. 6.5 Scanning the area within 10–15 mm of the site showed in Fig. 6.4: Electrograms show activation compatible with the entire circuit (initial part in RF2, second part in RF3 and throughout the circuit in RF1). These data are most compatible with left atrial activation coming centrifugally from a small re-entrant source central to the excursion of the catheter in this small area
RPVs
RFd3
CS
CSd
100 ms
LPVs
Case 6
Bibliography Haïssaguerre M, Hocini M, Sanders P, et al. Localized sources maintaining atrial fibrillation organized by prior ablation. Circulation. 2006;113(5):616-625. Haïssaguerre M, Shah DC, Jaïs P, et al. Electrophysiological breakthroughs from the left atrium to the pulmonary veins. Circulation. 2000;102:2463-2465.
27 Haissaguerre M, Sanders P, Hocini M, et al. Catheter ablation of longlasting persistent atrial fibrillation: critical structures for termination. J Cardiovasc Electrophys. 2005;16(11):1125-1137. Sanders P, Hocini M, Jais P, et al. Characterization of focal atrial tachycardia using high-density mapping. J Am Coll Cardiol. 6 December 2005;46(11):2088-2099. Mohamed U, Skanes AC, Gula LJ, et al. A novel pacing maneuver to localize focal atrial tachycardia. J Cardiovasc Electrophys. 2007;18(1):1-6.
Case 7 Michel Haissaguerre
Case Summary A 67-year-old male with an 8-year history of persistent atrial fibrillation (AF) and three external direct current (DC) cardioversions was referred for ablation. The inital procedure consisted of pulmonary vein (PV) isolation, extensive ablation based on fractionated electrograms in the left atrium (LA) and the right atrium (RA), and linear lesions at the roof and the left mitral isthmus. AF was not terminated and an external cardioversion was required. Linear lesions (left atrial roofline and mitral isthmus line) were completed during sinus rhythm. Early recurrence of AT necessitated a second ablation procedure one month later. A decapolar catheter was placed into the CS and showed a proximal-to-distal activation pattern. A quadripolar cooled-tip ablation catheter was then inserted into the LA. AT cycle length was 260 ms with 10% irregularity. PV isolation was confirmed. Extensive anterior scar was noted, and anterior endocardial activation was difficult to assess, but it appeared to be septal to lateral. Post pacing interval (PPI) at the RA was long (+130 ms), PPI at the posterior LA was long (+120 ms). PPI in the LAA was long (+60 ms), while PPI at the anterior LA was good (+10 ms). The anterior part of the LAA showed almost continuous activity (Fig. 7.1). Can this site be considered for ablation?
Case Discussion In the case of extensive scarring or ablation, mapping of AT can be challenging with the practical algorithm. In this situation, three methods can be used to facilitate identification
and ablation of the AT (Fig. 7.2). Entrainment manoeuvres performed during the arrhythmia point towards a focal origin by demonstrating long return cycle lengths around the mitral annulus, the roofline or the tricuspid annulus except at sites adjacent to the focus (by definition, less than two segments). For localized re-entry, local activity spanning all of the AT CL can be demonstrated at the area of earliest activity. For focal point AT, an area of early activity may exhibit middiastolic potential. These tracings illustrate a case with prior extensive LA ablation including the left mitral isthmus line. Conduction around the mitral annulus was compatible with a perimitral circuit (lateral to septal anteriorly and septal-to-lateral posteriorly). The PPI was 30 ms at the CS, which made a macro re-entrant peri-mitral circuit unlikely and suggested a focal AT located lateral to the complete left mitral isthmus line. Mapping was performed in this area where a site displaying most of the CL was found at the anterior mouth of the LAA (Fig. 7.1); however, the PPI was 100 ms at this site, suggesting it was far from the origin of the AT. A second site displaying most of the CL with an activation gradient on the electrogram recording and a short PPI (10 ms) was mapped on the anterior LA (Fig. 7.3). A threedimensional reconstruction of this spot with activation mapping confirmed that the activity spanned all the cycle lengths in a very localized region (Fig. 7.4). Ablation at this site restored sinus rhythm, and a local double potential was visualized after ablation (Fig. 7.5, arrows) in sinus rhythm. This case illustrates how useful the PPI can be in difficult cases. During entrainment, attention must be paid to avoid induction of AF at short CLs and one must be aware of the possibility of conversion to another AT.
M. Haissaguerre Department of Cardiology and Electrophysiology, CHU de Bordeaux, Hospital Cardiologique de Haut Leveque, Avenue de Magellan, Pessac 33604, France A. Natale et al. (eds.), Cardiac Electrophysiology, DOI: 10.1007/978-1-84996-390-9_7, © Springer-Verlag London Limited 2011
29
30
M. Haissaguerre I
55
56
II V1
RFd
RFp
CS 1-2 CS 3-4 CS 5-6 CS 7-8 CS 9-10
Fig. 7.1 Recordings of the radiofrequency RF ablation catheter and a decapolar catheter inserted into the coronary sinus (CS 1–2 to 9–10). Site of high-voltage continuous electrograms displaying almost all the cycle length
PPI to assess number of segment(s) involved
Fig. 7.2 Approach to atrial tachycardia in the context of atrial fibrillation (AF), in the case of extensive scar or ablation. See text for details
≤2 segments FOCAL
>2 segments MACRO RE-ENTRY
• EGM spanning all the AT cycle length in the involved segment(s) • Mid diastolic potentials
Activation compatible with: • Perimitral macro re-entry • Roof dependant macro re-entry • Peritricuspid macro re-entry More complex macro re-entry
Case 7
31 I
29
30
II V1
RFd
RFp
CS 1-2
CS 3-4
CS 5-6
CS 7-8 CS 9-10
Fig. 7.3 Recordings of the RF ablation catheter and a decapolar catheter inserted into the coronary sinus (CS 1–2 to 9–10). Site of complex fractionated electrograms with a gradient of activation between the proximal and distal bipoles (black arrows)
32
M. Haissaguerre
Fig. 7.4 NavX (EnSite System, St. Jude Medical, MN, USA) activation map (antero-posterior view) showing local activity spanning all the cycle lengths in a very localized region below the left atrial appendage
I
27
II V1
RFd 154 ms
CS 1-2 CS 3-4
Fig. 7.5 Recordings of the RF ablation catheter and a decapolar catheter inserted into the coronary sinus (CS 1–2 to 9–10). After restoration of sinus rhythm, local double potential visualized after ablation
CS 5-6 CS 7-8 CS 9-10
28
Case 7
Bibliography Haïssaguerre M, Shah DC, Jaïs P, et al. Electrophysiological breakthroughs from the left atrium to the pulmonary veins. Circulation. 2000;102:2463-2465. Haissaguerre M, Sanders P, Hocini M, et al. Catheter ablation of longlasting persistent atrial fibrillation: critical structures for termination. J Cardiovasc Electrophys. 2005;16(11):1125-1137.
33 Haïssaguerre M, Hocini M, Sanders P, et al. Localized sources maintaining atrial fibrillation organized by prior ablation. Circulation. 2006;113(5):616-625. Mohamed U, Skanes AC, Gula LJ, et al. A novel pacing maneuver to localize focal atrial tachycardia. J Cardiovasc Electrophys. 2007;18(1):1-6. Sanders P, Hocini M, Jais P, et al. Characterization of focal atrial tachycardia using high-density mapping. J Am Coll Cardiol. 2005 December 6;46(11):2088-2099.
Case 8 Michel Haissaguerre
Case Summary A 54-year-old female with persistent atrial fibrillation (AF) for 12 months, a 10-year history of AF, a dilated left atrium (LA) (54 mm diameter), and having failed five external cardioversions and amiodarone, was referred for ablation. Baseline AF cycle length (CL) before ablation was 135 ms in the left atrial appendage (LAA) and 132 ms in the right atrial appendage (RAA). The first procedure consisted of PV isolation, extensive ablation based on electrograms in the LA, and linear lesions at the roof and the left mitral isthmus. Then electrogram-based ablation in the RA resulted in a transient conversion to atrial tachycardia (AT). Early recurrence of AF
I II V1
A
I II V1
B
required external cardioversion to restore sinus rhythm. During sinus rhythm, both linear lesions were assessed and completed. Four months later, the woman developed an AT and she therefore had a second ablation procedure. A decapolar catheter was placed into the coronary sinus (CS) and demonstrated a colliding activation pattern in the proximal CS. A quadripolar cooled-tip ablation catheter was then inserted into the LA. AT cycle length was 271 ms with irregularity reaching 22% of the mean AT cycle length. Pulmonary vein isolation (PVI) was confirmed, and the endocardial activation was determined by moving the ablation catheter sequentially in the anterior LA (Fig. 8.1, panel A, low anterior; and
I II V1
C
I II V1
Abl
Abl
Abl
Abl
CS
CS
CS
CS
B
A
D
C
D
Fig. 8.1 Location of different recordings in the anterior and posterior left atrium, high and low
M. Haissaguerre Department of Cardiology and Electrophysiology, CHU de Bordeaux, Hospital Cardiologique de Haut Leveque, Avenue de Magellan, Pessac, 33604, France A. Natale et al. (eds.), Cardiac Electrophysiology, DOI: 10.1007/978-1-84996-390-9_8, © Springer-Verlag London Limited 2011
35
M. Haissaguerre
36
Fig. 8.1, panel B, high anterior) and in the posterior LA (Fig. 8.1, panel C, high posterior; and Fig. 8.1, panel D, low posterior). Activation was ascending in the anterior wall (Fig. 8.1, panels A and B) and descending at the posterior wall (Fig. 8.1, panels C and D) with the activation sequence covering all the AT cycle length. What inferences can be made from Fig. 8.1 with relevance to the mechanism of AT, and what maneuver has to be done to confirm the diagnosis?
Case Discussion First, for the purposes of describing the approach to ATs arising in the context of catheter ablation of AF, the following definitions are employed: macro re-entry is defined as a circuit involving more than three atrial segments, usually greater than 2 cm in diameter and where more than 75% of the circuit is mapped. Focal point tachycardia is defined as centrifugal activation originating from a discrete site and includes automaticity, triggered activity, and re-entrant mechanisms where 15%, suggestive of focal AT and if 15%
No
Focal
Focal or macroreentry
Map earliest region
Activation compatible with: • Perimitral macro re-entry • Roof dependant macro re-entry • Peritricuspid macro re-entry
• Localized re-entry • Focal point AT No
• ~100% cycle • Entrainment Yes Macro re-entry
Fig. 8.2 Practical approach to atrial tachycardia (AT) in the context of atrial fibrillation (AF) ablation. See text for details
2. Macro re-entry is first investigated by assessing the activation in the LA, in order to determine the likelihood of perimitral, roof-dependent, or peritricuspid circuit (entrainment maneuvers are used to confirm the diagnosis) 3. In the case of the absence of macro re-entrant AT, focal point/area AT is determined by tracking the earliest area of activity (entrainment maneuvers can be used to evaluate the site of origin). In the case of localized re-entry, long-duration fractionated potentials spanning most or all of the CL are tracked. Otherwise, the earliest activity is targeted 4. In the case of nonconsistency, more complex tachycardia are investigated, or a change in AT (mechanical or due to stimulation) is searched In our case, the activation front was consistent with a roofdependent macro-re-entry (from A to D): ascending the anterior wall (solid line, A to B) and descending the posterior wall (dashed line, C to D). The mapped activity spanned the entire CL (potentially compatible with roof dependent AT), but slow conduction in the anteroseptal area (A) could be misleading. Indeed, irregularity was >15% of the CL (inset ECG), suggesting in fact focal AT. Furthermore, PPI at the posterior wall was short (+20 ms), but the anterior PPI was very long (+200 ms), ruling out macro re-entrant roof-dependant AT. Because the roof line was previously blocked, it suggested a focal origin from the high posterior LA. Mapping this area close to the roof line revealed very low voltage local activity spanning most of the CL at three different spots very close to each other (Fig. 8.3, POST 1 to 3). This voltage was only 0.04 mV, which can be easily hidden in case of electrical noise. Ablation of a very low voltage continuous activity site located in between the three described spots terminated the tachycardia (Fig. 8.4). This case emphasizes the importance of localized re-entrant AT in the context of prior AF ablation.1 In this context, 53% of AT are focal and localized re-entries are the most frequent AT mechanism, representing 71% of focal ATs (and 37% of total ATs). The preferential regions for focal ATs are the PV–LA junction, left septum, and the mouth of the LAA. Focal AT are generally localized to sites targeted during electrogram-based AF ablation. Injury or edema to the atrial tissue induced by RF applications could generate the substrate for further arrhythmia by creating an anchoring point potentially able to maintain re-entry. Therefore the mechanism of AT after AF ablation may be different to that of spontaneous AT. The presence of preexisting LA scar (which may be idiopathic or related to underlying structural heart disease) may also result in local slow conduction areas predisposing to re-entry. Mapping and ablation of focal or macro re-entrant AT is a crucial step in the AF ablation process, and often represents the difference between procedural success and failure.
Case 8
37
Fig. 8.3 Antero-posterior fluoroscopic view of the left atrium (LA). Mapping of three close posterior spots (post 1–3). Local activity (within red rectangles) spanning the entire CL (pink bars)
I II V1
Post 1
Post 2
Post 3
CS
A
I II V1 0.05 mV Abl
CS
18
19
20
21
B
I II V1
CS 1-2 CS 3-4 CS 5-6 CS 7-8 CS 9-10
Fig. 8.4 Antero-posterior fluoroscopic view of the left atrium (LA) with a decapolar catheter placed into the coronary sinus (CS). Panel A: Ablation catheter (Abl) placed in the posterior LA and recording a very low voltage
( V. This excludes AVRT and makes AVNRT less likely unless there is block below the lower common pathway (see Figs. 10.4–10.8).
Fig. 10.1 12-lead resting ECG (paper speed 25 mm/s) showing normal sinus rhythm with a ventricular rate of 66 bpm and a normal PR interval (136 ms) and a QRS width of 100 ms
A. Rossillo (), S. Themistoclakis, A. Bonso, A. Corrado, and A. Raviele Ospedale dell’Angelo, Mestre, Venice, Italy A. Natale et al. (eds.), Cardiac Electrophysiology, DOI: 10.1007/978-1-84996-390-9_10, © Springer-Verlag London Limited 2011
41
42
A. Rossillo et al.
A
B
Fig. 10.2 Panel A: 12-lead ECG taken during atrial tachycardia (paper speed 25 mm/s). Panel B: a detail of lead D2, aVL, and V1
Case 10 Fig. 10.3 Intracardiac recordings taken during the electrophysiology study (paper speed 100 mm/s) revealing SVT with a cycle length of 300 ms. Four surface ECG leads (I, aVF, V1, V6), three bipolar recordings from the His bundle region (distal= HIS D, intermediate = HIS I, and proximal = HIS P), two bipolar recordings from the coronary sinus (CS prox = proximal coronary sinus and CS dist = distal coronary sinus), and the unipolar (MC U-CATH) and the distal bipolar recording of the mapping catheter (MC D)
Fig. 10.4 The electroanatomical mapping of the tachycardia showed the origin of the arrhythmia coming from the His bundle region (the red area in the LAO view). The orange tags show the area where it was possible to record the His bundle potentials
43
44
A. Rossillo et al.
Fig. 10.5 Electroanatomical mapping with CARTO: detailed remap of the area of interest; the orange tag shows the His bundle potential and the blue tag shows the site of earliest activation during tachycardia
Fig. 10.6 Electroanatomical mapping with CARTO: detailed remap of the area of interest; the orange tag reflects the His bundle potential, the blue tag shows the site of earliest activation during tachycardia, and the red tag shows where a single RF lesion is applied
Case 10 Fig. 10.7 Intracardiac recordings taken at the ablation site (paper speed 100 mm/s). Same display as that shown in Fig. 10.3. A atrium, V ventricle
Fig. 10.8 Intracardiac recordings taken at the ablation site (paper speed 100 mm/s) during RF application. A single RF application of 30 s with titration of the power starting from 30 W resulted in sinus rhythm restoration. RF energy was stopped as soon as the His bundle potential appeared on the ablation catheter. Same display as that shown in Fig. 10.3. A atrium, V ventricle, H His bundle
45
46
Bibliography Chen SA, Chiang CE, Yang CJ, Cheng CC, et al. Sustained atrialtachycardia in adult patients. Electrophysiological characteristics, pharmacological response, possible mechanisms, and effects of radiofrequency ablation. Circulation. 1994;90:1262-1278. Frey B, Kreiner G, Gwechenberger M, Gossinger H. Ablation of atrial tachycardia originating from the vicinity of the atrioventricular node: significance of mapping both sides of the interatrial septum. J Am Coll Cardiol. 2001;38:394-400. Kalman JM, Olgin JE, Karch MR, et al. “Cristal tachycardias”: origin of right atrial tachycardias from the crista terminalis identified by intracardiac echocardiography. J Am Coll Cardiol. 1998;31:451-459. Kay GN, Chong F, Epstein AE, Dailey SM, Plumb VJ. Radiofrequency ablation for treatment of primary atrial tachycardias. J AM Coll Cardiol. 1993;21:901-909. Knight BP, Zivin A, Souza J, et al. A technique for the rapid diagnosis of atrial tachycardia in the electrophysiology laboratory. J Am Coll Cardiol. 1999;33:775-781.
A. Rossillo et al. Lai LP, Lin GL, Chen TF, et al. Clinical electrophysiological characteristics and radiofrequency catheter ablation of atrial tachicardia near the apex of Koch’s triangle. PACE. 1998;21:367-374. Lesh MD, Van Hare GF, Epstein LM, et al. Radiofrequency Catheter ablation of atrial arrhythmias: results and mechanisms. Circulation. 1994;89:1074-1089. Pappone C, Stabile G, De Simone A, et al. Role of catheter-induced mechanical trauma in localisation of target sites of radiofrequency ablation of automatic atrial tachycardia. J Am Coll Cardiol. 1996;27:1090-1097. Poty H, Saudi N, Haissaguerre M, et al. Radiofrequency catheter ablation of atrial tachycardias. Am Heart J. 1996;131:481-489. Tang CW, Scheinmann MM, Van Hare GF, et al. Use of P wave configuration during atrial tachycardia to predict site of origin. J Am Coll Cardiol. 1995;26:1315-1324. Tracy CM, Swartz JF, Fletcher RD, et al. Radiofrequency catheter ablation of ectopic atrial tachycardia using paced activation sequence mapping. J Am Coll Cardiol. 1993;21:910-917.
Case 11 Yash Y. Lokhandwala, Anoop K. Gupta, and Ranjan K. Thakur
Case Summary
Case Discussion
A 17-year-old male with a normal heart has an electrocardiogram (ECG) showing intermittent pre-excitation suggestive of left free-wall accessory pathway (AP) and supraventricular tachycardia (SVT). The induced tachycardia is a narrow QRS regular tachycardia (Fig. 11.1) with ST depression in inferior leads. The patient is taken to the electrophysiological (EP) lab. In Fig. 11.2 the ablation catheter is positioned at the lateral tricuspid annulus (9 o’clock). Where is the site of the successful ablation?
The first two complexes show earliest A in CS 1-2 and in RFD; after this the coronary sinus (CS) activation reverses, but the A remains earliest in Radiofrequency Distal (RFD), suggesting that the orthodromic atrioventricular reentrant tachycardia (AVRT) is mediated by a right free-wall AP with a bystander left-lateral AP. Ablation of the right-sided AP resulted in success.
Y.Y. Lokhandwala () KEM Hospital, Parel, Mumbai, India A.K. Gupta Apollo Hospital, Ahmedabad, India R.K. Thakur Thoracic and Cardiovascular Institute Sparrow Health System, Michigan State University 405 West Greenlawn, Suite 400, Lansing, MI 48910 A. Natale et al. (eds.), Cardiac Electrophysiology, DOI: 10.1007/978-1-84996-390-9_11, © Springer-Verlag London Limited 2011
47
48
Y.Y. Lokhandwala et al. 2008021
I II III AVR AVL AVF V1 V2 V3 V4 V5 V6 10 mm/mV 25 mm/s
Fig. 11.1 The electrocardiogram (ECG) shows induced tachycardia
Case 11
I
49 2008021
AVF V1 V6 CS910 CS78 CS56 CS34 CS12 HISD HISP RFD RFP 10 mm/mV 200 mm/s
Fig. 11.2 Intracardiac recordings. CS 1-2 is distal. The ablation catheter (RFD) is placed along the lateral tricuspid annulus
Case 12 Eric Buch, Shiro Nakahara, Marmar Vaseghi, Noel G. Boyle, and Kalyanam Shivkumar
Case Summary A 59-year-old male with ischemic cardiomyopathy who underwent heart transplantation three years earlier was repeatedly hospitalized for atrial flutter (AFL) (Fig. 12.1, panel A) refractory to rate control and antiarrhythmic drugs. Left ventricular ejection fraction was normal by echocardiography. He was referred for electrophysiology study and ablation. Intracardiac recordings showed AFL with tachycardia cycle length 268 ms and earliest left atrial activation from the distal coronary sinus (CS), consistent with a left atrial circuit (Fig. 12.1, panel B). Entrainment pacing from the mid-CS A
performed during flutter demonstrated a long post-pacing interval of 442 ms. Noteworthy was that during entrainment pacing from the high right atrial (HRA) catheter, 2:1 block of right-to-left atrial conduction was observed without terminating the tachycardia (Fig. 12.1, panel C). Where is the likely circuit of this flutter?
Case Discussion Based on Fig. 12.1, which shows the left atrium dissociated from the flutter circuit during tachycardia, left AFL was excluded from the differential diagnosis. Electroanatomic
B
C
Fig. 12.1 Cardiac tracings recorded during the clinical atrial flutter (AFL). Panel A: Surface 12-lead ECG recorded during AFL. Panel B: Intracardiac tracings and ECG leads II, V2, and V5 showing earliest left atrial activation in the distal CS. Panel C: Intracardiac tracings and ECG leads II, V2, and V5 showing entrainment pacing from the HRA, and 2:1 block of the right-to-left atrial conduction. CS coronary sinus, HRA high right atrium, CS (HB) d,p the distal and proximal electrode pairs of the coronary sinus (His bundle) catheter
E. Buch (*), S. Nakahara, M. Vaseghi, N.G. Boyle, and K. Shivkumar UCLA Cardiac Arrhythmia Center, David Geffen School of Medicine at UCLA, 10833 Le Conte Ave, A2-237 CHS, Los Angeles, CA 90095-1679 e-mail:
[email protected] A. Natale et al. (eds.), Cardiac Electrophysiology, DOI: 10.1007/978-1-84996-390-9_12, © Springer-Verlag London Limited 2011
51
52
E. Buch et al.
mapping of the right atrium (RA) showed large areas of lowvoltage scar (colored gray in Fig. 12.2). Activation mapping revealed a macro-reentrant AT utilizing a channel in the lateral wall of the RA (Fig. 12.2). A line of ablation along the lateral RA wall, shown in Fig. 12.2, successfully terminated the tachycardia. It is known that conduction from RA to LA
occurs over multiple septal pathways, including Bachmann’s bundle, the foramen ovale, and CS.1 In this case, due to scarring over a large part of the RA, conduction through Bachmann’s bundle was more rapid than the lower septal sites, resulting in distal to proximal of LA activation as seen in the CS catheter.
200 ms
Fig. 12.2 Electroanatomic map of the right atrium during the clinical AFL. Red areas indicate earliest endocardial activation; orange, yellow, green, blue, and purple indicate progressively delayed activation. The scar is shown in gray ( S in V6. Hence, this is SVT, not VT. The diagnosis of SVT confirmed, note that the HV interval is normal. The first two complexes show A-waves in the coronary sinus (CS) simultaneous with the QRS, suggesting that this is most likely slow/fast atrioventricular nodal reentrant tachycardia (AVNRT). Then, two premature atrial complexes (PACs) are delivered from CS 5-6. The first PAC does not reset the tachycardia. The second PAC conducts with a longer AH interval, and subsequently the tachycardia has a longer CL, but the retrograde atrial activation remains the same, suggesting the presence of a different slow pathway. There is baseline RBBB. The patient has slow–fast AVNRT with two slow pathways, participating in AVNRT; hence, she has two CLs.
Y.Y. Lokhandwala (*) KEM Hospital, Parel, Mumbai, India A.K. Gupta Apollo Hospital, Ahmedabad, India R.K. Thakur Thoracic and Cardiovascular Institute, Sparrow Health System, Michigan State University, 405 West Greenlawn, Suite 400, Lansing, MI 48910
A. Natale et al. (eds.), Cardiac Electrophysiology, DOI: 10.1007/978-1-84996-390-9_19, © Springer-Verlag London Limited 2011
81
82
Y.Y. Lokhandwala et al. 2008070
I II III AVR AVL AVF V1 V2 V3 V4 V5 V6 10 mm/mV 25 mm/s
Fig. 19.1 Electrocardiogram (ECG) of wide complex tachycardia
2008070 I AVF V1 V6 CS910 CS78 CS56 CS34 CS12 HISD HISP
Fig. 19.2 Intracardiac recordings. CS 1-2 is distal
Case 20 Marmar Vaseghi, Noel G. Boyle, and Kalyanam Shivkumar
Case Summary A 21-year-old female with a history of viral cardiomyopathy status post orthotopic heart transplantation (OHT) in June 2002 presented with recurrent palpitations three years post transplant. She had received a right atrio–atrial anastomosis at the time of transplantation. She was found to have an atrial flutter (AFL) on surface electrocardiogram (ECG) that proved refractory to medications. Her cardiac catheterization and biopsy showed no evidence of transplant coronary artery vasculopathy or acute rejection, respectively. She was referred for ablation of her AFL.Fluoroscopic position of the duodecapolar catheter in the right anterior oblique (RAO) view and the corresponding ECG recordings are shown in
A
Fig. 20.1 Panel A: Fluoroscopic RAO view shows the position of the duodecapolar catheter. Panel B: Intracardiac activation sequence. CS coronary sinus, D1–D10 duodecapolar bipoles 1–10, DDC duodecapolar catheter, RV right ventricle, LV left ventricle
Fig. 20.1. Duodecapolar bipoles 6–9 were positioned more posteriorly at the native atrium. What type of AFL is demonstrated by the intracardiac duodecapolar ECGs? Is there activation of the native atrium? What is the rate of the native atrial rhythm?
Case Discussion The duodecapolar recordings from the donor atrium showed a typical tricuspid annulus dependent counterclockwise AFL; Fig. 20.2 (top panels). This is the most common type of supraventricular tachycardia (SVT) in stable OHT patients.1
B I AVF V1 V6 D10 D9 D8 D7 D6 D5 D4 D3 D2 D1
200ms
M. Vaseghi (*), N.G. Boyle, and K. Shivkumar UCLA Cardiac Arrhythmia Center, David Geffen School of Medicine at UCLA, 10833 Le Conte Ave, A2-237 CHS, Los Angeles, CA 90095-1679 e-mail:
[email protected] A. Natale et al. (eds.), Cardiac Electrophysiology, DOI: 10.1007/978-1-84996-390-9_20, © Springer-Verlag London Limited 2011
83
84 Fig. 20.2 Panel A: Fluoroscopic RAO view shows the position of the duodecapolar catheter with the distal bipoles (D1–D3) along the tricuspid valve isthmus and the proximal electrode (D10) along the superior aspect of the interatrial septum. Panel B: Intracardiac activation sequence in the donor atrium confirms counterclockwise activation. CS coronary sinus, D1–D10 duodecapolar bipoles 1–10, DDC duodecapolar catheter, RV right ventricle, LV left ventricle
M. Vaseghi et al.
A
B I AVF V1 V6 D10 D9 D8 D7 D6 D5 D4 D3 D2 D1
200ms
Fig. 20.3 Electroanatomic maps of atrio-atrial anastamosis (gray areas identify native atrial tissue). The flutter wavefront involves the donor atrium only and is propagating in a counterclockwise fashion along the tricuspid annulus
Poles D6–D9 are in contact with the native atrial tissue, which is not in flutter, and is in a dissociated regular bradycardic rhythm of 40 bpm (beats per minute) (arrow), separate from the tachycardia. A critical isthmus is noted on the intracardiac ECGs at duodecopolar position D5. The activation map is shown in Fig. 20.3. An inferior vena cava (IVC) – tricuspid isthmus ablation line terminated the donor atrial tachycardia. No ablation was performed on the native atrium.
Reference 1. Vaseghi M, Boyle NG, Kedia R, et al. Supraventricular tachycardia following orthotopic heart transplantation. J Am Coll Cardiol. 2008;51:2241-2249.
Case 21 Bradley P. Knight
Case Summary A young man underwent an electrophysiology (EP) procedure for recurrent supraventricular tachycardia (SVT). The recording obtained after induction of the tachycardia is shown in Fig. 21.1. Surface intracardiac recordings from the His bundle electrogram (HBE), mapping catheter (Map), coronary sinus (CS), and right ventricle (RV) are shown. What is the mechanism of the tachycardia?
Case Discussion The initial part of the tracing shows a wide complex tachycardia with a cycle length (CL) of 320 ms. There is a His-bundle recording before the QRS complex, which has a typical left bundle branch block (LBBB) pattern. Therefore, the tachycardia is consistent with a supraventricular mechanism. The
earliest atrial activation can be seen in the proximal CS at electrode pair CS 3-4. The QRS complex, however, normalizes during the second part of the recording. An important measurement to make – whenever a SVT is associated with both a normal QRS complex and a bundle branch block – is the VA interval. The reason for this is that when the tachycardia mechanism is orthodromic reentrant tachycardia (ORT), and a rate-related bundle branch block develops on the same side as the accessory pathway (AP), the reentrant circuit will increase in length (due to transeptal conduction) and results in a longer VA interval during the ipsilateral bundle branch block. In this case, the VA interval is clearly longer during the LBBB compared to when the QRS complex normalizes. This is diagnostic for ORT using a left-sided AP. This case is also interesting, because the tachycardia CL does not change significantly during the LBBB, despite an increase in the VA interval. This is because there is a compensatory decrease in the AH interval during the LBBB.
B.P. Knight Division of Cardiology, Northwestern Medical Center, 676 N. St. Clair, Suite 600, Chicago, IL 60611 A. Natale et al. (eds.), Cardiac Electrophysiology, DOI: 10.1007/978-1-84996-390-9_21, © Springer-Verlag London Limited 2011
85
86
B.P. Knight I II III V1 V5 HBE d HBE p Map d Map p CS 1-2 CS 3-4 CS 5-6 CS 7-8 CS 9-10 RV d Stim
Fig. 21.1 This tracing was obtained during an ablation procedure for supraventricular tachycardia. Note the change in the ventricular atrial interval when the wide-QRS complex normalizes. The vertical line
depicts the onset of the surface QRS complex in each case and the arrow points to the earliest intracardiac atrial activation
Case 22 Antonio Rossillo, Sakis Themistoclakis, Aldo Bonso, Andrea Corrado, and Antonio Raviele
Case Summary
Case Discussion
The patient was a 34-year-old female with a history of palpitations. No heart disease was documented. Two months prior to the ablation the arrhythmia episodes became more frequent. After examining Figs. 22.1–22.4, will ablation at a single site eliminate the patient’s palpitations?
In this case, catheter manipulation induced AVNRT. Ventricular pacing demonstrated retrograde conduction via a left-sided accessory pathway. AVRT was also easily induced and was consistent with the clinical tachycardia. Ablation of the left-sided AP and the slow pathway were performed (see Figs. 22.5–22.11).
I II III aVR aVL aVF V1 V2 V3 V4 V5 V6
Fig. 22.1 Twelve-lead resting ECG (paper speed 25 mm/s) showing short PQ interval with ventricular pre-exitation with an early transition of R wave between lead V1 and V2 and negative wave in leads III and aVF
A. Rossillo (), S. Themistoclakis, A. Bonso, A. Corrado, and A. Raviele Ospedale dell’Angelo, Mestre, Venice, Italy A. Natale et al. (eds.), Cardiac Electrophysiology, DOI: 10.1007/978-1-84996-390-9_22, © Springer-Verlag London Limited 2011
87
88
Fig. 22.2 Intracardiac recordings at baseline during the electrophysiology study (paper speed 100 mm/s). Four surface ECG leads (I, aVF, V1, V6), two bipolar recordings from the pacing catheter placed in right
A. Rossillo et al.
atrium or ventricle (HRA), three bipolar recordings from the His bundle region (HIS PROX, MED, and DIST), and five bipolar recordings from the coronary sinus (CS). A Atrium, V Ventricle, H His bundle
Fig. 22.3 Intracardiac recordings taken during SVT induced with catheter manipulation (paper speed 100 mm/s, cycle length 330 ms). Same display as that shown in Fig. 22.2. A Atrium, V Ventricle, H His bundle
Case 22
89
Fig. 22.4 Intracardiac recordings taken during programmed ventricular stimulation with induction of SVT (paper speed 100 mm/s, cycle length 290 ms) with earliest activation on CS 3. Same display as that shown in Fig. 22.2. A Atrium, V Ventricle
Fig. 22.5 Intracardiac recordings taken during AVRT (paper speed 100 mm/s, cycle length 264 ms). Same display as that shown in Fig. 22.2 and a bipolar recording from the distal mapping catheter (ABL dist).
A spontaneous ventricular ectopic beat reduced the cycle length of the tachycardia and the anterograde conduction after the ectopic beat is through nodal decremental fibers
90
A. Rossillo et al.
Fig. 22.6 Intracardiac recordings taken during sinus rhythm (paper speed 100 mm/s). Same display as that shown in Fig. 22.5. Ablation site
III aVF V1 V6 HRA 1-2 HRA 3-4 HIS PROX HIS MED HIS DIST CS 1 CS 2 CS 3 CS 4 CS 5 ABL dist
Fig. 22.7 Intracardiac recordings taken during RF energy delivery (paper speed 100 mm/s). Same display as that shown in Fig. 22.5. The red arrows show the absence of the ventricular pre-excitation
Case 22
91
Fig. 22.8 Intracardiac recordings taken during RF energy delivery (paper speed 25 mm/s). Same display as that shown in Fig. 22.5. The red arrows show the absence of ventricular pre-excitation
III aVF V1 V6 HRA 1-2 HRA 3-4 HIS PROX HIS MED HIS DIST CS 1 CS 2 CS 3 CS 4 CS 5 ABL dist V
A
V
A
Fig. 22.9 Intracardiac recordings (paper speed 100 mm/s) taken during programmed ventricular stimulation showing a decremental VA conduction (paper speed 100 mm/s). Same display as that shown in Fig. 22.5. A Atrium, V Ventricle
92
A. Rossillo et al.
Fig. 22.10 Intracardiac recordings (paper speed 100 mm/s) taken during programmed atrial stimulation with two extrastimuli showing the ERP of the AV node (paper speed 100 mm/s). Same display as that shown in Fig. 22.5. A Atrium, V Ventricle
I II III aVR aVL aVF V1 V2 V3 V4 V5 V6
Fig. 22.11 12-lead resting ECG (paper speed 25 mm/s) showing normal PQ interval with the absence of ventricular pre-excitation with negative T waves in the inferior leads
Case 22
Bibliography Calkins H, Langberg J, Sousa J, et al. Radiofrequency catheter ablation of accessory atrioventricular connections in 250 patients. Abbreviated therapeutic approach to Wolff-Parkinson-White syndrome. Circulation. 1992;85:1337-1346. Haïssaguerre M, Dartigues JF, Warin JF, Le Metayer P, Montserrat P, Salamon R. Electrogram patterns predictive of successful catheter
93 ablation of accessory pathways. Value of unipolar recording mode. Circulation. 1991;84:188-202. Jackman WM, Wang XZ, Friday KJ, et al. Catheter ablation of accessory atrioventricular pathways (Wolff-Parkinson-White syndrome) by radiofrequency current. N Engl J Med. 1991;324:1605-1611. Scheinman MM, Wang YS, Van Hare GF, Lesh MD. Electrocardio graphic and electrophysiologic characteristics of anterior, midseptal and right anterior free wall accessory pathways. J Am Coll Cardiol. 1992;20:1220-1229.
Case 23 Yash Y. Lokhandwala, Anoop K. Gupta, and Ranjan K. Thakur
Case Summary
Case Discussion
The patient is a 58-year-old male with a normal heart. The resting electrocardiogram (ECG) is normal. He has a narrow QRS tachycardia. He is taken to the electrophysiology (EP) lab; see Figs. 23.1 and 23.2. What is the likely mechanism of this tachycardia?
The ventriculo-atrial (VA) conduction pattern and VA time are different in the two beats in Fig. 23.1. The first retrograde A is bracketed, earliest in coronary sinus (CS) 3-4 and 5-6; the second retrograde A is also bracketed, but earliest in CS 5-6 and 7-8. Figure 23.2 shows para-Hisian pacing. The VA
Fig. 23.1 Ventricular pacing
Y.Y. Lokhandwala (*) KEM Hospital, Parel, Mumbai, India A.K. Gupta Apollo Hospital, Ahmedabad, India R.K. Thakur Thoracic and Cardiovascular Institute, Sparrow Health System, Michigan State University, 405 West Greenlawn, Suite 400, Lansing, MI 48910 A. Natale et al. (eds.), Cardiac Electrophysiology, DOI: 10.1007/978-1-84996-390-9_23, © Springer-Verlag London Limited 2011
95
96
Y.Y. Lokhandwala et al.
Fig. 23.2 Para-Hisian pacing
time clearly is shortest with His bundle capture (second complex). The VA pattern of the first complex is different from the other two complexes. The first retrograde A is earliest in CS 5-6. The second and third retrograde A are earliest in CS 7-8. Thus the second and third retrograde A are via the AV node; the first is likely via a left AP. The tachycardia displays varying degrees of right bundle branch block (RBBB) during SVT (Fig. 23.3) (normal HV). The earliest retrograde A is consistently in CS 5-6, with a constant VA interval. CS 9-10 was at CS os (ostium). Thus the retrograde A is via a concealed left posterior AP.
In Fig. 23.4, tachycardia has a left bundle branch block (LBBB) morphology. Compared to the previous tracing, the VA time is clearly longer in this case. This is consistent with our prior diagnosis of a left-sided AP. Ablation was performed during right ventricular (RV) pacing (Fig. 23.5) because this allows for catheter stability. If ablation is performed during reentrant tachycardia, the catheter may move suddenly when RT terminates during RF application. So, catheter stability can be obtained by entraining the RT at 10–20 ms faster than the tachycardia CL. The VA conduction during RV pacing changes from left AP to AV node.
Case 23
Fig. 23.3 Development of a right bundle branch block (RBBB)
97
98
Fig. 23.4 Development of a left bundle branch block (LBBB)
Y.Y. Lokhandwala et al.
Case 23
Fig. 23.5 Ablation of AP
99
Case 24 Marmar Vaseghi, Noel G. Boyle, and Kalyanam Shivkumar
Case Summary
graphy demonstrated a normal ejection fraction at the time of arrhythmia. Surface electrocardiogram (ECG) was consistent with atypical atrial flutter (AFL). The patient was taken to the electrophysiology (EP) laboratory. Intracardiac ECGs along with location of the catheters are shown in (Fig. 24.1). What is the likely mechanism of this arrhythmia based on the duodecapolar and coronary sinus (CS) catheter recordings?
A 60-year-old male status post orthotopic heart transplantation (OHT) in June 1999 presented with drug refractory palpitations 8 years after his transplantation. Cardiac catheterization and biopsy showed no evidence of transplant vasculopathy or acute rejection, respectively. Echocardio
A
B
I aVF V6
200 msec
D1 D2 D3 D4 D5 D6 D7 D8 D9 D10 CS 9,10 CS 7,8 CS 5,6 CS 3,4 CS 1,2 RVa
Fig. 24.1 Panel A: Fluoroscopic LAO view: The position of the duodecapolar and CS catheters are shown in right atrium. Panel B: Intracardiac tracings obtained from the duodecapolar and CS catheters are shown.
CS coronary sinus, D1–D10 duodcapolar bipoles 1–10, LAO left anterior oblique, RVa RV catheter bipole
M. Vaseghi (*), N.G. Boyle, and K. Shivkumar UCLA Cardiac Arrhythmia Center, David Geffen School of Medicine at UCLA, 10833 Le Conte Ave, A2-237 CHS, Los Angeles, CA 90095-1679 e-mail:
[email protected] A. Natale et al. (eds.), Cardiac Electrophysiology, DOI: 10.1007/978-1-84996-390-9_24, © Springer-Verlag London Limited 2011
101
102
Case Discussion Although typical clockwise AFL is the most common type of supraventricular tachycardia (SVT) in stable post-OHT patients, this patient’s intracardiac tracings show earliest activation in the distal CS consistent with a left AFL. Further, the flutter pattern spreads from the distal CS to the proximal CS and then to the right atrium where D10 (or septal activation is earliest). This is followed by activation
Fig. 24.2 Activation map: the flutter circuit begins in the inferior wall of the LA (pink) close to the CS 3-4 bipole, and then propagates across to the RA and around the tricuspid annulus, as well as around and over the mitral annulus. LA left atrium, LAA left atrial appendage, LIPV left inferior pulmonary vein, LSPV left superior pulmonary vein, RA right atrium, RSPV right superior pulmonary vein, SVC superior vena cava
M. Vaseghi et al.
around the tricuspid annulus. Hence this left AFL spreads in a figure-eight fashion around both the tricuspid and mitral annuli. Mapping with ablation catheters showed earliest activation at CS 1-2 and ostium of the left inferior pulmonary vein. The activation map obtained via the ablation catheter and the ablation line, which successfully terminated the tachycardia, are shown in (Fig. 24.2). The patient has been without recurrence during the 10 months of follow-up.
Case 25 Bradley P. Knight
Case Summary A female marathon runner without structural heart disease had a documented regular wide complex tachycardia (Fig. 25.1). At the time of electrophysiology (EP) testing, she had normal sinus and atrioventricular (AV) node function, no ventricular preexcitation, and no inducible ventricular tachycardia (VT). A wide complex tachycardia was induced with an atrial premature extrastimulus during sinus rhythm (Fig. 25.2). The QRS morphology was a right bundle branch block (RBBB) pattern with an inferior axis. There was not His-bundle potential recordable before the QRS during the tachycardia. What is the most likely diagnosis?
tachycardia was induced by an atrial premature beat that conducts to the ventricle, with a QRS morphology that is the same as that during tachycardia, makes VT very unlikely. SVT with aberrancy can also be excluded because a Hisbundle potential could not be recorded before the QRS complex. Therefore, the most likely diagnosis is ART. In a patient with no preexcitation during sinus rhythm, the most common mechanism of ART is a slowly conducting, right-sided atriofascicular accessory pathway (AP). A study of 384 patients with a single AP found that anterograde decremental conduction was seen only in the right free wall location. However, the QRS morphology in this patient is not consistent with a right-sided AP. In this case, the earliest ventricular activation was along the mitral annulus where ablation was successful (Fig. 25.3). This represents a very unusual location for a slowly conducting anterograde AP.
Case Discussion The differential of a wide complex tachycardia includes VT, supraventricular tachycardia (SVT) with aberrancy, and antidromic AV reentrant tachycardia (ART). The fact that this
B.P. Knight Division of Cardiology, Northwestern Medical Center, 676 N. St. Clair, Suite 600, Chicago, IL 60611 A. Natale et al. (eds.), Cardiac Electrophysiology, DOI: 10.1007/978-1-84996-390-9_25, © Springer-Verlag London Limited 2011
103
104
B.P. Knight
Fig. 25.1 Shown is a two lead rhythm strip showing the onset of a sustained wide-QRS complex tachycardia. The tracing was recorded from an ambulatory event recorder
Case 25
Fig. 25.2 This tracing was recorded during an ablation procedure in a patient with a wide complex tachycardia. Shown are surface electrograms from leads I, II, V1, and V3, and the intracardiac electrograms from the high right atrium (HRA), the ablation catheter (ABL) posi-
105
tioned at the His bundle location, and the right ventricular apex (RVA). Note the initiation of a wide complex tachycardia with a single atrial extrastimulus delivered during sinus rhythm
106
B.P. Knight
Fig. 25.3 This tracing was recorded after the ablation catheter was positioned along the mitral annulus at the site of the earliest ventricular local activation. The format is the same as figure 25.2
Case 26 Richard H. Hongo and Andrea Natale
Case Summary The patient is a 53-year-old woman with recurrent palpitations since age 16 with documented episodes of tachyarrhythmia (Fig. 26.1). Discrete P-waves at a rate of approximately 180 bpm have been apparent with atrioventricular (AV) node blockade from adenosine (Fig. 26.2).
She has been treated with a variety of antiarrhythmic agents including verapamil, sotalol, and amiodarone with variable success. She presented for electrophysiology (EP) study with possible catheter ablation because of highly symptomatic episodes of atrial arrhythmia that continued to recur despite medical therapy. High-dose isoproterenol infusion (20 mcg/min)
Fig. 26.1 Continuous 3-lead (II, V1, V5) ECG capturing recurrent episodes of narrow-complex regular tachyarrhythmia at a rate of approximately 180 bpm
R.H. Hongo Sutter Pacific Medical Foundation, California Pacific Medical Center, 2100 Webster Street, Suite 521, San Francisco, CA 94115, USA e-mail:
[email protected] A. Natale () Texas Cardiac Arrhythmia Institute, St. David’s Medical Center, 1015 East 32nd Street, Suite 516, Austin, TX 78705 e-mail:
[email protected] A. Natale et al. (eds.), Cardiac Electrophysiology, DOI: 10.1007/978-1-84996-390-9_26, © Springer-Verlag London Limited 2011
107
108
during EP study induced a tachyarrhythmia with a ventricular rate of 150 bpm (Fig. 26.3). Double transseptal puncture was performed and a 20-mm 10-electrode lasso catheter and a 3.5-mm F-curve saline irrigated ablation catheter were advanced into the left atrium (LA). Intracardiac activation of the atrial arrhythmia was recor ded (Fig. 26.4) with the lasso catheter at the os of the right superior pulmonary vein (RSPV) and the ablation catheter approximately 6 mm within the same vein (Fig. 26.5). A 20-electrode deflectable catheter was introduced from the right internal jugular vein and placed along the lateral wall of the right atrium (RA) with the distal 10 electrodes in the coronary sinus (CS). What is the tachyarrhythmia based on the surface electrocardiogram (ECG) before and during adenosine, and assuming it is the same arrhythmia induced with isoproterenol? What is the tachyarrhythmia based on the intracardiac ECGs? What is the most appropriate next step?
R.H. Hongo and A. Natale
Case Discussion AV block during adenosine infusion (Fig. 26.2) uncovers discrete P-waves most consistent with atrial tachycardia. Distinct isoelectric segments between the P-waves in all twelve leads of the ECG during isoproterenol infusion (Fig. 26.3) makes both typical and atypical flutter less likely. Loss of 1:1 AV association eliminates atrioventricular reciprocating tachycardia (ART) as a diagnosis. Intracardiac ECGs (Fig. 26.4) reveal the earliest activation to be at the distal ablation catheter positioned within the RSPV (Fig. 26.5), clearly before the activation around the lasso catheter at the vein os. The most appropriate ablation strategy is to isolate the RSPV. Ablation at the earliest activation site within the vein should be avoided because of the risk of PV stenosis. If either atrial fibrillation (AF) or typical atrial flutter (AFL) were observed clinically, isolation of all four pulmonary veins and cavotricuspid isthmus ablation, respectively, would also be appropriate.
Fig. 26.2 Continuous 3-lead (II, V1, V5) ECG during adenosine-induced AV block. Uncovered discrete P-waves are most consistent with atrial tachycardia
Case 26
109
Fig. 26.3 12-lead ECG during isoproterenol infusion (20 mcg/min) demonstrates sustained atrial tachyarrhythmia. Distinct isoelectric PR segments are seen in all leads I
200 ms
aVF V1
S
V6 HRA 19,20 HRA 17,18 HRA 15,16 HRA 13,14 HRA 11,12 ABL p ABL d CS 9,10 CS 7,8
C
CS 5,6
C
CS 3,4
C
CS d
C
Ls 1,2 Ls 2,3 Ls 3,4 Ls 4,5 Ls 5,6 Ls 6,7 Ls 7,8 Ls 8,9 Ls 9,10
10:06:09 AM
10:06:10 AM
CardioLab v6.5 GE Medical Systems
Fig. 26.4 Simultaneous surface and intracardiac ECGs during the atrial tachyarrhythmia reveal the earliest activation to be at the distal electrode pair of the ablation catheter (ABL d), clearly in front of the activation seen on the Lasso catheter. HRA = high right atrial catheter; CS = coronary sinus catheter; Ls = Lasso catheter
110
Fig. 26.5 Fluoroscopic image in shallow LAO projection, showing the position of the catheters that correspond with the intracardiac ECGs from Fig. 26.4. The Lasso catheter is placed at the right superior pulmonary vein (RSPV) os. The ablation catheter is shown positioned within the RSPV
R.H. Hongo and A. Natale
Case 27 M. Eyman Mortada, Jasbir S. Sra, and Masood Akhtar
Case Summary A 63-year-old female with a history of tachycardia 7 years ago with the electrocardiogram (ECG) shown in Fig. 27.1 presented complaining of recurrent palpitations with the tachycardia shown in Fig. 27.2. During an electrophysiology (EP) study, the responses from premature ventricular beat and ventricular pacing were recorded during the tachycardia, as shown in Figs. 27.3 and 27.4. What is the diagnosis?
Case Discussion The presenting tachycardia is characterized by a long RP, narrow complex tachycardia, with low septal to high atrial activity (negative P-wave in lead II and positive P-waves in leads aVR and aVL). Therefore, the differential diagnoses include: low septal atrial tachycardia, atypical AV nodal reentry tachycardia (AVNRT), and persistent junctional reciprocating tachycardia.
Seven years prior to this presentation, the patient had short RP and narrow complex tachycardia. The differential diagnoses of that rhythm include: typical AVNRT, atrial tachycardia (AT) with long PR interval, and AV reentry tachycardia via an accessory pathway (AP). The presence of both rhythms in one patient means this is probably AVNRT. The first rhythm was probably a typical tachycardia. The patient now presents with atypical AVNRT. The first intracardiac tracing reveals earliest atrial activation is in the proximal coronary sinus (CS), suggesting an AT originating in the lower septum, retrograde activation over the AV node, or retrograde activation over a septal AP. A spontaneous premature ventricular capture (PVC) during the tachycardia, at the time of the His refractory, was observed. At that time, the tachycardia cycle length (CL) was not changed. The presence of a PVC reduces the likelihood that the mechanism is an AV reentry tachycardia via a septal AP. The second intracardiac tracing illustrates ventricular entrainment during the tachycardia. Post–ventricular pacing shows a V-A-V response, excluding the diagnosis of a low septal AT. Thus, the diagnosis is atypical AVNRT.
M.E. Mortada (*) J.S. Sra, and M. Akhtar Department of Electrophysiology, Aurora Cardiovascular Services, Aurora Sinai/Aurora St. Luke’s Medical Centers, University of Wisconsin School of Medicine and Public Health, 2801 W. Kinnickinnic River Parkway, #777, Milwaukee, WI 53215 A. Natale et al. (eds.), Cardiac Electrophysiology, DOI: 10.1007/978-1-84996-390-9_27, © Springer-Verlag London Limited 2011
111
112
Fig. 27.1 Surface ECG of the tachycardia seven years prior to the current presentation
Fig. 27.2 Surface ECG of the current tachycardia
M.E. Mortada et al.
Case 27
113
Fig. 27.3 Spontaneous PVC during the tachycardia at the His refractory period. From top to bottom: surface ECG with leads I, II and V1, followed by intracardiac tracing of HRA, proximal to distal CS, and finally His catheters
Fig. 27.4 Response from ventricular entrainment of the tachycardia
Case 28 Yash Y. Lokhandwala, Anoop K. Gupta, and Ranjan K. Thakur
Case Summary A 21-year-old female with a normal heart comes to the electrophysiology (EP) lab because of palpitations. A diagnosis of typical atrioventricular reentrant tachycardia (AVNRT) is made (Fig. 28.1). However, after atrial pacing, long PR and sudden left bundle branch block (LBBB) – like morphology is seen (Figs. 28.2 and 28.3). What maneuver is being performed? What does this say about the tachycardia mechanism?
during tachycardia; this caused advancement of the next QRS. The QRS advancement by a “His-refractory” APD proves the presence of an extranodal (atriofascicular or atrioventricular accessory pathway) (Fig. 28.4). The HA during tachycardia is identical to HA during RV pacing; thus the Mahaim-like accessory pathway is not a bystander. Final diagnosis: (1) Right lateral Mahaim-like AP with true antidromic AVRT and (2) slow–fast, typical AVNRT (Figs. 28.5 and 28.6).
Case Discussion An atrial premature depolarization (APD) was delivered from a radiofrequency (RF) catheter in the right atrium (RA),
Y.Y. Lokhandwala () KEM Hospital, Parel, Mumbai, India A.K. Gupta Apollo Hospital, Ahmedabad, India R.K. Thakur Thoracic and Cardiovascular Institute, Sparrow Health System, Michigan State University, 405 West Greenlawn, Suite 400, Lansing, MI 48910
A. Natale et al. (eds.), Cardiac Electrophysiology, DOI: 10.1007/978-1-84996-390-9_28, © Springer-Verlag London Limited 2011
115
116
Y.Y. Lokhandwala et al.
RUBY HALL CLINIC I
2008051
AVF V1 V6
CS910
CS78
CS56
CS34
CS12
HISP
RFD
RFP 10 mm/mV 100 mm/s
Fig. 28.1 Typical atrioventricular reentrant tachycardia (AVNRT)
Version WIN2000/XP : EPTRACER V0.77
Case 28 RUBY HALL CLINIC 2008051 I II III AVR AVL AVF V1 V2 V3 V4 V5 V6 10 mm/mV 25 mm/s
Fig. 28.2 Change to left bundle branch block (LBBB) morphology is seen with atrial pacing
117 Version WIN2000/XP : EPTRACER V0.77
118
Y.Y. Lokhandwala et al. Version WIN2000/XP : EPTRACER V0.77
I
2008051
AVF V1 V6
CS910
CS78
CS56
CS34
CS12
HISP
RFD
10 mm/mV 100 mm/s
Fig. 28.3 A diagnostic maneuver. Does this help with a diagnosis?
Case 28
119 Version WIN2000/XP : EPTRACER V0.77
I
2008051
AVF V1 V6
CS910
CS78
CS56
CS34
CS12
HISP
RFD
10 mm/mV 200 mm/s
Fig. 28.4 APC delivered from the RA free wall
120
Y.Y. Lokhandwala et al.
RUBY HALL CLINIC 2008051 I AVF V1 V6 CS78 CS56 CS34 CS12 HISP HISM HISD RBP RBD RFD 10 mm/mV 200 mm/s
Fig. 28.5 Intracardiac recording during SVT
Version WIN2000/XP : EPTRACER V0.77
Case 28
121 Version WIN2000/XP : EPTRACER V0.77
2008051 I AVF V1 V6 CS78 CS56 CS34 CS12 HISP HISM HISD RBP RBD RFD 10 mm/mV 200 mm/s
Fig. 28.6 RV pacing
Case 291 Eric Buch, Shiro Nakahara, Marmar Vaseghi, Noel G. Boyle, and Kalyanam Shivkumar
Case Summary
Case Discussion
A 57-year-old female with recurrent atrial flutter (AFL) was referred for catheter ablation. The surface electrocardiogram (ECG) suggested clockwise AFL with positive F-waves in II, III, and AVF, and negative F-wave in V1. The complete absence of an R-wave in lead I was noted. The chest x-ray demonstrated dextrocardia, and echocardiography established complete situs inversus with otherwise normal intracardiac anatomy. Based on the ECG, what is the likely diagnosis?
A duodecapolar (DD) catheter was positioned around the tricuspid annulus in the anatomical right atrium (RA) with the distal tip located at the medial cavotricuspid isthmus (Fig. 29.1). Electrophysiological mapping of the AFL demonstrated earliest activation in the electrogram recorded from DD poles 19,20 and latest activation at DD poles 1,2. Top left image shows an LAO view of the heart (camera in the RAO position relative to the torso) and direction of flutter (white arrows) as shown by
Fig. 29.1 Fluoroscopy of intracardiac catheters. Top left panel shows LAO view of the heart (camera in the RAO position relative to the torso). Top right panel shows the RAO view of the heart (camera in the
LAO position relative to the torso). CS coronary sinus catheter, Abl ablation catheter, HRA high right atrial catheter, His His bundle catheter
This case is adapted from heart rhythm volume 2, issue 6, pages 673-674 (June 2005)
1
E. Buch (), S. Nakahara, M. Vaseghi, N.G. Boyle, and K. Shivkumar UCLA Cardiac Arrhythmia Center,David Geffen School of Medicine at UCLA, 10833 Le Conte Ave, A2-237 CHS, Los Angeles, CA 90095-1679 e-mail:
[email protected] A. Natale et al. (eds.), Cardiac Electrophysiology, DOI: 10.1007/978-1-84996-390-9_29, © Springer-Verlag London Limited 2011
123
124
E. Buch et al.
the intracardiac tracings, demonstrating early activation in DD19, 20 and counterclockwise rotation of flutter in the RA (Fig. 29.2). Entrainment pacing demonstrated that the
tricuspid valve – inferior vena cava (IVC) isthmus was a part of the circuit and this tachycardia was cured by ablation of the isthmus.
I II III aVF V1 HRA DD19,20 DD17,18 DD15,16 DD13,14 DD11,12 DD9,10 DD7,8 DD5,6 DD3,4 DD1,2 HIS p HIS m HIS d CS 9,10 CS 7,8 CS 5,6 CS 3,4 CS 1,2 RVa Stim 3
200 ms
12:32:15 PM
12:32:16 PM
Fig. 29.2 Intracardiac electrograms. In this patient with mirror-image dextrocardia, typical flutter was counterclockwise (CCW) in the RA, as demonstrated by the wave front of activation on duodecapolar (DD) catheter. However, the inferior leads of the EKG inscribe F-waves that are positive, which is suggestive of clockwise (CW) flutter. This discrepancy
12:32:17 PM
is attributed to the fact that in the situs inversus heart, because of the rotated position of the atria and ventricles (mirror image of a situs solitus heart), CCW flutter appears CW (its mirror image) on the surface EKG. Hence, the position of the heart in the thorax must be kept in mind when evaluating CW versus CCW flutter based on a surface EKG
Case 30 Luis C. Sáenz and Miguel A. Vacca
Case Summary The patient is a 5-year-old male with a past medical history significant for paroxysmal palpitations which became incessant 8 months ago despite taking various anti-arrhythmic agents (AADs). The patient had worsening congestive heart failure and was hospitalized. He had no other significant past medical history. The tachycardia EKG taken in the emergency room is showed in Fig. 30.1. His echocardiogram showed left ventricular dilatation and ejection fraction of 38%. While the patient was in the hospital another tracing was taken showing spontaneous changes in his electrical rhythm (Fig. 30.2a and b). Variable atrioventricular conduction during the tachycardia with an isoelectric interval between the p waves is showed in Fig. 30.2a suggesting an atrial tachycardia as the mechanism of the arrhythmia. After
increasing the doses of the AAD medications, a transitory and regular rhythm with “1 per 1” atrioventricular conduction was registered, and is showed in Fig. 30.2b. Considering tachycardiomyopathy and the failure of the AAD to prevent arrhythmia recurrences, the patient was brought to the electrophysiology lab for ablation. CARTO was used during the procedure. A decapolar multielectrode catheter was introduced into the coronary sinus for mapping. In Fig. 30.3, the coronary sinus channels are showed from proximal (SC5) to distal (SCD). The third coronary sinus channel is not showed due to electrical noise. The intra-cardiac electrograms from the roving CARTO mapping and ablation catheter are showed as RFD and RFP for the bipolar electrograms that were taken from the distal1,2 and proximal3,4 par of electrodes, respectively. The unipolar electrogram from the electrodes 1 and 3 of the CARTO roving catheter are showed as 1 and 3, respectively. A CARTO
Fig. 30.1 EKG of tachycardia taken in the emergency room L.C. Sáenz (*) Jefe Servicio de Electrofisiología, Cardiólogo-Electrofisiólogo, Fundación Cardio Infantil-Instituto de Cardiología, Calle 163A No 28-60, Bogotá, Colombia M.A. Vacca Cardiac Electrophysiologist, Centro Internacional de Arritmias FCI, Fundacion Cardioinfantil, Instituto de Cardiologia, Calle 163 A No. 13B-60, Bogota, Colombia A. Natale et al. (eds.), Cardiac Electrophysiology, DOI: 10.1007/978-1-84996-390-9_30, © Springer-Verlag London Limited 2011
125
126
L.C. Sáenz and M.A. Vacca
Fig. 30.2 (a and b) Spontaneous changes in the electrical rhythm
right atrial and superior vena cava activation map was constructed during the tachycardia and shown in Fig. 30.3. The CARTO map suggested a focal origin of the tachycardia and showed the site of earliest electrical activation (in red) of the mapped chambers over the posterior aspect of the superior vena cava where fractionated and almost double electrograms were found and marked as blue points, Fig. 30.3.
Remarkably, the electrograms registered from the coronary sinus are almost isochronic with the electrograms taken from the earliest CARTO zone in the posterior aspect of the superior vena cava, Fig. 30.3. In order to simplify the case, the CARTO activation and the sequence of the propagation maps (from left to right) of the right atrium, superior vena cava, and coronary sinus is shown in Fig. 30.4.
Case 30
127
Fig. 30.3 Right atrium and superior vena cava CARTO activation map and intra-cardiac electrograms registered during tachycardia
shows a really broad zone of early activation (in red) suggesting a passive activation from another site.2 Moreover, the isochronal activation between the distal coronary sinus electrode and the earliest electrogram over the superior vena cava suggests that the cardiac site responsible for originating the tachycardia is activating these structures almost at the same time. In normal conditions this place would be equidistant between the distal coronary sinus and the superior vena cava. In this way, the origin of the tachyCase Discussion cardia would be in the left atrium.3 In fact, the EKG of the tachycardia (Fig. 30.1) shows a positive p wave in the preThe accuracy and reliability of a CARTO map are dependent cordial and inferior leads with negative p wave in aVL and on having an adequate number of contact points taken in iso-biphasic in DI suggesting that the origin of the tachycarorder to construct the chamber(s) of interest. The CARTO dia is from the left pulmonary veins.4 system constructs a map solely with information taken from A CARTO left atrial activation map integrating the elecsites where the tip of the roving catheter contacts cardiac tis- trical information of the right atrium, superior vena cava, and sue. The system can construct a map that provides informa- coronary sinus was performed and showed in Fig. 30.5. This tion about the site of earliest activation. When using the map showed the actual site of earliest activity to be over the CARTO system it is important to correlate the 3D color maps Carina between the left pulmonary veins as shown by the with information provided from the electrograms taken from tachycardia EKG. different cardiac places and verify and appropriate informaThe corresponding propagation CARTO map is showed tion provided by the system. in Fig. 30.6. The sequence shows a focal origin of the tachyIn Fig. 30.3, the CARTO activation map shows the ear- cardia between the left pulmonary veins (photo 1 of the liest activation zone over the posterior aspect of the supe- sequence) with spreading of the activation to the posterior rior vena cava. However, the intra-cardiac electrograms in wall and the roof of the left atrium (photo 2 and 3 of the Fig. 30.3 showed an almost isochronal activation between sequence). The activation reaches the anterior wall of the LA the distal coronary sinus electrode and the earliest electro- and the posterior aspect of the superior vena cava and almost gram over the superior vena cava. Moreover, the coronary at the same time the distal coronary sinus is depolarized from sinus electrodes show a discrete distal to proximal activa- the posterior wall of the LA (photo 4 of the sequence). tion. The bipolar electrograms registered from the posteFinally, the RA is passively activated through an isorior aspect of the superior vena cava are double potentials chronic front of waves from the posterior aspect of the supeof high amplitude and just 13 ms before the onset of the p rior vena cava and the distal coronary sinus (photo 4 of the wave. The corresponding unipolar electrograms from this sequence). place showed an r/S pattern. The analysis of the intra- The passive activation of one atrial chamber with the eleccardiac bipolar and unipolar electrograms suggests that trical activity originating in the other atrium is a phenomenon despite the CARTO showing that the earliest activation is that can cause confusion during mapping. Unnecessary radi over the posterior aspect of the superior vena cava, it would ofrequency applications maybe made targeting the passive be a passive electrical activation from another cardiac site.1 activation sites without having an effect on the tachycardia. Furthermore, the CARTO map of the superior vena cava Inter-atrial connections have been described and are By analyzing the EKG in Fig. 30.1, what is the possible origin of the tachycardia? Considering both the electrograms and CARTO maps, why does the posterior aspect of the superior vena cava and the distal coronary sinus have almost the same time of activation? What is the best next step to do in this case?
128
L.C. Sáenz and M.A. Vacca
Fig. 30.4 CARTO activation and propagation maps of the right atrium, superior vena cava, and coronary sinus shown in a posterior view
Posterior view
Fig. 30.5 The CARTO activation map integrating the electrical information of the right atrium, superior vena cava, coronary sinus, and the left atrium
Anterior view
Case 30
129
Fig. 30.6 CARTO propagation map integrating the electrical information of the right atrium, superior vena cava, coronary sinus, and the left atrium shown as a sequence of pictures from the superior to inferior views
130
L.C. Sáenz and M.A. Vacca
responsible for this phenomenon.5 The most recognized interatrial connection between the atrial chambers is through the Bachmann Bundle, which connects the left atrial appendage and the superior portion of the Crista Terminalis in the anterior aspect of the superior vena cava, Fig. 30.7. Another connection is between the posterior wall of the LA and the coronary sinus and between the coronary sinus and the RA, Fig. 30.7. There are other less recognized inter-atrial connec-
tions as through the fossa ovalis and between the right pulmonary veins and the superior vena cava, Fig. 30.8. Remarkably, in this particular case the earliest activation of the RA was founded over the posterior aspect of the superior vena cava. So, it seems to correspond to a passive activation through a connection between the right superior pulmonary vein and this part of the superior vena cava as showed in Fig. 30.8.
Anterior-superior view
LAO view
Bachmann CT
Fig. 30.7 Anatomical and CARTO pictures showing the inter-atrial connections
LAO view
Fig. 30.8 Drawing and CARTO pictures showing the inter-atrial connections
Posterior view
Case 30
The atrial tachycardia was slowed by radiofrequency applications over the carina between the left pulmonary veins as showed in Fig. 30.9. This site revealed atrial bipolar electrogram of low amplitude, fractionated, and earlier 52 ms than the onset of the p wave. The corresponding unipolar electrogram showed a QS pattern. See Fig. 30.3 for comparison. The EKG post ablation showed a p wave with a positive/ negative morphology in V1, positive in lead I and aVL, and was predominantly negative in the inferior leads suggesting an escape low CT rhythm, Fig. 30.10a. Retrospectively, the
131
organized and 1:1 atrioventricular conduction rhythm (showed in Fig. 30.2b and magnified in Fig. 30.10a) would be the same ablated atrial tachycardia with fixed exit block of the conduction from the pulmonary vein to the LA that can be confused with a normal sinus rhythm. This case points out that although electroanatomic mapping is a useful tool, the information obtained by the system needs to be validated with electrograms and entrainment. The inter-atrial connections can muddy the interpretation of the activation map. Because of that registration of electrical activity from only one chamber can be misleading.
Fig. 30.9 CARTO and ICE showing the location of successful RF ablation
132
a
L.C. Sáenz and M.A. Vacca Pre ablation
Post ablation I II III AVR AVL AVF V1 V2 V3 V4 V5 V6
Fig. 30.10 (a and b) Comparison between the pre- and post-ablation p waves morphology
References 1. Stevenson WG, Soejima K. Recording techniques for clinical electrophysiology. J Cardiovasc Electrophysiol. 2005;16:1017-1022. 2. Markowitz SM, Lerman BB. How to interpret electroanatomic maps. Heart Rhythm. 2006;3:240-246. 3. Lemery R, Soucie L, Martin B, et al. Human study of biatrial electrical coupling. Circulation. 2004;110:2083-2089.
4. Kistler PM, Roberts-Thomson KC, Haqqani HM, et al. P-wave morphology in focal atrial tachycardia. J Am Coll Cardiol. 2006;48:1010-1017. 5. Ho SY, Sanchez-Quintana D, Cabrera JA, et al. Anatomy of the left atrium: implications for radiofrequency ablation of atrial fibrillation. J Cardiovasc Electrophysiol. 1999;10:1525-1533.
Case 31 Bradley P. Knight
Case Summary A young woman with a permanent form of reciprocating tachycardia (PJRT) underwent an electrophysiology
procedure. The 12-lead ECG is shown in Fig. 31.1. The results of pacing maneuvers were consistent with orthodromic AV reentry using a slowly conducting accessory pathway (AP). Where is the AP most likely located?
Fig. 31.1 A 12-lead rhythm strip showing a nearly incessant, supraventricular tachycardia with a long RP interval
B.P. Knight Division of Cardiology, Northwestern Medical Center, 676 N. St. Clair, Suite 600, Chicago, IL 60611 A. Natale et al. (eds.), Cardiac Electrophysiology, DOI: 10.1007/978-1-84996-390-9_31, © Springer-Verlag London Limited 2011
133
134
B.P. Knight
Case Discussion The most likely location for an AP with decremental conduction that participates in PJRT is the posterior septum. Therefore, efforts to map the atrial insertion during tachycardia should initially focus in this region. However, it is not uncommon for these pathways to be located elsewhere. At least two studies have found that the site of earliest retrograde atrial activation occurs at annular locations other than the posteroseptal region in approximately one fourth of cases. In this case, the earliest retrograde activation during single echo beats was along the lateral mitral annulus across from the distal coronary sinus electrode (Fig. 31.2). Radiofrequency ablation at this site eliminated AP conduction. Less than 5% of cases are found in the left lateral location.
Fig. 31.2 This tracing was recorded during catheter ablation of a supraventricular tachycardia. Shown are surface electrograms from leads I, II, III, V1, and V5, and the intracardiac electrograms from the high right atrium (HRA), the ablation catheter (Abl) positioned at the lateral mitral annulus where the earliest local activation could be recorded, and the coronary sinus (CS). The vertical line shows that the earliest local atrial activation precedes the onset of the p-wave on the surface tracing
Bibliography Gaita F, Haissaguerre M, Giustetto C, et al. Catheter ablation of PJRT with RF current. J Am Coll Cardiol. 1995;25:648-654. Meiltz A, Weber R, Halimi F, et al. PJRT in adults: Peculiar Features and Results of RF Ablation. Europace. 2006;8:21-28.
Case 32 Luigi Di Biase, Rodney P. Horton, and Andrea Natale
Case Summary A 66-year-old male with essential hypertension and a history of bilateral hernia surgery, was self-referred to our institution regarding persistent shortness of breath status post-“redo” pulmonary vein isolation performed at another institution. At admission he denied any chest discomfort, palpitations, presyncope, or syncope. The patient began experiencing symptomatic episodes of atrial fibrillation in 2003. His echocardiogram and blood screening resulted within the normal range. The arrhythmia was initially treated both with propafenone and flecainade at the appropriate dosages. These AADs were ineffective. Since his AF became more persistent, the patient underwent several cardioversions. In 2006 he finally underwent pulmonary vein isolation (PVI). The procedure had no complications and a CT scan performed 3 months after the ablation showed no pulmonary vein stenosis. Five months following the ablation, the patient went into atrial flutter which required cardioversion. In 2008 the patient underwent a repeat procedure. One month following the second ablation procedure, he started experiencing progressively worsening dyspnea on exertion with daily activities such as walking across the street. He also reported orthopnea. At admission, his blood screening and physical examination resulted normal.
L. Di Biase (*) Department of Electrophysiology, St. David’s Medical Center, 1015 32nd Street, Suite 506, Austin, TX 78705 email:
[email protected] R.P. Horton Department of Electrophysiology, Texas Cardiac Arrhythmia Institute, 1015 East 32nd Street, Austin, TX 78705 A. Natale Texas Cardiac Arrhythmia Institute, St. David’s Medical Center, 1015 East 32nd Street, Suite 408, Austin, TX 78705
Which should be the diagnostic and therapeutic steps for this patient? a) Perform an Echocardiogram b) Perform a Transesophageal Echocardiogram c) Perform a CT scan e/o MRI scan d) Perform a CT scan and a Transesophageal Echocardiogram e) Perform a ventilation/perfusion scan f) Perform a chest x ray g) Perform all the above
Case Discussion The patient had a CT scan and an echocardiogram 9 months after the redo procedure. The echocardiogram showed normal EF and an enlarged pulmonary artery with increased pulmonic valve velocities consistent with pulmonary hypertension. Additionally there was mild right ventricular hypertrophy, and mild tricuspid regurgitation. The TEE was technically difficult and with suboptimal echocardiographic images. No thrombus was appreciable in the left atrium and the left atrium appendage. Solely the left superior pulmonary vein was visualized. EF was estimated at 60%. Generally all the structures were poorly visualized. The Doppler echocardiogram showed increased velocities in the distal portion of the left superior pulmonary vein, consistent with a significant stenosis. The CT scan showed complete occlusion of the LIPV and severe stenosis of both left and right superior pulmonary veins. An enlarged pulmonary artery was also noted (Fig. 32.1 and 32.2). Based on the CT reports which should be the treatment stategy? a) Anticoagulation and clinical follow up b) Dilation with baloon and anticoagulation c) Dilation with balloon, anticoagulation and follow up with VQ/scan d) None of the above e) All the above
A. Natale et al. (eds.), Cardiac Electrophysiology, DOI: 10.1007/978-1-84996-390-9_32, © Springer-Verlag London Limited 2011
135
136
L. Di Biase et al.
There were no significant gradient across these stenosis following the dilation. Based on the good angiographic results of the balloon angiography the patient should: a) Stop anticoagulation therapy b) Continue anticoagulation therapy and repeat the CT scan at follow up c) Continue anticoagulation therapy and repeat TEE at follow up d) All the above e) None of the above
Fig. 32.1 CT scan showing complete occlusion of the LIPV and severe stenosis of both left and right superior pulmonary veins
Fig. 32.2 CT scan showing an enlarged pulmonary artery with severe stenosis of the right and left superior pulmonary veins
Because of the significant dyspnea and pulmonary hypertension related to pulmonary vein stenosis of at least three pulmonary veins, the decision was made to undergo balloon angioplasty and possible stenting. The left superior, left inferior, and right superior pulmonary veins were found severely occluded (³90%) at the angiograms and were dilated with balloon angioplasty.
The reported incidence of PV stenosis/occlusion defined as >70% narrowing or ³90% affects 3.4% of patients following catheter ablation of atrial fibrillation.1–3 The incidence has decreased in the most recent years due the utilization of different techniques for the PV isolation that limit the burnings at the antrum of the pulmonary veins which is at a considerable distance from the true PV ostia.1 The clinical presentation of these patients is variable; from totally asymptomatic or with mild dyspnea to severe dyspnea with hemoptysis, fever, or pleuritic chest pain.1,3 The pathogenesis is due to an initial ablative insult that precipitates a healing reaction culminating in an endovascular contraction and proliferation of the elastic lamina/intima. Misdiagnosis is very common in these patients (pulmonary embolism, lung cancer, pneumonia, and new onset of asthma, are the most common1,3), because symptoms may occur far from the procedural time. This is why imaging following catheter ablation is crucial also in asymptomatic patient. In fact, PV stenosis may progress insidiously. Patients can present with a variety of respiratory symptoms, but may also remain asymptomatic especially when only one vein demonstrates severe stenosis. Chest x-ray is usually not helpful in diagnosing this condition. As shown in this case, TEE does not always provide clear images of the pulmonary veins. CT scan and/or magnetic resonance imaging are the best tools for diagnosis.4,5 Many groups suggest that the assessment of PV diameter using CT scan or MRI 3 months after ablation provides the best identification of PV stenosis since the caliber remain relatively stable beyond 3 months after ablation. However, late progression from a mild to a severe PV stenosis has been described and a repeat imaging is required for any patient who develops new symptoms consistent with stenosis.1 In patients with moderate to severe stenosis, ventilation/perfusion (V/Q) scan may be useful because it provides a good measure of the lung functionality. The CSI index (cumulative stenosis index [CSI] = sum of the percent stenosis of the unilateral veins divided by the total number of ipsilateral veins) has been proposed with a cutoff value of 75% to identify patients at greatest risk of severe symptoms and lung diseases. In these patients,
Case 32
early and, when required, repeated PV intervention should be considered for restoration of pulmonary flow and prevention of associated lung disease.1 Late opening of the vessel although feasible will not be able to reduce patient’s symptoms and restore lung functionality. In the absence of symptoms, there is no consensus on the best treatment strategy and the CSI index may be useful to identify patients at higher risk. Balloon angioplasty with or without stenting has been shown to achieve satisfactory results although restenosis requiring repeat intervention is necessary in nearly 45–50% of patients.1,6 As shown in this case, after the second ablation, the patient did not undergo CT scan. As described earlier in the text, imaging is very important to detect stenosis even in asymptomatic patients. The dyspnea reported by the patient was underestimated for several months. This resulted in severe pulmonary hypertension and pulmonary artery dilatation (as demonstrated by the CT scan). In such a case, late dilatation of the occluded pulmonary veins although angiographically successful may not solve the patient’s symptoms.
137
References 1. Di Biase L, Fahmy TS, Wazni OM, et al. Pulmonary vein total occlusion following catheter ablation for atrial fibrillation: clinical implications after long-term follow-up. J Am Coll Cardiol. 2006;48: 2493-2499. 2. Saad EB, Marrouche NF, Saad CP, Natale A et al. Pulmonary vein stenosis after catheter ablation of atrial fibrillation: emergence of a new clinical syndrome. Ann Intern Med. 2003 Apr 15; 138(8):634-638. 3. Saad EB, Rossillo A, Saad CP, et al. Pulmonary vein stenosis after radiofrequency ablation of atrial fibrillation: functional characterization, evolution, and influence of the ablation strategy. Circulation. 2003;108:3102-3107. 4. Packer DL, Keelan P, Munger TM, et al. Clinical presentation, investigation, and management of pulmonary vein stenosis complicating ablation for atrial fibrillation. Circulation. 2005;111: 546-554. 5. Neumann T, Sperzel J, Dill T, et al. Percutaneous pulmonary vein stenting for the treatment of severe stenosis after pulmonary vein isolation. J Cardiovasc Electrophysiol. 2005;16:1180-1188. 6. Qureshi AM, Prieto LR, Latson LA, et al. Transcatheter angioplasty for acquired pulmonary vein stenosis after radiofrequency ablation. Circulation. 2003;108:1336-1342.
Case 33 M. Eyman Mortada, Jasbir S. Sra, and Masood Akhtar
Case Summary A 55-year-old male presented with recurrent symptoms of dizziness and palpitations. He was found to have stable vitals and abnormal rhythm (Fig. 33.1). The 12-lead ECG at baseline during sinus rhythm is shown in Fig. 33.2. The patient underwent a complex electrophysiology study. The intracardiac tracing shown in Fig. 33.3 demonstrates the wide complex tachycardia rhythm.
What is the diagnosis? Radiofrequency ablation was performed successfully. Where was the ablation performed?
Case Discussion In defining a wide complex tachycardia, it is important to differentiate between ventricular tachycardia (VT) and supraventricular tachycardia (SVT) (associated with aberrancy,
Fig. 33.1 At presentation, the patient was found to have stable vitals and abnormal rhythm demonstrated in this 12-lead ECG
M.E. Mortada (*), J.S. Sra, and M. Akhtar Department of Electrophysiology, Aurora Cardiovascular Services, Aurora Sinai/Aurora St. Luke’s Medical Centers, University of Wisconsin School of Medicine and Public Health, 2801 W. Kinnickinnic River Parkway, #777, Milwaukee, WI 53215 e-mail:
[email protected] A. Natale et al. (eds.), Cardiac Electrophysiology, DOI: 10.1007/978-1-84996-390-9_33, © Springer-Verlag London Limited 2011
139
140
preexcitation, preexistent wide QRS complex, or antidromic AV reentry tachycardia). All of these are possible diagnostic options in this case. The seventh complex on the wide QRS complex tachycardia is premature and results in an earlier initiation of the tachycardia after the premature beat. This response fits all of the above mentioned diagnostic possibilities. If this is a VT, the premature beat could be a premature ventricular capture (PVC) that entrained the ventricular tachycardia and accelerated the rhythm for that one beat (Fig. 33.4). If this is atrial flutter with 2:1 AV conduction or reentry atrial tachycardia associated with left bundle branch block, the premature beat could be a premature atrial capture (PAC) that entrained the reentry circuit and accelerated the rhythm for that one beat (though atrial flutter option is the least possible, due to the fast rate of the atrial flutter, 366 bpm [160 ms]) (Fig. 33.5). If this is an orthodromic AV reentry tachycardia with left bundle branch block, the premature beat could be a PAC or PVC that entrained the circuit and accelerated the rhythm for that one beat (Fig. 33.6). If this is an AV nodal reentry tachycardia with left bundle branch block, the premature beat could be a PAC that entrained the circuit and accelerated the rhythm for that one
Fig. 33.2 The patient’s 12-lead ECG at baseline during sinus rhythm
M.E. Mortada et al.
beat (Fig. 33.7). Intravenous adenosine affects the atrioventricular node; hence it affects almost all SVTs by terminating the tachycardia or producing atrioventricular dissociation. Few ventricular tachycardias are sensitive to adenosine (e.g., normal heart VT), thus response to adenosine is no help in making a diagnosis. The absence of structural heart disease is an important piece of information, increasing the likelihood that the tachycardia is SVT rather than VT in origin. The most helpful piece of information for differential diagnosis in this case was the baseline 12-lead ECG, which showed the same QRS morphology. Therefore, VT could be easily excluded. Acceleration of the tachycardia by one premature beat, as seen in the first ECG (the seventh beat), favors the diagnosis of orthodromic AV reentry tachycardia (AVRT) over AV nodal reentry tachycardia (AVNRT). Intracardiac electrocardiograms taken during the tachycardia revealed the activation of the atria to be from the distal to the proximal coronary sinus, followed by the high right atrium and His (septum). It also revealed constant VA linking. These findings confirmed the diagnosis of an orthodromic AVRT using a left lateral free-wall accessory pathway and the best location for successful radiofrequency ablation is the lateral side of the mitral valve annulus.
Case 33
141
Fig. 33.3 Intracardiac tracing demonstrating the wide complex tachycardia rhythm (Rhythm tracing from top to bottom: Lead I, Lead II, Lead V1, high right atrium recording “HRA,” proximal coronary sinus recording (CS 9,10) to distal coronary sinus recording (CS 1,2), and His bundle recording “proximal HSp, and distal HSd.”) HRAp: proximal high right atrium; CS: coronary sinus; HSp: proximal His; HSd: distal His
Fig. 33.4 The seventh complex on the wide QRS complex tachycardia is premature and results in an earlier initiation of the tachycardia after the premature beat. If this patient has a ventricular tachycardia (VT), the premature beat could be a premature ventricular capture that entrained the VT and accelerated the rhythm for that one beat
Fig. 33.5 If this patient has atrial flutter with 2:1 AV conduction associated with left bundle branch block, the premature beat could be a premature atrial capture that entrained the atrial flutter and accelerated the rhythm for that one beat (though this option is the least possible, due to the fast rate of the atrial flutter--366 bpm “160 ms”)
Fig. 33.6 If this patient has an orthodromic AV reentry tachycardia with left bundle branch block, the premature beat could be a PAC or PVC that entrained the circuit and accelerated the rhythm for that one beat
Fig. 33.7 If this patient has an AV nodal reentry tachycardia with left bundle branch block, the premature beat could be a PAC that entrained the circuit and accelerated the rhythm for that one beat
Case 34 Antonio Rossillo, Sakis Themistoclakis, Aldo Bonso, Andrea Corrado, and Antonio Raviele
Case Summary
Case Discussion
The patient is a 28-year-old female with frequent runs of SVT. No relevant structural changes on echocardiography. Based on Figs. 34.1 and 34.2, what is the likely mechanism of the tachycardia?
This tachycardia is a long RP tachycardia, thus the differential diagnosis includes atypical AVNRT, atrial tachycardia, or AVRT that utilizes a slowly conducting AP. When a single VPC is delivered the tachycardia terminates without reaching
Fig. 34.1 12-lead ECG taken during SVT (paper speed 25 mm/s) showing a negative P wave in the inferior leads with a short PQ interval and a heart rate of 120 bpm
A. Rossillo (*), S. Themistoclakis, A. Bonso, and A. Corrado Cardiovascular Department, Ospedale dell’Angelo, Mestre, Venice, Italy A. Raviele Chief of Cardiology, Chief of Cardiovascular, Cardiovascular Department, Ospedale dell’Angelo, Mestre, Venice, Italy A. Natale et al. (eds.), Cardiac Electrophysiology, DOI: 10.1007/978-1-84996-390-9_34, © Springer-Verlag London Limited 2011
143
144
the atrium. This makes AT unlikely. Distinguishing AVRT that utilizes a slowly conducting AP versus AVNRT can be difficult, but as the tachycardia terminated during a HIS
Fig. 34.2 Intracardiac recordings taken at baseline during the electrophysiology study (paper speed 100 mm/s). Four surface ECG leads are shown (I, aVF, V1, V6), one bipolar recording from the pacing catheter (HRA), three bipolar recordings from the distal His bundle region (HIS D = distal, HIS I = medial, and HIS P = proximal), two bipolar recordings from the coronary sinus (CS1 = distal CS and CS4 prox CS), and the distal bipolar recording of the mapping catheter (MC D Bi). A single paced beat from the right ventricle with a coupling interval allows for anterograde activation of the His bundle which interrupted the tachycardia twice. (A atrium, V ventricle, and H His bundle)
Fig. 34.3 Intracardiac recordings (paper speed 200 mm/s). Same display as shown in Fig. 34.2. Ablation site: earliest activation on MC D Bi
A. Rossillo et al.
refractory VPC, the likely diagnosis is AVRT or PJRT. This tachycardia is often clinically frequent or incessant. In Fig. 34.3 and 34.4 the ablation site is shown.
Case 34
145
Fig. 34.4 RAO and LAO projection of mapping catheter at the ablation site
RAO
Bibliography Coumel PH, Attuel P, Leclerque JF. Permanent form of junctional reciprocating tachycardia: mechanism, clinical and therapeutic impli cation. In: Narula OS, ed. Cardiac Arrhythmias. Electrophysiology, Diagnosis and Management. Baltimore/London: Williams & Wilkins; 1979:347-363. Critelli G, Scherillo M, Monda V, D’Ascia C, Musumeci S, Antignano A. Transvenous catheter ablation of the accessory atrioventricular
LAO
p athway in the permanent form of junctional reciprocating tachycardia. Am J Cardiol. 1985;55:1639-1641. Farré J, Ross D, Wiener I, Bär FW, Vanagt EJ, Wellens HJ. Reciprocal tachycardias using accessory pathways with long conduction times. Am J Cardiol. 1979;44:1099-1109. Smith RT, Gillette PC, Massumi A, McVey P, Garson A Jr. Transcatheter ablative techniques for treatment of the permanent form of junctional reciprocating tachycardia in young patients. J Am Coll Cardiol. 1986;8:385-390.
Case 35 Yash Y. Lokhandwala, Anoop K. Gupta, and Ranjan K. Thakur
Case Summary
Case Discussion
A 47-year-old man presented with recurrent tachycardia (Fig. 35.1) which is responsive to verapamil. The patient has a normal heart. His ECG is normal in sinus rhythm. In the EP lab he is diagnosed with AVRT. Explain what occurs during RF delivery in Fig. 35.2.
Local AP conduction in the RF channel is eliminated evidenced by prolongation of the local VA interval. Yet, an eccentric atrial activation suggesting a left-sided AP persists in the CS recordings. RF delivery at a different location resulted in VA block (Fig. 35.3). There were two left-sided APs: the first was anterolateral and the second was posterolateral; they were 3–4 cm away from each other.
Y.Y. Lokhandwala (*) KEM Hospital, Parel, Mumbai, India e-mail:
[email protected] A.K. Gupta Apollo Hospital, Ahmedabad, India e-mail:
[email protected] R.K. Thakur Thoracic and Cardiovascular Institute, Sparrow Health System, Michigan State University, 405 West Greenlawn, Suite 400, Lansing, MI 48910 A. Natale et al. (eds.), Cardiac Electrophysiology, DOI: 10.1007/978-1-84996-390-9_35, © Springer-Verlag London Limited 2011
147
148
Y.Y. Lokhandwala et al. Your Hospital Name in File HEADING.TXT
Version WIN200 : EPTRACER V0.48
2008017 I II III AVR AVL AVF V1 V2 V3 V4 V5 V6 10 mm/mV 25mm/s
Fig. 35.1 The ECG suggests that this is likely AVRT, with a left-sided AP (long RP, ST elevation in aVR and ST depression in lead I)
Case 35
149 Your Hospital Name in File HEADING.TXT
I
2008017
AVF V1 V6 CS910 CS78 CS56 CS34 CS12 HISD HISP RFD RFP 10 mm/mV 100mm/s
Fig. 35.2 Intracardiac EGMs during RF energy application. What happens?
Version WIN200 : EPTRACER V0.48
150
Y.Y. Lokhandwala et al. Your Hospital Name in File HEADING.TXT
Version WIN200 : EPTRACER V0.48
2008017 I AVF V1 V6 CS910 CS78 CS56 CS34 CS12 HISD HISP RFD RFP 10 mm/mV 100mm/s Offline printed on 17-05-2008 at 14:23:17.
Fig. 35.3 VA block is achieved
Case 36 Eric Buch, Shiro Nakahara, Marmar Vaseghi, Noel G. Boyle, and Kalyanam Shivkumar
Case Summary A 60-year-old female with a history of left atrial myxoma, resected 9 years ago, subsequently developed palpitations, fatigue, and tachycardia. Despite antiarrhythmic medications, including sotalol and amiodarone, and multiple
cardioversions, she continued to experience tachycardia and was referred for electrophysiology study and catheter ablation. Surface electrocardiogram showed wide-complex tachycardia with a ventricular rate of 115 beats per minute (Fig. 36.1). What is the differential diagnosis of this arrhythmia based on the electrocardiogram?
Fig. 36.1 Surface electrocardiogram shows a regular wide-complex tachycardia at approximately 115 beats per minute
E. Buch (*), S. Nakahara, M. Vaseghi, N.G. Boyle, and K. Shivkumar UCLA Cardiac Arrhythmia Center, David Geffen School of Medicine at UCLA, 10833 Le Conte Ave, A2-237 CHS, Los Angeles, CA 90095-1679 e-mail:
[email protected] A. Natale et al. (eds.), Cardiac Electrophysiology, DOI: 10.1007/978-1-84996-390-9_36, © Springer-Verlag London Limited 2011
151
152
Case Discussion Diagnostic electrophysiology catheters were placed in the high right atrium, at the His bundle, the right ventricular apical septum, and in the coronary sinus. Intracardiac electrograms were recorded during tachycardia (Fig. 36.2).
Fig. 36.2 Intracardiac recordings during tachycardia. Left atrial activation as seen in coronary sinus catheter proceeds from medial to lateral, consistent with a focus or reentrant circuit in the right atrium
E. Buch et al.
Electroanatomic mapping was used to show the right atrial activation sequence. The tachycardia circuit was around the lateral right atriotomy scar from previous cardiac surgery. Ablation was performed at the superior margin of the scar and the tachycardia terminated (Fig. 36.3). It could not be reinduced on multiple attempts.
Fig. 36.3 Left anterior oblique fluoroscopic view of intracardiac electrophysiology catheters. The tip of the ablation catheter is in the lateral right atrium at the superior margin of the atriotomy scar, at the site of tachycardia termination
Case 37 Bradley P. Knight
Case Summary A 25-year-old woman with Wolff–Parkinson–White syndrome was referred for catheter ablation. The 12-lead EKG is consistent with an anteroseptal accessory pathway (AP) (Fig. 37.1). A previous attempt at ablation of the AP was aborted due to its proximity to the His bundle. Another attempt at ablation was made. The baseline rhythm was sinus with ventricular preexcitation. Orthodromic AVRT was inducible and the AP had a short refractory period that was less than 250 ms. The earliest ventricular activation during sinus rhythm was at a site where there was also a large His bundle recording during block in the AP. How would you proceed?
Case Discussion In this case, the patient has symptomatic WPW with a high risk AP. There is a significant risk of causing irreversible AV block with RF current. Cryoenergy ablation is an
alternative to radiofrequency ablation. One advantage of cryoenergy over radiofrequency current is the ability to “cryo-map” to minimize the likelihood of inadvertent AV block.1 As the ablation electrode temperature is lowered below freezing, the tissue reaches a temperature where temporary conduction block occurs well before it reaches a temperature where permanent tissue destruction occurs. This permits the ability to monitor during ablation for undesirable results, such as AV block, before a permanent lesion is created. Fig. 37.2 shows early activation at the para-Hisian region with an activation time of -30 ms. Fig. 37.3 shows disappearance of ventricular preexcitation when the temperature is lowered during cryoablation. As an iceball forms on the tip of the electrode, the electrogram is lost. After rewarming of the electrode occurs, a large His bundle recording can be seen (Fig. 37.4). After elimination of the AP, AV conduction was intact and there was no VA conduction. The AP did not recover during long-term follow-up. Cryoablation should be considered when attempting to ablate an AP that is located in the para-Hisian or midseptal region, to minimize the risk of AV block.
B.P. Knight Division of Cardiology, Northwestern Medical Center, 676 N. St. Clair, Suite 600, Chicago, IL 60611 e-mail:
[email protected] A. Natale et al. (eds.), Cardiac Electrophysiology, DOI: 10.1007/978-1-84996-390-9_37, © Springer-Verlag London Limited 2011
153
154
B.P. Knight
Fig. 37.1 This 12-lead EKG shows ventricular preexcitation consistent with an anteroseptal accessory pathway
Successful cryo site
Fig. 37.2 This tracing was recorded during catheter ablation of an anteroseptal accessory pathway. Shown are surface electrograms from leads I, II, III, V1, and V5, and the intracardiac electrograms from the high right atrium (HRA), the radiofrequency ablation catheter (Abl) positioned near the His bundle region, the cryo catheter (CRYO) with the tip positioned at the earliest local ventricular activation, and the right ventricular apex (RVA). The local ventricular activation recorded from the cryo catheter is 30 msec prior to the onset of the delta wave
−30 ms
University of Chivago
Case 37
155
Fig. 37.3 This figure shows disappearance of preexcitation during cryo-ablation. The format is the same as figure 37.2
During cryoablation
University of Chicago
Appearance of his bundle during rewarming after cryo
Fig. 37.4 This figure shows the reappearance of the His-bundle recording after rewarming of the cryo-ablation electrode. The format is the same as figure 37.2
University of Chicago
Reference 1. Gaita F et al. Safety and efficacy of cryoablation of accessory pathways adjacent to the normal conduction system. J Cardiovasc Electrophysiol. 2003;14:825-829.
Case 38 Luigi Di Biase, Rodney P. Horton, and Andrea Natale
Case Summary A 65-year-old male with a 15-year history of symptomatic persistent atrial fibrillation was referred to our institution. The patient had a normal left ventricular ejection fraction. The patient denied chest discomfort, presyncope, or syncope. Atrial fibrillation was initially chemically cardioverted with Quinidine and Diltiazem. A few years later the drugs were ineffective and the patient underwent three electrical cardioversions. Quinidine and Diltiazem were replaced with Tikosyn. Because Tikosyn was ineffective, the patient decided to undergo pulmonary vein isolation. Five months following the procedure, the patient experienced symptoms related to atrial flutter with rapid ventricular response. For these reasons the patient underwent a second ablation procedure. During the second procedure, the patient did not show any recovery of conduction around the pulmonary veins. The presenting arrhythmia was atrial flutter, which was terminated during ablation in the coronary sinus. Two months after the second procedure the patient developed recurrence of left atrial flutter. The CT scan after the second procedure is shown (Figs. 38.1 and 38.2). What do you propose next for management of this patient’s rhythm abnormality, would a repeat ablation be an option, if so, what is the ablation target? What do you propose next? a) Clinical follow-up with AADs for rate control b) Propose a MAZE or COX procedure
L. Di Biase (*) Department of Electrophysiology, St. David’s Medical Center, 1015 32nd Street, Suite 506, Austin, TX 78705 e-mail:
[email protected] R.P. Horton Department of Electrophysiology, Texas Cardiac Arrhythmia Institute, 1015 East 32nd Street, Austin, TX 78705 e-mail:
[email protected] A. Natale Texas Cardiac Arrhythmia Institute, St. David’s Medical Center, 1015 East 32nd Street, Suite 408, Austin, TX 78705
c) Repeat catheter ablation targeting non PV site d) AV node ablation and pacing e) None of the above
Case Discussion The patient was taken to the electrophysiology laboratory for a repeat ablation. During the third procedure, before ablation in the left atrium, ablation in the persistent left superior vena cava (SVC) was performed. Once isolation was achieved, the ablation catheter was moved to the left atrium. The atrial flutter was prolonged and interrupted during ablation along the mitral annulus. During and after the infusion of high-dose isoproterenol (20 mcg/min), there were no evidence of additional firing sites. After 9 months of follow-up, the patient was asymptomatic and free from arrhythmia recurrences. This case shows that in a number of patients electrical disconnection of the pulmonary veins is not sufficient to prevent arrhythmic recurrences. This is possible in patients with left atrial scar preceding the ablation procedure, non-paroxysmal AF patients, females,1 and in patients with venous anomaly including persistent left SVC.2 In these patients, isolation of the left SVC is necessary together with the isolation of the pulmonary veins to prevent recurrences. Whether isolation of the left SVC without isolation of the PVs would have been sufficient to treat these patients is not known. Persistence of the left SVC is a congenital anomaly resulting from an abnormal development of the coronary sinus. In these patients, the left cardinal vein does not obliterate during the fetal life, and on the contrary, persists draining into either the left atrium or the right atrium through an enlarged CS. The described prevalence of left SVC persistence varies between 0.3% and 2% in individuals with a normal heart.3,4 A persistence of the left SVC can be diagnosed with echocardiography based an unusual enlarged CS, but the gold standards are CT scan and MRI.
A. Natale et al. (eds.), Cardiac Electrophysiology, DOI: 10.1007/978-1-84996-390-9_38, © Springer-Verlag London Limited 2011
157
158 Fig. 38.1 CT scan showing the left atrium, the pulmonary veins, and the persistent LSVC
L. Di Biase et al.
In the above-mentioned case, spontaneous ectopies originating at different levels in the left SVC were present. It is always important to look for unusual sources of AF/Aflutter especially in patients presenting for repeat procedures. Unusual sources of AF should be considered after the failure of one or more procedures for AF ablation. Isolation of the PVs is not sufficient to prevent recurrences when a left SVC is present. Thus, diagnosis and isolation of the left SVC appears critical to avoid AF recurrence in all patients with AF and with this venous anomaly.
References
Fig. 38.2 CT scan showing a persistent LSVC draining into the coronary sinus
The left SVC when present is always a source of ectopies that can initiate AF through the electrical connections with the LA and CS. Hsu et al.5 suggested that as a consequence of the abnormal embryologic development, the persistent of the left SVC may be associated with the presence of electrical tissue, responsible for arrhythmias.
1. Verma A, Wazni OM, Marrouche NF, et al. Pre-existent left atrial scarring in patients undergoing pulmonary vein antrum isolation: an independent predictor of procedural failure. J Am Coll Cardiol. 2005;45:285-292. 2. Elayi CS, Fahmy TS, Wazni OM, Patel D, Saliba W, Natale A. Left superior vena cava isolation in patients undergoing pulmonary vein antrum isolation: impact on atrial fibrillation recurrence. Heart Rhythm. 2006;3:1019-1023. 3. Gonzalez-Juanatey C, Testa A, Vidan J, et al. Persistent left superior vena cava draining into the coronary sinus: report of 10 cases and literature review. Clin Cardiol. 2004;27:515-518. 4. Fraser RS, Dvorkin J, Rossall RE, Eidem R. Left superior vena cava: a review of associated congenital heart lesions, catheterization data and roentgenologic findings. Am J Med. 1961;31:7 11-716. 5. Hsu LF, Jais P, Keane D, et al. Atrial fibrillation originating from persistent left superior vena cava. Circulation. 2004;109: 828-832.
Case 39 M. Eyman Mortada, Jasbir S. Sra, and Masood Akhtar
Case Summary A 38-year-old female complained of recurrent episodes of lightheadedness and palpitations over several months. She came to the emergency room with the same symptoms, at which time her 12-lead ECG was as follows (Figs. 39.1–39.4). Two ventricular extrastimuli (S1) at cycle lengths of 400 ms were administered during the tachycardia (Fig. 39.5). What is the best initial management, and what is the most likely diagnosis?
Case Discussion The P-wave occurring between the two R-waves, seen in the first 12-lead ECG of this young female with documented normal PR interval during sinus rhythm in the past, suggests a diagnosis of supraventricular tachycardia, specifically AV nodal reentry tachycardia (AVNRT) with 2:1 AV conduction. Since we were unable to exclude other possibilities, such as atrial tachycardia a complex electrophysiology study was required. When the patient’s rhythm spontaneously changed from a 460- to a 230-ms R-R cycle length (Figs 39.1 and 39.3), it revealed the presence of 2:1 AV conduction, which then
M.E. Mortada (*), J.S. Sra, and M. Akhtar Department of Electrophysiology, Aurora Cardiovascular Services, Aurora Sinai/Aurora St. Luke’s Medical Centers, University of Wisconsin School of Medicine and Public Health, 2801 W. Kinnickinnic River Parkway, #777, Milwaukee, WI 53215 e-mail:
[email protected] changed to 1:1 AV conduction. This excludes the diagnosis of accessory pathway-mediated tachycardia, as it would not occur in the presence of 2:1 AV block. The patient was symptomatic with her tachycardia, so the best next step in management is to convert her to normal rhythm and then perform further work-up to diagnose and treat her condition. She was hemodynamically was stable, therefore DC cardioversion is not indicated, but adenosine IV bolus is the best first-line approach. b-blockers or calcium channel blockers are acceptable therapies, yet, complex electrophysiology study post conversion is more appropriate to confirm the diagnosis and cure the patient’s arrhythmia by ablation therapy. The first intracardiac tracing (Fig. 39.4) showed narrow complex tachycardia with two atrial activities recorded for each incident of ventricular activity. When there ventricular activity, the VA relationship was fixed. Notice that the His was present before each atrial activity, which suggests the location of the block is in the His–Purkinje system in a 2:1 pattern due to refractoriness. The first ventricular extrastimuli penetrated the Purkinje system but not the His. Therefore, it did not engage the circuit and did not change the cycle length of the tachycardia. The second ventricular extrastimuli was given earlier in the tachycardia cycle length and it penetrated also to a level below the His. Hence, the cycle length of activation at this vulnerable location of block has shortened. The His-Purkinje system’s refractoriness is directly related to the duration of proceeding cycle length1. Therefore, the site of block’s refractory period is also shortened. That leads to the penetration of the next atrial activity into the His-Purkinje system and setting a 1:1 AV conduction. Notice the fixed VA relationship before and after the ventricular extrastimulation. This linking suggests that the tachycardia diagnosis is most likely AV nodal reentry tachycardia.
A. Natale et al. (eds.), Cardiac Electrophysiology, DOI: 10.1007/978-1-84996-390-9_39, © Springer-Verlag London Limited 2011
159
160
M.E. Mortada et al.
Fig. 39.1 The patient’s 12-lead ECG upon admission to emergency room with symptoms of lightheadedness and palpitations
Fig. 39.2 The patient’s previous baseline 12-lead ECG
Case 39
161
Fig. 39.3 The patient spontaneously switched into the rhythm shown in this 12-lead ECG, accompanied by worsening symptoms, but she remained hemodynamically stable
Fig. 39.4 Intracardiac tracing of the same tachycardia the patient presented first to the ER. (Rhythm tracing from top to bottom: lead I, Lead II, Lead V1, high right atrium recording “HRA”, His bundle recording
“HIS”, and right ventricular apex recording “RVa”. Labels used in this figure are similar to the labels used in subsequent figures)
162
M.E. Mortada et al.
Fig. 39.5 Two ventricular extrastimuli (S1) at cycle lengths of 400 ms were administered during the tachycardia. The tachycardia accelerated from 2:1 AV conduction to 1:1 AV conduction
Reference 1. Akhtar M, Mahmud R, Tchou P, Denker S, Gilbert CJ. Normal electrophysiologic responses of the human heart. Cardiol Clin 1986;4: 365–386.
Case 40 Antonio Rossillo, Sakis Themistoclakis, Aldo Bonso, Andrea Corrado, and Antonio Raviele
Case Summary
Case Discussion
The patient is a 40-year-old female with a prior history of palpitations and is being medically managed for mild hypertension. The episodes became more frequent 2 months prior to the time of ablation. The ECG recordings were taken while the patient was having palpitations. The ECG revealed both orthodromic and antidromic tachycardia. Where is the likely position of the AP?
This bypass tract is likely located in the posterior septal space given the morphology of the maximally preexcited tracing (Figs. 40.1, 40.2) and it was successfully as shown in figs. 40.3–40.14.
A. Rossillo, S. Themistoclakis, A. Bonso, A. Corrado, and A. Raviele (*) Cardiovascular Department, Ospedale dell’Angelo, Mestre, Venice, Italy e-mail:
[email protected] A. Natale et al. (eds.), Cardiac Electrophysiology, DOI: 10.1007/978-1-84996-390-9_40, © Springer-Verlag London Limited 2011
163
164
A. Rossillo et al.
Fig. 40.1 The 12-lead resting ECG (paper speed 25 mm/s) showing normal sinus rhythm with a ventricular rate of 109 bpm with a short PR interval (118 ms) and a QRS width of 81 ms with mild delta wave in the initial portion of the QRS
Case 40
165
Fig. 40.2 12-lead ECG taken during atrial pacing (paper speed 25 mm/s) showing maximum ventricular preexitation with an early transition of the R wave between leads V1 and V2 and negative wave in the inferior leads
166
Fig. 40.3 Intracardiac recordings taken at baseline during the electrophysiology study (paper speed 200 mm/s). Four surface ECG leads are shown (I, aVF, V1, V6): (1) bipolar recording from the high right atrium (HRA), (2) bipolar recording from the distal His bundle region (HIS D)
A. Rossillo et al.
(3) five unipolar recordings from the coronary sinus (CS), (4) the unipolar (MC U-CATH) and the distal bipolar recording of the mapping catheter (MC D). (A atrium, V ventricle and H His bundle)
Case 40
167
Fig. 40.4 Intracardiac recordings taken during baseline while ventricular pacing (paper speed 200 mm/s). Same display as shown in Fig. 40.3 (A atrium, V ventricle)
168
A. Rossillo et al.
A Fig. 40.5 Intracardiac recordings taken during programmed atrial stimulation (paper speed 100 mm/s). Same display as shown in Fig. 40.3. Panel A: At a coupling interval of 290 ms the ventricular
preexitation was still present and in Panel B: With a coupling interval of 280 ms suddenly disappeared (ERP of bypass fiber). (A atrium, V ventricle and H His bundle)
Case 40
B Fig. 40.5 (continued)
169
170
A. Rossillo et al.
A Fig. 40.6 Intracardiac recordings taken during programmed ventricular stimulation (paper speed 200 mm/s). Same display as shown in Fig. 40.3. Panel A: At a coupling interval of 280 ms, the Stim-A interval
was 130 ms and suddenly increased and in Panel B to 180 ms with a coupling interval of 270 ms (retrograde ERP of a bypass fiber). (A atrium, V ventricle)
Case 40
B Fig. 40.6 (continued)
171
172
A. Rossillo et al.
Fig. 40.7 Intracardiac recordings taken during programmed atrial stimulation (paper speed 200 mm/s). Same display as shown in Fig. 40.3. AVRT induction (AA 280 ms) with a single extrastimulus (drive 500 ms and S1–S2 230 ms). (A atrium, V ventricle)
Case 40
Fig. 40.8 Intracardiac recordings (paper speed 200 mm/s). Same display as shown in Fig. 40.3. Ablation site. The bipolar recording of the ablation catheter shows presence of an atrial signal and of the bypass
173
fiber without ventricle activity that is present only in the unipolar recording. (A atrium, K Kent bundle, and V ventricle)
174
Fig. 40.9 Intracardiac recordings (paper speed 100 mm/s). Same display as shown in Fig. 40.3. Ablation site. The bipolar recording of the ablation catheter shows presence of an atrial signal and of the bypass
A. Rossillo et al.
fiber without ventricle activity that is present only in the unipolar recording (A atrium, K Kent bundle, and V ventricle)
Case 40
Fig. 40.10 Intracardiac recording taken during radiofrequency delivery (paper speed 200 mm/s). Same display as shown in Fig. 40.3. The second beat disappeared during ventricular preexitation and the intrac-
175
ardiac recording showed a clear separation between atrial (A) and ventricular signals (V). H His signal
176
Fig. 40.11 Intracardiac recordings taken during radiofrequency delivery (paper speed 25 mm/s). Same display as in Fig. 40.3. The arrow shows the elimination of the ventricular preexcitation. The circle shows
A. Rossillo et al.
the ventricle signal on the ablation catheter before and after the elimination of the bypass fibers
Case 40
177
A Fig. 40.12 Intracardiac recordings taken during programmed ventricular stimulation (paper speed 200 mm/s). Same display as shown in Fig. 40.3. Panel A: At a coupling interval of 280 ms, the Stim-A interval
was 160 ms and Panel B: At a coupling interval of 270 a decremental VA conduction was present (A atrium, V ventricle)
178
B Fig. 40.12 (continued)
A. Rossillo et al.
Case 40
179
Fig. 40.13 Intracardiac recording taken at the end of the procedure (paper speed 200 mm/s). The same display as shown in Fig. 40.3 (A atrium, V ventricle, H His bundle)
180
A. Rossillo et al.
Bibliography
A
RAO
Calkins H, Langberg J, Sousa J, et al. Radiofrequency catheter ablation of accessory atrioventricular connections in 250 patients. Abbreviated therapeutic approach to Wolff-Parkinson-White syndrome. Circulation. 1992;85:1337-1346. Haïssaguerre M, Dartigues JF, Warin JF, Le Metayer P, Montserrat P, Salamon R. Electrogram patterns predictive of successful catheter ablation of accessory pathways. Value of unipolar recording mode. Circulation. 1991;84:188-202. Jackman WM, Wang XZ, Friday KJ, et al. Catheter ablation of accessory atrioventricular pathways (Wolff-Parkinson-White syndrome) by radiofrequency current. N Engl J Med. 1991;324:1605-1611. Scheinman MM, Wang YS, Van Hare GF, Lesh MD. Electrocardio graphic and electrophysiologic characteristics of anterior, midseptal and right anterior free wall accessory pathways. J Am Coll Cardiol. 1992;20:1220-1229.
LAO
MV
H RA TT
CS MCV
B
TV
Fig. 40.14 Panel A: RAO and LAO of the mapping catheter at the ablation site in the proximal coronary sinus. Panel B: LAO picture of the mapping catheter at the ablation site in the proximal coronary sinus (RA right atrium, TT Todaro’s tendon, TV tricuspid valve, CS coronary sinus, MCV middle cardiac vein, H His bundle, MV mitral valve)
Case 41 Yash Y. Lokhandwala, Anoop K. Gupta, and Ranjan K. Thakur
Case Summary
Case Discussion
A 36-year-old woman with a normal heart had this ECG in the lab (Fig. 41.1). A narrow QRS tachycardia is seen without visible P waves; sharp J point in V1; this likely represents AVNRT. Multiple APDs change or accelerate tachycardia – how?
Tachycardias, pre- and post-APDs, are AVNRT. The APDs block in the first slow pathway and conduct by another slow pathway (Fig. 41.2). RF energy delivered in slow pathway region eliminated dual AVN physiology and inducibility.
Y.Y. Lokhandwala (*) KEM Hospital, Parel, Mumbai, India e-mail:
[email protected] A.K. Gupta Apollo Hospital, Ahmedabad, India e-mail:
[email protected] R.K. Thakur Thoracic and Cardiovascular Institute, Sparrow Health System, Michigan State University, 405 West Greenlawn, Suite 400, Lansing, MI 48910 e-mail:
[email protected] A. Natale et al. (eds.), Cardiac Electrophysiology, DOI: 10.1007/978-1-84996-390-9_41, © Springer-Verlag London Limited 2011
181
182
Y.Y. Lokhandwala et al. 2008083
I II III AVR AVL AVF V1 V2 V3 V4 V5 V6 10 mm/mV 25 mm/s
Fig. 41.1 SVT with change in cycle length
Case 41 2008083 I AVF V1 V6
CS78
CS34
CS12
HISD
HISP
rfD
rfP 10 mm/mV 50 mm/s
Fig. 41.2 SVT with change in antegrade limb resulting in faster cycle length
183
Case 42 Eric Buch, Shiro Nakahara, Marmar Vaseghi, Noel G. Boyle, and Kalyanam Shivkumar
Case Summary A 59-year-old male was referred for radiofrequency catheter ablation of atrial flutter and atrial fibrillation. He had a 3-year history of highly symptomatic paroxysmal atrial fibrillation that failed to respond to multiple medications. He underwent catheter-based radiofrequency ablation (RFA) with pulmonary vein isolation and subsequently a modified left- and right-sided surgical maze procedure after recurrence of atrial fibrillation. Following the maze procedure, patient developed incessant atrial flutter causing palpitations and fatigue. He was referred for catheter ablation. Diagnostic electrophysiology catheters were placed at the His bundle, the right ventricular apical septum, and in the coronary sinus. A multipolar halo catheter was placed in the right atrium with the distal electrodes in the coronary
sinus, middle electrodes along the lateral right atrial wall, and proximal electrodes near the roof of the right atrium. Intracardiac activation map showed atrial flutter with distal to proximal coronary sinus activation and passive counterclockwise activation of the right atrium (Fig. 42.1). After transseptal catheterization, electroanatomic mapping of the left atrium suggested that the tachycardia circuit utilized an isthmus between the mitral annulus and the left inferior pulmonary vein. Entrainment mapping confirmed that this left-sided isthmus was critical to the tachycardia circuit. During ablation in the region between the left inferior pulmonary vein and the mitral annulus (Fig. 42.2b), the intracardiac activation sequence changed abruptly, yet tachycardia persisted (Fig. 42.3). What is responsible for this change in sequence?
E. Buch (*), S. Nakahara, M. Vaseghi, N.G. Boyle, and K. Shivkumar UCLA Cardiac Arrhythmia Center, David Geffen School of Medicine at UCLA, 10833 Le Conte Ave, A2-237 CHS, Los Angeles, CA 90095-1679 e-mail:
[email protected] A. Natale et al. (eds.), Cardiac Electrophysiology, DOI: 10.1007/978-1-84996-390-9_42, © Springer-Verlag London Limited 2011
185
186 Fig. 42.1 Intracardiac recordings during tachycardia. Coronary sinus activation is from distal to proximal, consistent with a left atrial tachycardia circuit. Right atrial activation is earliest at the high septum (HALO 10) due to conduction through Bachman’s bundle, then conducts along the roof and down the lateral right atrial wall (HALO 9 to HALO 5). There is competing activation through the coronary sinus (HALO 1 to HALO 4), resulting in fusion of right atrial activation
E. Buch et al. I aVF V1 V6 ABL d HIS p HIS m HIS d HALO 10 HALO 9 HALO 8 HALO 7 HALO 6 HALO 5 HALO 4 HALO 3 HALO 2 HALO 1 CS 9, 10 CS 7, 8 CS 5, 6 CS 3, 4 CS 1, 2
Case Discussion Successful ablation of the mitral isthmus, which was critical for perpetuation of the left atrial macroreentrant tachycardia, resulted in termination of that rhythm. However, it unmasked an underlying typical counterclockwise right
atrial flutter with a longer tachycardia cycle length. In effect, the typical right atrial flutter had been entrained by the left atrial flutter. The patient then underwent further ablation at the cavotricuspid isthmus, resulting in termination of the atrial flutter, bidirectional isthmus block, and noninducibility of either tachycardia.
Case 42 Fig. 42.2 (a) Three-dimensional voltage map of the left atrium, left posterior oblique view. Large areas of scar are seen (in red), likely from prior catheter and surgical ablation procedures. (b) Posteroanterior view of same patient’s left atrium, showing ablation sites (red points) between left inferior pulmonary vein and mitral annulus. Pacing at these sites showed entrainment of the atrial flutter with postpacing interval near tachycardia cycle length
187
a
b
188 Fig. 42.3 Intracardiac recordings during ablation between left inferior pulmonary vein and mitral annulus. Tachycardia showed abrupt change in activation sequence and cycle length during ablation, with counterclockwise right atrial activation followed by medial to lateral coronary sinus activation. The last few beats are consistent with typical counterclockwise right atrial flutter
E. Buch et al. I aVF V1 V6 ABL d HIS p HIS m HIS d HALO 10 HALO 9 HALO 8 HALO 7 HALO 6 HALO 5 HALO 4 HALO 3 HALO 2 HALO 1 CS 9, 10 CS 7, 8 CS 5, 6 CS 3, 4 CS 1, 2
TCL 228 ms
TCL 280 ms
Case 43 Bradley P. Knight
Case Summary A patient with recurrent PSVT undergoes an electrophysiology procedure. A regular narrow QRS complex tachycardia is induced with a CL 360 ms and a 1:1 AV relationship with simultaneous atrial and ventricular activation. Figure 43.1 shows the response immediately after overdrive atrial pacing at a CL of 350 ms. Shown are the surface recordings I, II, V1, and V5 and intracardiac recordings from the high right atrium (HRA), at the His bundle region with the ablation catheter (Abl), and right ventricular apex (RV). A stimulation channel is also displayed. What is the diagnosis of the tachycardia?
Case Discussion During a regular supraventricular tachycardia with a 1:1 AV relationship, the differential diagnosis includes AV nodal reentry (AVNRT), orthodromic AV reentry (ORT), and atrial
tachycardia (AT). Atrial overdrive pacing can be used to help “rule in” or “rule out” atrial tachycardia as the mechanism.1,2 The principle is that during atrial overdrive pacing just slightly faster than the tachycardia, a 1:1 AV relationship can sometimes be maintained. In the case of an AT, the first atrial depolarization immediately after atrial pacing is stopped will occur independent of the last QRS complex that was a result of AV conduction during atrial pacing. Therefore the first VA interval would most likely differ from the VA interval during tachycardia. In contrast, in the case of an AV nodal– dependent tachycardia, such as AVNRT or ORT, the VA interval of the first return beat is usually the same as the VA interval during tachycardia because the atrial depolarization is dependent on VA conduction. However, there is always a small chance that the first VA interval will be the same or similar as that during tachycardia by chance alone. In this example, the AH interval is increasing during atrial pacing, but 1:1 AV conduction remains intact. The first VA interval after pacing is the same as that during tachycardia. Because the short VA interval excludes ORT as a mechanism,2 the diagnosis is most likely AVNRT.
B.P. Knight Division of Cardiology, Northwestern Medical Center, 676 N. St. Clair, Suite 600, Chicago, IL 60611 e-mail:
[email protected] A. Natale et al. (eds.), Cardiac Electrophysiology, DOI: 10.1007/978-1-84996-390-9_43, © Springer-Verlag London Limited 2011
189
190
B.P. Knight
Fig. 43.1 Example of atrial overdrive pacing during tachycardia to determine the mechanism of tachycardia. See text for format and abbreviations
References 1. Knight BP, Zivin A, Souza J, Flemming M, Pelosi F, Goyal R, Man KC, Strickberger SA, Morady F. A technique for the rapid diagnosis of atrial tachycardia in the electrophysiology laboratory. J Am Coll Cardiol 1999;33:775-81.
2. Knight BP, Ebinger M, Oral H, Kim MH, Sticherling C, Flemming M, Pelosi F, Michaud GF, Strickberger SA, Morady F. Diagnostic value of tachycardia features and pacing maneuvers during paroxysmal supraventricular tachycardia. J Am Coll Cardiol 2000;36:574-82.
Case 44 Antonio Rossillo, Sakis Themistoclakis, Aldo Bonso, Andrea Corrado, and Antonio Raviele
Case Summary The patient is a 76-year-old male with history of AVNRT since childhood and very frequent episodes occurring in the last 2 months. Ischemic heart disease with mild reduction of LVEF was present. Coronary angiography showed a moderate stenosis in the right and circumflex coronary and no significant stenosis of LAD (Fig. 44.1). An EP study was scheduled to define the arrhythmia (Fig. 44.2). What type of tachycardia is this? What ablative strategy would you choose?
Case Discussion The tachycardia is clearly an AVNRT and the presence of a first-degree AV block suggests the absence or pathological
anterograde conduction through the fast pathway. According to this finding, a standard ablative approach targeting the slow pathway may result in complete AV block. Pace mapping Koch’s triangle to define anterograde and retrograde structures of the AV node is useful in preventing this complication. In this case, pace mapping showed an absence of an anterograde fast pathway and the presence of a retrograde fast pathway in the anteroseptal region of the Koch’s triangle. Therefore, a single RF lesion was applied in the region of the fast pathway, and Fig. 44.3 shows a junctional rhythm during the ablation. The red arrow shows when the retrograde conduction skips from the fast to the slow pathway (A = atrium, V = ventricle). A transient complete AV Block (15s) was present at the end of RF energy delivery. The EP study performed at the end of the procedure showed a normal Wenckebach point and the tachycardia was no longer inducible.
A. Rossillo (*), S. Themistoclakis, A. Bonso, A. Corrado, and A. Raviele, Cardiovascular Department, Ospedale dell’Angelo, Mestre, Venice, Italy e-mail:
[email protected] A. Natale et al. (eds.), Cardiac Electrophysiology, DOI: 10.1007/978-1-84996-390-9_44, © Springer-Verlag London Limited 2011
191
192
A. Rossillo et al.
Fig. 44.1 12-lead resting ECG (paper speed 25 mm/s) showing sinus rhythm with first-degree AV block
Fig. 44.2 Intracardiac recordings taken during programmed atrial stimulation (paper speed 200 mm/s) with the induction of the tachycardia with a cycle length of 428 ms study (A atrium, V ventricle, and H His bundle)
Case 44
193
Fig. 44.3 A junctional rhythm during the ablation. The red arrow shows when the retrograde conduction skips from the fast to the slow pathway (A atrium, V ventricle)
Bibliography Delise P, Gianfranchi L, Paparella N, et al. Clinical usefulness of slow pathway ablation in patients with both paroxysmal atrioventricular nodal reentrant tachycardia and atrial fibrillation. Am J Cardiol. 1997;79:1421-1423. Delise P, Sitta N, Bonso A, et al. Pace mapping of Koch’s triangle reduces risk of atrioventricular block during ablation of atrioventricular nodal reentrant tachycardia. J Cardiovasc Electrophysiol. 2005;16:30-35. Haissaguerre M, Gaita F, Fischer B, et al. Elimination of atrioventricular nodal reentrant tachycardia using discrete slow potentials to
guide application of radiofrequency energy. Circulation. 1992;85: 2162-2175. Haïssaguerre M, Jaïs P, Shah DC, et al. Analysis of electrophysiological activity in Koch’s triangle relevant to ablation of the slow AV nodal pathway. Pacing Clin Electrophysiol. 1997;20:2470-2481. Jackman WM, Beckman KJ, McClelland JH, et al. Treatment of supraventricular tachycardia due to atrioventricular nodal reentry, by radiofrequency catheter ablation of slow-pathway conduction. N Engl J Med. 1992;327:313-318. Natale A, Greenfield RA, Geiger MJ, et al. Safety of slow pathway ablation in patients with long PR interval: further evidence of fast and slow pathway interaction. Pacing Clin Electrophysiol. 1997; 20:1698-1703.
Case 45 Yash Y. Lokhandwala, Anoop K. Gupta, and Ranjan K. Thakur
Case Summary
Case Discussion
A 25-year-old man with normal heart presents with SVT. His tachycardia is initiated with ventricular pacing (Figs. 45.1 and 45.2). The response to ventricular pacing is shown in Fig. 45.3. What is the diagnosis?
Tachycardia is instantiated by VPDs. VA interval is long during tachycardia and the atrial pattern during tachycardia is similar to that of the retrograde A waves (in the limited leads available). V pacing at a faster rate than the tachycardia shows VA dissociation suggesting that this is atrial tachycardia.
Y.Y. Lokhandwala (*) KEM Hospital, Parel, Mumbai, India e-mail:
[email protected] A.K. Gupta Apollo Hospital, Ahmedabad, India e-mail:
[email protected] R.K. Thakur Thoracic and Cardiovascular Institute, Sparrow Health System, Michigan State University, 405 West Greenlawn, Suite 400, Lansing, MI 48910, USA e-mail:
[email protected] A. Natale et al. (eds.), Cardiac Electrophysiology, DOI: 10.1007/978-1-84996-390-9_45, © Springer-Verlag London Limited 2011
195
196
Y.Y. Lokhandwala et al.
Ruby hall clinic 2008046 I AVF V1 V6
RAD
RAP
HISD
HISP
10 mm/mV 100 mm/s
Fig. 45.1 Initiation of SVT
Version WIN2000/XP : EPTRACER V0.77
Case 45 2008046
I
AVF
V1
V6
RAD
RAP
HISD
HISP
10 mm/mV 100 mm/s
Fig. 45.2 Activation sequence of the SVT
197
198
Y.Y. Lokhandwala et al. 2008046
I AVF V1 V6
RAD
RAP
HISD
HISP
10 mm/mV 100 mm/s
Fig. 45.3 Ventricular pacing during SVT
Case 46 Eric Buch, Shiro Nakahara, Marmar Vaseghi, Noel G. Boyle, and Kalyanam Shivkumar
Case Summary A 79-year-old male with a history of hypertension, coronary artery bypass grafting, and implantable cardioverterdefibrillator for ventricular fibrillation, presented with symptomatic supraventricular tachycardia. He had failed antiarrhythmic medications and cardioversion attempts, and was referred for electrophysiology study and catheter ablation. Twelve-lead electrocardiogram showed organized
atrial activity with positive flutter waves in lead V1 and inferior leads, not suggestive of typical counterclockwise atrial flutter (Fig. 46.1). Diagnostic electrophysiology catheters were placed at the His bundle, the right ventricular apical septum, and in the coronary sinus. Intracardiac activation map obtained during tachycardia showed counterclockwise activation of the right atrium. Ablation of the cavotricuspid isthmus was performed without tachycardia termination. Why did the tachycardia persist?
Fig. 46.1 Surface electrocardiogram of clinical tachycardia. There is an organized atrial rhythm with low-amplitude flutter waves that are predominantly positive in lead V1. Variable conduction to the ventricles results in an irregular ventricular rhythm
E. Buch (*), S. Nakahara, M. Vaseghi, N.G. Boyle, and K. Shivkumar UCLA Cardiac Arrhythmia Center, David Geffen School of Medicine at UCLA, 10833 Le Conte Ave, A2-237 CHS, Los Angeles, CA 90095-1679 e-mail:
[email protected] A. Natale et al. (eds.), Cardiac Electrophysiology, DOI: 10.1007/978-1-84996-390-9_46, © Springer-Verlag London Limited 2011
199
200
E. Buch et al.
Case Discussion Entrainment mapping from the septal side of the cavotricuspid isthmus and the distal coronary sinus were performed. Although the tachycardia could be entrained from both sites, the post-pacing interval was much shorter from the distal coronary sinus, suggesting a left atrial flutter circuit with passive counterclockwise activation of the right atrium. After transseptal catheterization, electroanatomic mapping of the left atrium was performed. Voltage mapping revealed an area of scar on the left atrial roof. Activation a
map shown in Fig. 46.2a demonstrates a left atrial tachycardia with centrifugal activation of the left atrium originating near a scar on the left atrial roof. Several viable sites were found adjacent to the scar, with early activation (Fig. 46.2b). Pacing from these locations resulted in entrainment with concealed fusion, and post-pacing interval near tachycardia cycle length. During ablation (Fig. 46.3) the tachycardia spontaneously terminated, and could not be reinduced. This case illustrates that typical counterclockwise activation of the right atrium does not prove that it is part of the tachycardia circuit. b
Fig. 46.2 (a) Three-dimensional electroanatomical activation map of the left atrium during tachycardia, anteroposterior view. Earliest activation of the left atrium is at the roof (red area), just lateral to a region of scar (gray
area) near the superior aspect of the left atrial septum. (b) Right posterior oblique view. Radiofrequency energy was applied at viable sites adjacent to the scar (red points within red area) with low-amplitude potentials
ISI
200 ms
aVF V1 V5 ABL d ABL p HIS p
C
HIS m
C
HIS d
C
CS 9, 10
Fig. 46.3 During ablation at one of the sites shown in Fig.46.2b, the tachycardia terminated and could not be reinduced. Note the resumption of atrioventricular paced rhythm at the right side of the tracing
CS 7, 8 CS 5, 6 CS 3, 4 CS 1, 2
11:34:13 AM
11:34:14 AM
Case 47 Bradley P. Knight
Case Summary A patient with recurrent PSVT undergoes an electrophysiology procedure. Fig. 47.1 shows a recording during the procedure. Shown are the surface recordings and intracardiac recordings from the His bundle electrogram (HBE) ablation catheter (Abl), coronary sinus (CS), and right ventricular apex (RV). What is the diagnosis of the tachycardia?
Case Discussion The differential diagnosis of a regular narrow QRS complex tachycardia in the EP lab includes AV nodal reentry, orthodromic AV reentry, and atrial tachycardia. The transition zones during tachycardia can be very useful when trying to make a diagnosis. In this case the tachycardia terminates. Before termination, one can see that there is simultaneous
activation of the atrium and ventricle with a septal VA interval that is close to zero. This excludes AV reentry using an accessory pathway. When attempting to measure the septal VA interval, it can be difficult at times to differentiate the atrial signal from the ventricular signal on the His bundle recording. In this example, the proximal coronary sinus recordings (CS9-10) can be used to identify the timing of septal atrial activation. The termination of the tachycardia is associated with spontaneous AV block. This observation excludes an AT which would be expected to conduct to the ventricle upon termination if there is a 1:1 conduction during the tachycardia. In this case the tachycardia actually slows slightly before termination. Therefore, it would be extremely unlikely that an atrial tachycardia that spontaneously slows just before termination would be associated with AV block on the last beat. Another observation that excludes an atrial tachycardia is that as the tachycardia cycle length increases the VA interval remains constant. This fixed relationship suggests an AV nodal–dependent tachycardia.
B.P. Knight Division of Cardiology, Northwestern Medical Center, 676 N. St. Clair, Suite 600, Chicago, IL, 60611 e-mail:
[email protected] A. Natale et al. (eds.), Cardiac Electrophysiology, DOI: 10.1007/978-1-84996-390-9_47, © Springer-Verlag London Limited 2011
201
202
B.P. Knight I
320
330
340
400
780
430
680
390
780
450
II III V1 V5 HBE d HBE p Map d Map p CS 1-2 CS 3-4 CS 5-6 CS 7-8 CS 9-10 RV d Stim
Fig. 47.1 Tracing recorded during an electrophysiology procedure in a patient with recurrent paroxysmal supraventricular tachycardia. Spontaneous termination of tachycardia occurs. See text for format and abbreviations
Case 48 Antonio Rossillo, Sakis Themistoclakis, Aldo Bonso, Andrea Corrado, and Antonio Raviele
Case Summary A 47-year-old male patient without any structural heart disease presented with a history of palpitations since childhood. All ECGs recorded while the patient experienced palpitations showed a wide QRS complex tachycardia with a left bundle branch block pattern. The arrhythmia was always
sustained and external DC shock was necessary on three occasions to restore sinus rhythm. Pharmacologic treatment (including amiodarone and propafenone) was unsuccessful and the two last episodes of tachycardia were reported to be faster than usual and poorly tolerated (Figs. 48.1–48.8). Based on the previous tracings, what is the tachycardia mechanism?
Fig. 48.1 12-lead resting ECG (paper speed 25 mm/s) showing sinus rhythm, a normal PR interval (170 ms), and a QRS width of 100 ms with slight slurring of the initial portion of the QRS
A. Rossillo (*), S. Themistoclakis, A. Bonso, A. Corrado, and A. Raviele Cardiovascular Department, Ospedale dell’Angelo, Mestre, Venice, Italy A. Natale et al. (eds.), Cardiac Electrophysiology, DOI: 10.1007/978-1-84996-390-9_48, © Springer-Verlag London Limited 2011
203
204
Fig. 48.2 Intracardiac recordings at baseline during the electrophysiology study (paper speed 100 mm/s). Four surface ECG leads (I, aVF, V1, V6), two bipolar recordings from the His bundle region (HIS D for distal and HIS P for proximal), seven bipolar recordings from the epi-
A. Rossillo et al.
cardial side of the tricuspid annulus obtained by a Cardima catheter positioned in the right coronary artery (C1–C7), and one bipolar recording from the coronary sinus (SC). AH interval is 80 ms and HV interval 30 ms
Case 48
Fig. 48.3 Intracardiac recordings taken during programmed atrial stimulation (paper speed 200 mm/s). Same display as shown in Fig. 48.2. During programmed atrial stimulation (coupling interval between S1 and S2: 375 ms) A-H interval increased from 105 to 125 ms,
205
H-V interval decreased from 30 to 10 ms, and the QRS complex on the surface ECG became wider with a left bundle branch block morphology
206
A. Rossillo et al.
Fig. 48.4 Intracardiac recordings taken during programmed atrial stimulation (paper speed 200 mm/s). Same display as shown in Fig. 48.2. At the coupling interval of 300 ms, atrioventricular node duality was demonstrated with an A-H interval which increased suddenly from 111 to 192 ms
Case 48
Fig. 48.5 Intracardiac recordings taken during atrial stimulation (cycle length 410 ms; paper speed 100 mm/s). Same display as shown in Fig. 48.2. Ventricular preexcitation is present with a negative (−10 ms)
207
H-V interval, a His bundle potential identified at the very beginning of the QRS complex, and a left bundle branch block morphology on the surface ECG
208
A. Rossillo et al.
Fig. 48.6 Intracardiac recordings during atrial stimulation (paper speed 100 mm/s). Same display as that shown in Fig. 48.2. When atrial pacing at a cycle length of 270 ms is abruptly interrupted, two atrioventricular node echoes appeared (slow–fast type)
Case 48
Fig. 48.7 Intracardiac recordings taken during the tachycardia induced during programmed atrial stimulation (paper speed 100 mm/s). Same display as Fig. 48.2. The tachycardia had a cycle length of 330 ms and
209
a left bundle branch block morphology. Intracardiac recordings showed that the site of earliest atrial activation occurred in the His bundle region with a retrograde His deflection
210
A. Rossillo et al.
Fig. 48.8 12-lead ECG taken during the induced tachycardia (rate: 186 bpm; paper speed 25 mm/s) showing left bundle branch block morphology. In fact, the induced tachycardia on 12-lead ECG is identical to the one observed during rapid atrial stimulation (see Fig. 48.6)
Case Discussion The patient has an atriofascicular pathway or so-called Manheim. Based on Fig. 48.9 this is likely antidromic tachycardia with retrograde conduction over either the fast or slow pathway (Figs. 48.10–48.13).
Case 48
Fig. 48.9 Intracardiac recordings taken during the tachycardia (paper 200 mm/s). Same display as shown in Fig. 48.2. A spontaneous change in H-A interval was observed (from 120 to 69 ms) without any change in atrial retrograde activation pattern but with shortening of the tachy-
211
cardia cycle length (from 670 to 570 ms). This figure illustrates an abrupt switch from the slow to the fast retrograde pathway excluding the slow–fast atrioventricular node reentrant tachycardia as the mechanism of the induced tachycardia
212
Fig. 48.10 Intracardiac recordings taken during mapping of the tricuspid annulus during sinus rhythm. Same representation as shown in Fig. 48.2, except that a bipolar recording obtained from the tip of the
A. Rossillo et al.
ablation catheter (MC BIP) is now displayed. In the lateral region of the tricuspid annulus a specific potential was recorded (M potential)
Case 48
Fig. 48.11 Intracardiac recordings taken during mapping of the tricuspid annulus during sinus rhythm. Same representation as shown in Fig. 48.2. When mapping the region where the M potential was
213
recorded, mechanical block occurred (normalization of the QRS complex on the surface ECG, normalization of H-V interval) (arrow). Ablation was performed at this site
214
A. Rossillo et al.
Fig. 48.12 12-lead ECG recorded after the ablation procedure (paper speed 25 mm/s). Radiofrequency current was applied on the site shown in Fig. 48.12. The 12-lead ECG is now normal without any ventricular preexcitation
Case 48
215
Bibliography Ao
Cdx H
VM
M VT
SC
Cdx
Fig. 48.13 Picture showing the circuit of the arrhythmia (VT = tricuspid valve, MV = mitral valve, Ao = Aorta, Cdx = right coronary artery, H = His bundle, and M = mahaim fibers)
Aliot E, de Chillou C, Revault d’Allones G. Mahaim tachycardias. Eur Heart J. 1998;19(suppl E):E25-E31. Ellenbogen KA, Ramirez NM, Packer DL, et al. Accessory nodoventricular (Mahaim) fibers: a clinical review. PACE. 1986;9:868-875. Gallagher JJ, Smith WM, Kasell JH, et al. Role of Mahaim fibers in cardiac arrhythmias in man. Circulation. 1981;64:176-184. Haissaguerre M, Cauchemez B, Marcus F, et al. Characteristics of the ventricular insertion sites of accessory pathways with anterograde decremental conduction properties. Circulation. 1995;91:10771085. Josephson ME. Nodoventricular and fasciculoventricular bypass tracts. In: Josephson ME, ed. Clinical Cardiac Electrophysiology: Techniques and Interpretation. 2nd ed. Malvern: Lea & Febiger; 1993:396-416. Mounsey JP, Griffith MJ, McComb JM, et al. Radiofrequency ablation of Mahaim fiber following localization of Mahaim pathway potentials. J Cardiovasc Electrophysiol. 1994;5:432-434. Tchou P, Lehmann MH, Jazayeri M, et al. Atriofascicular connections or a nodoventricular Mahaim fiber? Electrophysiologic elucidation of the pathway and associated reentrant circuit. Circulation. 1988;77:837-841.
Case 49 Eric Buch, Shiro Nakahara, Marmar Vaseghi, Noel G. Boyle, and Kalyanam Shivkumar
Case Summary
Case Discussion
A 30-year-old female with primary pulmonary hypertension had recurrent atrial flutter despite amiodarone treatment and multiple cardioversions. Because of worsened dyspnea resulting from atrial flutter, she was referred for electrophysiology study and catheter ablation. Preprocedural surface electrocardiograms, both recorded during the month prior to ablation, showed organized atrial activity with variable flutter wave morphology (Fig. 49.1). Based on the 12-lead ECG, what is the diagnosis? Do these arrhythmias arise from multiple flutter circuits?
By definition, typical atrial flutter is a macroreentrant tachycardia bound anteriorly by the tricuspid isthmus and posteriorly by the inferior vena cava, the terminal crest, and most commonly, the superior vena cava.1 The majority of these tachycardias are counterclockwise (CCW), with activation descending the anterolateral and ascending the septal RA. Less commonly, isthmus-dependent atrial flutter is clockwise (CW), with activation descending the septal and ascending the anterolateral RA. Multiple studies have attempted to characterize the surface ECG appearance of
Fig. 49.1 Surface electrocardiogram showing flutter waves that are predominantly negative in lead V1 and biphasic in the inferior leads, not typical of cavotricuspid isthmus dependent counterclockwise atrial flutter
E. Buch (*), S. Nakahara, M. Vaseghi, N.G. Boyle, and K. Shivkumar UCLA Cardiac Arrhythmia Center, David Geffen School of Medicine at UCLA, 10833 Le Conte Ave, A2-237 CHS, Los Angeles, CA 90095-1679 e-mail:
[email protected] A. Natale et al. (eds.), Cardiac Electrophysiology, DOI: 10.1007/978-1-84996-390-9_49, © Springer-Verlag London Limited 2011
217
218
CW and CCW flutter.2 It is currently accepted that predominantly negative F wave deflections in the inferior leads and in lead V6 and positive deflection in V1 indicate CCW flutter, while CW flutter has positive F waves in the inferior leads and V6, and negative F waves in lead V1.1,2 In this
E. Buch et al.
case, negative F wave deflection in V1 and positive deflection in inferior leads (Fig. 49.1a) predicted CW flutter, which was established by intracardiac mapping (Fig. 49.2). The other surface ECG (Fig. 49.1b) was likely CCW flutter. Neither arrhythmia was inducible after ablation at the
a
b
Fig. 49.2 (a) Left anterior oblique fluoroscopic view of intracardiac electrophysiology catheters. The duopecapolar catheter (D1–D10) is curled in the right atrium, with the proximal bipoles (D9, D10) near the right atrial roof, the middle bipoles (D4–D8) along the lateral right atrial wall, and the distal bipoles (D2, D3) near the cavotricuspid
isthmus, with the most distal bipole (D1) in the proximal coronary sinus. (b) Surface and intracardiac recordings. The activation sequence proceeds from distal to proximal, consistent with clockwise isthmusdependent atrial flutter
Case 49
219
I aVF V1 V6 aVR ABL d ABL p D10 D9 D8 D7 D6 D5 D4 D3 D2 D1
200 ms
11:15:07 AM
11:15:08 AM
Fig. 49.3 With the ablation catheter positioned as shown in Fig. 49.2 above, radiofrequency energy was applied. Intracardiac recordings during ablation, showing termination of tachycardia
cavotricuspid isthmus (Fig. 49.3). Of note, F wave morphology not adhering to the above criteria has been described in a minority of patients when the left atrium is activated by Bachmann’s bundle. In this case, CCW flutter can manifest as positive F waves in the inferior leads, and clockwise flutter can manifest as negative F waves in the inferior leads. (Figs 49.2 and 49.3).3
References 1. Saoudi N, Cosio F, Waldo A, et al. Classification of atrial flutter and regular atrial tachycardia according to electrophysiologic mechanism and anatomic bases: a statement from a joint expert group
from the Working Group of Arrhythmias of the European Society of Cardiology and the North American Society of Pacing and Electrophysiology. J Cardiovasc Electrophysiol. 2001;12: 852-866. 2. Milliez P, Richardson AW, Obioha-Ngwu O, Zimetbaum PJ, Papageorgiou P, Josephson ME. Variable electrocardiographic characteristics of isthmus-dependent atrial flutter. J Am Coll Cardiol. 2002;40:1125-1132. 3. Oshikawa N, Watanabe I, Masaki R, et al. Relationship between polarity of the flutter wave in the surface ECG and endocardial atrial activation sequence in patients with typical counterclockwise and clockwise atrial flutter. J Interv Card Electrophysiol. 2002; 7:215-223.
Case 50 Bradley P. Knight
Case Summary
Left-sided slow pathway ablation
An attempt was made to ablate the slow pathway in a 45-year-old woman with recurrent SVT and reproducibly inducible typical AV nodal reentry. Using a standard 4-mmtip electrode ablation catheter, a total of 55 applications of RF current were delivered using 50 W at several sites just outside and slightly within the coronary sinus ostium. Despite adequate tissue heating and the occurrence of an accelerated junctional rhythm with most lesions, the tachycardia remained inducible. What other approach could be used?
RF site
Case Discussion A very small number of patients who have what appear to be the slow–fast type of AV nodal reentry cannot be successfully ablated using a right-sided approach. It is likely that there are left-sided inputs to the AV node that require ablation to eliminate slow pathway function. Using a retrograde aortic approach the left posterior septal aspect of the mitral annulus was explored (Fig. 50.1). A site was identified that was associated with a complex, fractionated atrial electrogram (Fig. 50.2). A single application of RF current at this site resulted in an accelerated junctional rhythm and elimination of slow pathway function (Fig. 50.3).
Fig. 50.1 A fluoroscopic image in a left anterior oblique projection during ablation of a slow pathway using a left-sided approach. Three catheters can be seen in the right heart including a right ventricular, His-bundle, and coronary sinus catheter. There is also a left sided ablation catheter that has been positioned along the posteroseptal mitral annulus using a retrograde aortic approach
B.P. Knight Division of Cardiology, Northwestern Medical Center, 676 N. St. Clair, Suite 600, Chicago, IL 60611, USA e-mail:
[email protected] A. Natale et al. (eds.), Cardiac Electrophysiology, DOI: 10.1007/978-1-84996-390-9_50, © Springer-Verlag London Limited 2011
221
222
Fig. 50.2 Tracing showing the electrogram at the successful ablation site during sinus rhythm in a patient undergoing a left-sided slow AV nodal pathway ablation. Shown are surface recordings from leads I, II, V1, and V5, and the intracardiac recordings from high right atrium (HRA), His-bundle electrogram region (HBE), left ventricle (LV), and right ventricular apex (RVA)
Bibliography Katritsis DG, Becker AE, Ellenbogen KA, et al. Right and left inferior extensions of the atrioventricular node may represent the anatomic substrate of the slow pathway in humans. Heart Rhythm. 2004;1:582-586.
B.P. Knight
Fig. 50.3 Tracing demonstrating an accelerated junctional rhythm during delivery of radiofrequency current at the successful ablation site in a patient undergoing a left-sided slow AV nodal pathway ablation. The format and abbreviations are the same as in Figure 50.2
Case 51 Yash Y. Lokhandwala, Anoop K. Gupta, and Ranjan K. Thakur
Case Summary A 54-year-old woman with a normal heart presents to the EP lab with an SVT (Fig. 51.1). The tachycardia could be induced with three APCs or with ventricular premature beats (Figs. 51.2 and 51.3). Explain the mechanism of the tachycardia?
Tachycardia is initiated by a decrementally conducting PVC. VA time is long; earliest in proximal CS. With PVC placed when the His was refractory, no atrial preexcitation was observed, suggesting that the likely mechanism was atypical AVNRT. There is slow–intermediate and slow–slow AVNRT, hence ablation of the slow pathway was performed; no tachycardia was observed after this. The VA time at initiation of even typical AVNRT often shortens over the initial few complexes, more so in older patients; hence an apparently atypical AVNRT may later appear typical (Fig. 51.4).
Case Discussion With application of three atrial extrastimuli, tachycardia is induced. Induction is preceded by lengthening of the AH interval; this suggests that the likely mechanism is AVNRT.
Y.Y. Lokhandwala (*) KEM Hospital, Parel, Mumbai, India e-mail:
[email protected] A.K. Gupta Apollo Hospital, Ahmedabad, India e-mail:
[email protected] R.K. Thakur Thoracic and Cardiovascular Institute, Sparrow Health System, Michigan State University, 405 West Greenlawn, Suite 400, Lansing, MI 48910, USA e-mail:
[email protected] A. Natale et al. (eds.), Cardiac Electrophysiology, DOI: 10.1007/978-1-84996-390-9_51, © Springer-Verlag London Limited 2011
223
224
Y.Y. Lokhandwala et al.
RUBY HALL CLINIC 2008045 I II III AVR AVL AVF V1 V2 V3 V4 V5 V6 10 mm/mV 25 mm/s
Fig. 51.1 SVT
Version WIN2000/XP : EPTRACER V0.77
Case 51
225 2008045
I AVF V1 V6 CS910 CS78 CS56 CS34 CS12 HISD HISP RFD RFP 10 mm/mV 100 mm/s
Fig. 51.2 Three PACs initiate tachycardia
226
Y.Y. Lokhandwala et al. 2008045
I AVF V1 V6
CS34
CS12
HISD
HISP
RFD
10 mm/mV 100 mm/s
Fig. 51.3 Tachycardia initiation with ventricular premature beat
Case 51
227 2008045
I AVF V1 V6 CS910 CS78 CS56 CS34 CS12 HISD HISP RFD
Fig. 51.4 The patient later showed this pattern – typical AVNRT
Case 52 Eric Buch, Shiro Nakahara, Marmar Vaseghi, Noel G. Boyle, and Kalyanam Shivkumar
Case Summary A 79-year-old male with a history of hypertension, diabetes, coronary artery bypass grafting, recently diagnosed with atrial flutter and referred for electrophysiology study and catheter ablation. Initial 12-lead electrocardiogram (Fig. 52.1) showed organized atrial activity with a cycle length of
approximately 220 ms with negative F waves in leads II, III, and aVF, positive F waves in V1, suggesting typical counterclockwise atrial flutter. Conduction to ventricles was predominantly 4:1. Electroanatomic mapping suggested macroreentrant arrhythmia in the right atrium. As seen in Fig. 52.2, the direction of activation was counterclockwise, and the circuit
Fig. 52.1 Surface electrocardiogram of clinical tachycardia. Atrial flutter is seen, with F waves that are predominantly negative in the inferior leads and upright in V1, consistent with typical counterclockwise atrial flutter
E. Buch (*), S. Nakahara, M. Vaseghi, N.G. Boyle, and K. Shivkumar UCLA Cardiac Arrhythmia Center, David Geffen School of Medicine at UCLA, 10833 Le Conte Ave, A2-237 CHS, Los Angeles, CA 90095-1679 e-mail:
[email protected] A. Natale et al. (eds.), Cardiac Electrophysiology, DOI: 10.1007/978-1-84996-390-9_52, © Springer-Verlag London Limited 2011
229
230
E. Buch et al.
Fig. 52.2 Three-dimensional electroanatomic mapping during tachycardia. Anteroposterior view of right atrium shows the reentrant circuit, with early meeting late activation in a circuit around the tricuspid annulus. Pacing from the tip of the ablation catheter, which was positioned at the cavotricuspid isthmus, showed entrainment with concealed fusion and a postpacing interval of 10 ms longer than tachycardia cycle length
involved the cavotricuspid isthmus. What maneuvers are useful in proving that the cavotricuspid isthmus is part of the tachycardia?
Case Discussion One maneuver that can show whether a given location is close to the arrhythmia circuit is entrainment mapping.1 This is accomplished by pacing slightly faster than the tachycardia cycle length to repeatedly reset the reentrant circuit. If paced P wave morphology is identical to that observed during atrial flutter, this is termed entrainment with concealed fusion. If tachycardia continues after cessation of pacing, the first postpacing interval will be close to the tachycardia cycle length, provided that the pacing site is near the reentrant circuit.2 Postpacing interval is defined as the time from last paced stimulus to the next local electrogram during tachycardia, measured at the site of pacing.
In this case, pacing from the cavotricuspid isthmus showed entrainment with concealed fusion, with identical surface flutter wave morphology and intracardiac activation pattern as compared to tachycardia. The postpacing interval was within 10 ms of the flutter cycle length. This showed that the cavotricuspid isthmus was likely part of the tachycardia circuit, and therefore radiofrequency ablation of the isthmus was performed. The ablation catheter was advanced into the right ventricle, curved, and pulled inferiorly to seat it on the cavotricuspid isthmus, with the catheter tip at 6 o’clock in the left anterior oblique view. A series of radiofrequency applications were delivered to create a line of conduction block across the cavotricuspid isthmus. During ablation, the cycle length suddenly prolonged and the arrhythmia terminated with a long pause (Fig. 52.3). Normal sinus node function and atrioventricular conduction resumed after a few seconds, and atrial flutter was no longer inducible at the end of the ablation procedure.
Case 52
231
Fig. 52.3 Three-dimensional electroanatomic mapping during radiofrequency ablation, caudal left anterior oblique view. As the line of conduction block across the cavotricuspid isthmus was completed, the flutter terminated spontaneously and could not be reinduced
References 1. Cosio FG, Lopez Gil M, Arribas F, Palacios J, Goicolea A, Nunez A. Mechanisms of entrainment of human common flutter studied with multiple endocardial recordings. Circulation. 1994;89:2117-2125.
2. Stevenson WG, Khan H, Sager P, et al. Identification of reentry circuit sites during catheter mapping and radiofrequency ablation of ventricular tachycardia late after myocardial infarction. Circulation. 1993;88:1647-1670.
Case 53 Bradley P. Knight
Case Summary A patient is scheduled to undergo a catheter ablation procedure for recurrent PSVT, which appears most likely to be typical AV nodal reentry. You discover that the patient underwent insertion of an inferior vena cava filter 2 years ago after a pulmonary embolism. How would you proceed with this case?
Case Discussion When performing a slow pathway ablation procedure, it is important to monitor VA conduction during the accelerated junctional rhythm to avoid AV block. Therefore, it is usually necessary to place two catheters in the heart during the procedure. Unfortunately, some patients represent vascular access challenges. In this case, the patient has a filter in the
inferior vena cava. Although there are some risks associated with crossing the filter with a catheter, it is clear that an electrophysiology procedure can be performed safely in these patients from the femoral vein.1 It is helpful to be certain that the vena cava is patent before the procedure with a CT scan or venography. But assuming that the IVC is patent, long guiding sheaths and electrophysiology catheters can be passed safely through the filter. Figure 53.1 shows a long sheath already through the filter, and a second one being advanced over a guidewire that has been passed through the filter. There are times, however, when the vena cava is completely obstructed. In this situation, options include a superior or transhepatic central venous approach. One reasonable approach that has been shown to be successful in patients with AV nodal reentry is to place two central venous sheaths in the right internal jugular vein.2 This allows placement of a diagnostic catheter in the high right atrium and an ablation catheter along the tricuspid annulus (Fig. 53.2).
B.P. Knight Division of Cardiology, Northwestern Medical Center, 676 N. St. Clair, Suite 600, Chicago, IL 60611 e-mail:
[email protected] A. Natale et al. (eds.), Cardiac Electrophysiology, DOI: 10.1007/978-1-84996-390-9_53, © Springer-Verlag London Limited 2011
233
234
Fig. 53.1 Fluoroscopic image during catheter placement from a femoral vein approach through an inferior vena cava filter. Two sheaths were advanced over a guidewire to access the heart with two electrophysiology catheters. See text for details
References 1. Sinha SK, Harnick D, Gomes JA, Mehta D. Electrophysiologic interventions in patients with inferior vena cava filters: safety and efficacy of the transfemoral approach. Heart Rhythm. 2005;2:15-18.
B.P. Knight
Fig. 53.2 Right anterior oblique fluoroscopic view after placement of an ablation catheter and diagnostic quadrapolar catheter via the internal jugular vein, during a catheter ablation procedure for typical atrioventricular nodal reentry in a patient without inferior vena cava access 2. Salem Y, Burke MC, Morady F, Knight BP. Slow pathway ablation for atrioventricular nodal reentry using a right internal jugular vein approach: a case series. Pacing Card Electrophysiol. 2006; 29:59-62.
Case 54 Yash Y. Lokhandwala, Anoop K. Gupta, and Ranjan K. Thakur
Case Summary An 18-year-old young man with normal heart presents to the EP lab. His initial tracing shows AF followed by an SVT (Fig. 54.1). Figures 54.2 and 54.3 show the response to ventricular premature beats and ventricular pacing. Does this help elucidate the cause of the tachycardia?
Case Discussion Even a very early PVC from the right ventricle does not advance the “A.” This makes AVRT extremely unlikely because (1) left-sided AP is ruled out by the activation
sequence and (2) a right-sided AP should have caused atrial preexcitation. This PVC does not rule out AVNRT because to do so, the PVC must be delivered when the His is refractory. Thus, the differential diagnoses at this point include AVNRT and atrial tachycardia. Ventricular pacing is performed during tachycardia. VA dissociation is seen with continuation of tachycardia, proving a diagnosis of atrial tachycardia. The tachycardia itself does not lend itself to analysis of the P waves. The tachycardia continues in spite of bursts of PVCs, suggesting that this is atrial tachycardia. P wave morphology is revealed after the bursts; upright in I, II, III and aVF, suggesting high right atrial origin (Fig. 54.4)
Y.Y. Lokhandwala (*) KEM Hospital, Parel, Mumbai, India e-mail:
[email protected] A.K. Gupta Apollo Hospital, Ahmedabad, India e-mail:
[email protected] R.K. Thakur Thoracic and Cardiovascular Institute, Sparrow Health System, Michigan State University, 405 West Greenlawn, Suite 400, Lansing, MI 48910, USA e-mail:
[email protected] A. Natale et al. (eds.), Cardiac Electrophysiology, DOI: 10.1007/978-1-84996-390-9_54, © Springer-Verlag London Limited 2011
235
236
Y.Y. Lokhandwala et al. 2008043
I AVF V1 V6 CS910 CS56 CS12 HISD HISP
Fig. 54.1 The left half shows AF which initiates a narrow complex SVT. AF termination is followed by a junctional beat and then, SVT. SVT starts with an “A”
Case 54
237 2008043
I AVF V1 V6 CS910 CS56 CS12 HISD HISP RFD RFP 10 mm/mV 300 mm/s
Fig. 54.2 A VPC is delivered during the tachycardia
238
Y.Y. Lokhandwala et al. 2008043
I AVF V1 V6 CS910 CS56 CS12 HISD HISP RFD RFP 10 mm/mV 150 mm/s
Fig. 54.3 V pacing during the tachycardia
Case 54
239 2008043
I AVF V1 V6 CS910 CS56 CS12 HISD HISP RFD RFP 10 mm/mV 150 mm/s
Fig. 54.4 P wave morphology can often be uncovered after ventricular pacing
Case 55 Eric Buch, Shiro Nakahara, Marmar Vaseghi, Noel G. Boyle, and Kalyanam Shivkumar
Case Summary A 52-year-old male with history of hypertension, diabetes, and coronary artery disease status post coronary artery bypass grafting was recently seen for palpitations and dyspnea. Surface
electrocardiogram showed a regular supraventricular tachycardia at a rate of approximately 250 per minute, with 2:1 conduction to the ventricles (Fig. 55.1). He was referred for electrophysiology study and catheter ablation. Based on this ECG tracing, what is the most likely mechanism of this tachycardia?
Fig. 55.1 Surface electrocardiogram of the clinical tachycardia. Sawtooth flutter waves are predominantly negative in the inferior leads, upright in V1, and inverted in V6
E. Buch (*), S. Nakahara, M. Vaseghi, N.G. Boyle, and K. Shivkumar UCLA Cardiac Arrhythmia Center, David Geffen School of Medicine at UCLA, 10833 Le Conte Ave, A2-237 CHS, Los Angeles, CA 90095-1679 e-mail:
[email protected] A. Natale et al. (eds.), Cardiac Electrophysiology, DOI: 10.1007/978-1-84996-390-9_55, © Springer-Verlag London Limited 2011
241
242
E. Buch et al.
a
b I aVF V1 V6 HRA HIS p HIS m HIS d CS 9,10 CS 7,8 CS 5,6 CS 3,4 CS 1,2 DD 19,20 DD 17,18 DD 15,16 DD 13,14 DD 11,12 DD 9,10 DD 7,8 DD 5,6 DD 3,4 DD 1,2 RVA
100 ms
T C C C
Fig. 55.2 (a) Fluoroscopic left anterior oblique view showing catheter positions. The duodecapolar (DD) catheter is curled in the right atrium such that the proximal, mid, and distal bipoles of the catheter are near the right atrial roof, anterolateral wall, and coronary sinus os, respectively. Arrows show the direction of activation. (b) Intracardiac tracings.
3:15:00 PM
Right atrial activation is seen to proceed from septal to lateral along the superior right atrium, down the anterolateral wall to the cavotricuspid isthmus area, then up the atrial septum to complete the circuit. As seen in the CS catheter, bystander left atrial activation occurs simultaneously with right atrial septal activation
Case 55
Case Discussion Diagnostic electrophysiology catheters were placed at the right atrium, the His bundle, the right ventricular septum, and in the coronary sinus. A multipolar deflectable catheter was curled in the right atrium with its distal bipoles near the mouth of the coronary sinus, and the proximal bipoles near the roof of the right atrium (Fig. 55.2a). Intracardiac recordings from these catheters show that right atrial activation proceeds in a counterclockwise direction in the left anterior oblique view, from proximal to distal bipole on the duodecapolar catheter (Fig. 55.2b). An ablation catheter was advanced into the right ventricle, pulled back slowly to seat the tip on the cavotricuspid isthmus. Beginning at the edge of the tricuspid annulus, where the ventricular electrogram from the distal bipole of the ablation catheter was larger than the atrial electrogram, a line of
References 1. Disertori M, Inama G, Vergara G, Guarnerio M, Del Favero A, Furlanello F. Evidence of a reentry circuit in the common type of atrial flutter in man. Circulation. 1983;67:434-440. 2. Cabrera JA, Sanchez-Quintana D, Ho SY, Medina A, Anderson RH. The architecture of the atrial musculature between the orifice of the inferior caval vein and the tricuspid valve: the
243
ablation was created across the isthmus, connecting the tricuspid annulus to the inferior vena cava. In the right anterior oblique projection, the ablation catheter was seen to be well separated from the His bundle catheter. In the left anterior oblique projection, the ablation catheter was near the 6 o’clock position. During ablation, the flutter terminated. After ablation, bidirectional conduction block across the isthmus was verified by pacing from either side of the isthmus. The common form of atrial flutter is a macroreentrant right atrial arrhythmia with a counterclockwise pattern of activation as seen from the left anterior oblique view.1 Critical to perpetuation of the arrhythmia is a narrow area of slow conduction bounded by the inferior vena cava and tricuspid valve, called the cavotricuspid isthmus.2 Surgical or catheterbased ablation of the isthmus can result in conduction block and effective treatment of this arrhythmia.3,4
anatomy of the isthmus. J Cardiovasc Electrophysiol. 1998;9: 1186-1195. 3. Klein GJ, Guiraudon GM, Sharma AD, Milstein S. Demonstration of macroreentry and feasibility of operative therapy in the common type of atrial flutter. Am J Cardiol. 1986;57:587-591. 4. Feld GK, Fleck RP, Chen PS, et al. Radiofrequency catheter ablation for the treatment of human type 1 atrial flutter. Identification of a critical zone in the reentrant circuit by endocardial mapping techniques. Circulation. 1992;86:1233-1240.
Case 56 Bradley P. Knight
Case Summary A regular supraventricular tachycardia with a 1:1 AV relationship and a CL of 420 ms is induced in the EP lab. Activation of the high right atrium is coincident with the
QRS complex. Ventricular overdrive pacing is delivered from the right ventricular apex during tachycardia at a cycle length 30 ms less than the tachycardia CL. Figure 56.1 shows the response immediately upon cessation of ventricular pacing. What is the mechanism of tachycardia?
I II V1 V5
Fig. 56.1 This tracing was recorded when ventricular overdrive pacing was stopped after being delivered during a regular supraventricular tachycardia with 1:1 atrioventricular conduction. Shown are surface recordings from leads I, II, V1, and V5, and the intracardiac recordings from high right atrium (HRA), His-bundle electrogram region (HBE), and right ventricular apex (RVA). Note the response after pacing can be described as a VAV response. See text for further description
HRA
390
390
480
420
430
420
420
HBEd HBEm HBEp RVA
390
390
Stim University of Chicago
B.P. Knight Division of Cardiology, Northwestern Medical Center, 676 N. St. Clair, Suite 600, Chicago, IL 60611, USA e-mail:
[email protected] A. Natale et al. (eds.), Cardiac Electrophysiology, DOI: 10.1007/978-1-84996-390-9_56, © Springer-Verlag London Limited 2011
245
246
Case Discussion During a regular supraventricular tachycardia with a 1:1 AV relationship, the differential diagnosis includes AV nodal reentry (AVNRT), orthodromic AV reentry (ORT), and atrial tachycardia (AT). The ventricular overdrive pacing can be used in the EP laboratory to help “rule in” or “rule out” atrial tachycardia as the mechanism.1,2 The principle involved is that when ventricular pacing during tachycardia often results in 1:1 retrograde conduction, the atrial rate accelerates to the ventricular pacing rate until ventricular pacing is stopped. In the case of an AT, the last atrial depolarization that is a result of retrograde VA conduction will block antegrade, because the AV node will be refractory in the antegrade direction after recent retrograde activation. After resumption of the AT, the first atrial depolarization will conduct to the ventricle. After ventricular pacing is stopped, the sequence of activation can be described as atrial–atrial–ventricular (VAAV; Figs. 56.1 and 56.2). In contrast, in the case of an AV nodal dependent tachycardia, such as AVNRT or ORT, ventricular pacing with 1:1 VA conduction entrains the tachycardia which, if it does not terminate, will conduct anterograde over the AV node as soon as the tachycardia resumes. After entraining an AV nodal dependent tachycardia, the sequence A
V
Fig. 56.2 Ladder diagram showing the response to overdrive ventricular pacing during an atrial tachycardia. The response after pacing is a VAAV response. See text for more detail
B.P. Knight A
V
Fig. 56.3 Ladder diagram showing the response to overdrive ventricular pacing during an atrial tachycardia. The response after pacing is a VAV response. This ladder diagram best describes the response seen in Fig. 56.1. See text for more detail
of activation can be described as atrial–ventricular (VAV; Fig. 56.3). When analyzing the response to ventricular pacing the following steps should be taken: 1. Confirm ventricular capture. 2. Confirm acceleration of atrial rate to pacing rate. 3. Identify last atrial depolarization arising from last ventricular paced beat. 4. Identify the next conducted ventricular beat. 5. Categorize the sequence as VAV or VAAV. In this case, the response immediately after ventricular pacing is VAV, which rules out AT. Because the short VA interval excludes ORT as a mechanism,2 the diagnosis must be AVNRT.
References 1. Knight BP, Zivin A, Souza J, et al. A technique for the rapid diagnosis of atrial tachycardia in the electrophysiology laboratory. J Am Coll Cardiol. 1999;33:775-781. 2. Knight BP, Ebinger M, Oral H, et al. Diagnostic value of tachycardia features and pacing maneuvers during paroxysmal supraventricular tachycardia. J Am Coll Cardiol. 2000;36:574-582.
Case 57 Yash Y. Lokhandwala, Anoop K. Gupta, and Ranjan K. Thakur
Case Summary A 23-year-old man with recurrent episodes of paroxysmal palpitations associated with near-syncope was referred for evaluation. Cardiac evaluation showed a normal heart.
Y.Y. Lokhandwala (*) KEM Hospital, Parel, Mumbai, India e-mail:
[email protected] A.K. Gupta Apollo Hospital, Ahmedabad, India e-mail:
[email protected] R.K. Thakur Thoracic and Cardiovascular Institute, Sparrow Health System, Michigan State University, 405 West Greenlawn, Suite 400, Lansing, MI 48910, USA e-mail:
[email protected] Figure 57.1 shows the tachycardia induced during EP study. What is the interpretation?
Case Discussion The tachycardia is an orthodromic AVRT mediated by a right-sided accessory pathway because the tachycardia becomes faster with the disappearance of RBBB. This conclusion is based on the fact that if the bundle branch block is on the same side as the accessory pathway, then the tachycardia path is longer during bundle branch block, leading to a slower tachycardia rate during BBB and faster rate with its disappearance (Fig. 57.2). Parahisian pacing was also performed before and after successful ablation (Figs. 57.3 and 57.4).
A. Natale et al. (eds.), Cardiac Electrophysiology, DOI: 10.1007/978-1-84996-390-9_57, © Springer-Verlag London Limited 2011
247
248
Y.Y. Lokhandwala et al.
RUBY HALL CLINIC 2008037 I II III AVR AVL AVF V1 V2 V3 V4 V5 V6 10 mm/mV 25 mm/s
Fig. 57.1 Induced SVT
Version WIN2000/XP : EPTRACER V0.77
Case 57 RUBY HALL CLINIC
249 Version WIN2000/XP : EPTRACER V0.77
2008037 I
AVF
V1
V6
CS34
CS12
HISD
HISP
RFD
10 mm/mV 100 mm/s
Fig. 57.2 Intracardiac signals during transition. The A is earliest in the RF catheter, followed by the His, and still later in the CS. The two programmed PVCs eliminate the RBBB by depolarizing the IVS and
breaking the transseptal linking which was perpetuating the RBBB. The VA time shortens with the disappearance of RBBB
250
Y.Y. Lokhandwala et al.
RUBY HALL CLINIC
Version WIN2000/XP : EPTRACER V0.77
2008037 I
AVF
V1
V6
CS34
CS12
HISD
HISP
RFD
10 mm/mV 300 mm/s
Fig. 57.3 Para-Hisian pacing before RF. The second complex shows His bundle capture evidenced by a narrower QRS complex. The VA time does not shorten with His bundle capture. Thus the VA conduction is via an AP
Case 57 RUBY HALL CLINIC
251 Version WIN2000/XP : EPTRACER V0.77
2008037 I
AVF
V1
V6
CS34
CS12
HISD
HISP
RFD
10 mm/mV 100 mm/s
Fig. 57.4 Para-Hisian pacing after ablation. There was no VA conduction after ablation; with isoproterenol, VA conduction was seen and, therefore, para-Hisian pacing was performed. Second through sixth
complexes show His capture evidenced by narrower complexes. The VA time shortens with His bundle capture suggesting that the VA conduction is via the AV node
Case 58 Roopinder Sandhu, Dimpi Patel, William R. Lewis, and Andrea Natale
Case Presentation A 22-year-old male with no significant medical history presented with recurrent palpitations, dizziness, and chest discomfort. The patient reports having symptoms of shortness of breath and fatigue. He is currently not taking any medications. His 12-Lead ECG (Fig. 58.1) shows ventricular preexcitation. The baseline intracardiac electrograms prior to atrial pacing are shown in Fig. 58.2. The baseline intervals are: PR, 143; QRS, 104; HV, 27. Figures 58.3 and 58.4 show programmed atrial stimulation. The accessory pathway ERP is 330 ms. Figure 58.5 shows that atrial fibrillation was induced with atrial pacing. Figure 58.6 illustrates that the shortest preexcited RR interval is 298 ms or 201 BPM. These two electrograms suggest that the accessory pathway is not a high-risk pathway. Figure 58.7 shows that during ventricular pacing at 700 ms retrograde conduction is occurring over the AV node and not the accessory pathway. Figure 58.8 shows that VA block while pacing at 680 ms. Figure 58.9 reveals that a wide complex tachycardia was inducible. Figure 58.10 shows the 12-lead ECG of the wide QRS tachycardia. R. Sandhu (*) Department of Cardiology, University of Alberta, Walter Mackenzie Center, Suite 2C2, 8440 112 St, Edmonton, AB T6G 2B7, Canada e-mail:
[email protected] D. Patel St. David’s Texas Cardiac Arrhythmia Institute, 1015 E. 32nd St. #516, Austin, TX 78705, USA e-mail:
[email protected] W.R. Lewis Heart and Vascular Center, MetroHealth Medical Center, Case Western Reserve University, 2500 MetroHealth Drive, Suite H322, Cleveland, OH 44109, USA e-mail:
[email protected] A. Natale Texas Cardiac Arrhythmia Institute, St. David’s Medical Center, 1015 East 32nd Street, Suite 516, Austin, TX 78705 e-mail:
[email protected] Figure 58.11 shows an atrial extrastimulus (pac) during tachycardia. What does the preceding pacing maneuver prove?
Case Discussion The differential diagnosis for patients with a preexcited tachycardia include: atrial arrhythmias, true antidromic AV reciprocating tachycardia, AV nodal reentry with a bystander pathway, or antidromic tachycardia using multiple pathways. The atrial extrastimulus at the time of the refractory His advances the ventricular activation which demonstrates the presence of an accessory pathway and its participation in the tachycardia circuit. The retrograde activation is concentric and a retrograde His prior to atrial activation is shown. The AV node is the likely retrograde conducting pathway. This is not consistent with an atrial tachycardia. We cannot exclude the presence of another pathway (ex concealed accessory pathway not participating in this reentry circuit or a slow conducting retrograde accessory pathway). The optimal ablation site is the region of earliest ventricular activation during maximal preexcitation. Presence of an accessory pathway potential can be guided by a unipolar recording. Figure 58.12 shows three sites that may eliminate the preexcitation. Site three is the most likely to eliminate preexcitation. Figure 58.13 shows application of RF at the preexcitation site and Fig. 58.14 shows that the PR, 176; QRS,100; HV, 50 after ablation. Figure 58.15 shows the post-ablation ECG. In summary, the criteria for diagnosing an antidromic AVRT includes that the QRS configuration during the tachycardia is identical to that obtained during maximal preexcitation; demonstrate the ventricles participate during the tachycardia (1:1 conduction, atrial premature depolarization advances the tachycardia with an identical morphology); the participation of the atria in the tachycardia; and to exclude atrial tachycardia, atrial flutter, or AVNRT using a bystander pathway.
A. Natale et al. (eds.), Cardiac Electrophysiology, DOI: 10.1007/978-1-84996-390-9_58, © Springer-Verlag London Limited 2011
253
254 Fig. 58.1 Baseline ECG
Fig 58.2 Baseline intervals: PR, 143; QRS, 104; HV, 27
R. Sandhu et al.
Baseline ECG
Case 58
Fig. 58.3 Atrial extrastimuli
Fig. 58.4 AP ERP/AV ERP: ³600/330
255
256
Fig. 58.5 Atrial fibrillation
Fig. 58.6 Shortest preexcited RR
R. Sandhu et al.
Case 58
Fig. 58.7 Ventricular pacing: 700 ms
Fig. 58.8 VA block: 680 ms
257
258
Fig. 58.9 Wide complex tachycardia
Fig. 58.10 Antidromic AVRT
R. Sandhu et al.
Case 58
259
Fig. 58.11 Atrial S2 advances tachycardia. CS distal pacing
Site 1
Fig. 58.12 Which site would you select for ablation?
Site 2
Site 3
260
Fig. 58.13 Application of RF energy
Fig. 58.14 Post-ablation intervals: Pr, 176msec; QRS 100 msec; HV 50 msec
R. Sandhu et al.
Case 58
Fig. 58.15 Post-ablation ECG
Bibliography Yee R, Klein GJ, Sharma AD, et al. Tachycardia associated with accessory atrioventricular pathways. In: Zipes DP, Jalife J, eds. Cardiac Electrophysiology. Philadelphia, PA: WB Saunders; 1990:463.
261
Case 59 M. Eyman Mortada, Jasbir S. Sra, and Masood Akhtar
Case Summary
Case Discussion
During ventricular programmed stimulation, the following response was observed (note CS 1,2 is distal) (Fig. 59.1). What are the mechanisms of retrograde conduction seen in this tracing?
To recognize the direction of the atrial activation, it is essential to be acquainted with the normal atrial activation originating from the sinus node “high right atrium.” In this patient, the last atrial beat comes from the high right atrium “sinus node” (Fig. 59.2).
Fig. 59.1 ECG demonstrating the response of program stimulation from the right ventricle with one premature beat after pacing in a steady cycle for six beats. At the top is Lead V1 from surface ECG, followed by the intracardiac tracing. From top to bottom: tracing from the right atrial appendage “HRA,” then proximal coronary sinus to
d istal coronary sinus recording, followed by the His recording, and finally the recording from the right ventricular apex. HRA high right atrium; CS coronary sinus; HISp proximal His; HISm mid His; RVa right ventricular apex
M.E. Mortada (*), J.S. Sra , and M. Akhtar Department of Electrophysiology, Aurora Cardiovascular Services, Aurora Sinai/Aurora St. Luke’s Medical Centers, University of Wisconsin School of Medicine and Public Health, 2801 W. Kinnickinnic River Parkway, #777, Milwaukee, WI 53215, USA e-mail:
[email protected] A. Natale et al. (eds.), Cardiac Electrophysiology, DOI: 10.1007/978-1-84996-390-9_59, © Springer-Verlag London Limited 2011
263
264
M.E. Mortada et al.
Fig. 59.2 Response of program stimulation from the right ventricle with one premature beat after pacing in a steady cycle for 6 beats, with illustration of the activation sequence (arrows). At the top are three surface ECGs including lead I, II, and V1, followed by the intracardiac tracing. From top to bottom: tracing from the right atrial appendage “HRA,” then proximal coronary sinus to distal coronary sinus recording,
followed by the His recording, and finally the recording from the right ventricular apex. HRA high right atrium; CS coronary sinus; HISp proximal His; HISm mid His; HISd distal His; RVa right ventricular apex; H annotation of the His deflection; A annotation of the atrial deflection; V annotation of the ventricular deflection
The first atrial beat appears after a ventricular pacing rhythm, with earliest activity seen in the His, suggestive of retrograde activity over the atrioventricular node (AVN). However, the atrial activity in the second complex is clearly retrograde over the AVN, with earliest activation in the His, followed by the proximal coronary sinus, then the high right atrium. Therefore, the first atrial beat is a fusion beat between the sinus node (the activity in the high right atrium appears prior to the activity in the proximal coronary sinus) and the retrograde activation over the AVN (the activity in the His is earliest). The third atrial beat starts after premature ventricular extrastimuli. Conduction is seen from the distal to the proximal coronary sinus followed by the high right atrium and His, suggestive of retrograde activation over a left free-wall accessory pathway. The fourth atrial beat follows a left bundle QRS complex with the same VA duration and activation sequence as the third atrial complex; hence conduction is retrograde over the left free-wall accessory pathway. The fifth atrial beat has the same activation sequence as the previous two atrial complexes, but the VA duration is shorter. The rationale behind the difference in VA duration is
that the initial electrical activation for the third and fourth ventricular complexes is in the right ventricle apex, which is far from the left free-wall accessory pathway. On the other hand, the initial electrical activation of the fifth ventricular complex is over the normal conduction system to both ventricles simultaneously; thus the electrical activity reaches the accessory pathway rapidly and, subsequently, the VA duration becomes shorter. The origin of the first three ventricular complexes (left bundle branch block morphology) is right ventricular pacing. To understand the fourth ventricular complex, it is first necessary to evaluate the fifth ventricular complex. The fourth atrial complex, as explained previously, has been activated retrogradely through the left free-wall accessory pathway. After activating the atrium, it comes down through the AVN and His-Purkinje system to both ventricles. The AH duration is longer than the baseline sinus rhythm AH (compared to the sixth complex) due to the decremental property of the AVN node. The HV duration in this complex (the fifth complex) is the most accurate duration from the His to the earliest ventricular activation over the normal conduction system. The fourth ventricular complex has left bundle branch block morphology. It is preceded by a His deflection that
Case 59
comes prior to atrial activation. Therefore, this complex did not result from atrial activation. It is either a premature ventricular capture, or it is related to the previous ventricular activity (e.g. bundle branch reentry or repetitive ventricular response). The HV duration is slightly longer than the baseline HV duration when compared to the fifth complex, which is considered the most accurate HV duration over the normal conduction system, as previously mentioned. Hence, the source of this complex is probably bundle branch reentry.
265
Finally, the sixth ventricular complex comes after a sinus beat, which conducts over the AV node, down to the HisPurkinje system, then to both ventricles. The possibility of a ventricular fusion beat occurring over the AV node and the left free-wall accessory pathway in the sixth ventricular complex is excluded due to the fact that the HV duration of this beat is the same as the duration of the most accurate HV duration over the AVN and His-Purkinje system (the fifth beat).
Case 60 Matthew D. Hutchinson
Case Summary A 41-year-old woman without structural heart disease presents with recurrent palpitations due to ventricular bigeminy. She had failed multiple antiarrhythmic medications, including amiodarone. She was referred to our institution for consideration of ablation. Her clinical PVC is shown in Fig. 60.1; the morphology is left bundle, inferior axis with a precordial transition at V3. Based upon the morphology of the PVCs in Fig. 60.1, what is the probable site of origin of the patient’s arrhythmia?
Case Discussion The patient presents with frequent PVCs of a left bundle, inferior axis morphology. When differentiating sites of origin for outflow tract tachycardias, it is most important to consider the precordial transition in light of the anatomical relationships between the RV and LV outflow tracts (RVOT, LVOT). The RVOT is positioned anterior to the LVOT relative to the chest wall; thus the precordial transition for RVOT sites tends to be later than LVOT sites (typically V4 or later).1 Early transitions at V1 or V2 suggest an origin more posterior than the RVOT, most commonly the aortic sinuses of Valsalva (ASOV) or the LVOT. Due to the higher prevalence of RV outflow tract tachycardias, the majority of left bundle inferior axis tachycardias with precordial transition at V3 will originate from the septal (posterior or anterior) aspect of the RV outflow tract. This is because the septal aspect is frequently crescent shaped and both extreme posterior and anterior aspects of the septal surface of the outflow region
M.D. Hutchinson Cardiovascular Division, Department of Medicine, University of Pennsylvania, 3400 Spruce Street, 9 Founders, Philadelphia, PA 19104, USA e-mail:
[email protected] may be recorded to the left of the midseptum. Approximately 20% of outflow tract tachycardias will originate from left sided structures such as: the ASOV, the LVOT, the coronary arterial or coronary venous musculature, or the epicardial surface. Pacemaps from the ASOV produce very different and characteristic morphologies. The non-coronary cusp sits adjacent to the inter-atrial septum, and a large atrial signal (and occasionally a His bundle defection) is recorded there. The right cusp is situated anterior and rightward to the left cusp, producing morphologies with later precordial transitions (usually at V2 or V3) and a larger R wave in lead I. Pacing more leftward within the right cusp as well as within the left cusp produces a progressively more rightward frontal plane axis, a more prominent rS complex in lead I, and an earlier precordial transition (at V1 or V2). Tachycardias originating from between the right and left cusps have a characteristic notching in the S wave in V1. The most characteristic feature of outflow-type tachycardias originating from epicardial structures is delayed initial activation in the precordial leads. Daniels and colleagues describe the maximum deflection index as the ratio of the longest time from QRS onset to maximal QRS deflection (either positive or negative) in any precordial lead to the total QRS duration.2 Ratios of ³0.55 were predictive of epicardial origin with high sensitivity and specificity. Bazan and colleagues further described the presence of Q wave in lead I and the absence of Q waves in the inferior leads as highly predictive of epicardial origin from the basal superior LV.3 Thus for our patient, the precordial transition at V3 presents a variety of possibilities for the site of origin.4 A combination of activation and pacemapping is utilized in these cases. Activation times were later than the onset of the QRS in the RVOT, and the pacemap was a poor match (Fig. 60.2). Detailed mapping of the ASOV was performed which revealed an excellent pacemap from the left cusp with an activation time 31 ms pre-QRS; however ablation at this site was unsuccessful (Fig. 60.2). We then performed mapping of the distal coronary sinus near the origin of the anterior interventricular vein (AIV); the activation time was also 31 ms pre-QRS, and the pacemap was a near perfect match
A. Natale et al. (eds.), Cardiac Electrophysiology, DOI: 10.1007/978-1-84996-390-9_60, © Springer-Verlag London Limited 2011
267
268 Fig. 60.1 12-lead ECG revealing left bundle, inferior axis PVCs in a bigeminal pattern
M.D. Hutchinson I II III aVR aVL aVF V1 V2 V3 V4 V5 V6
a
b LCC activation
Clinical
I
I
II
II
III
RVOT
RCC
LCC
III
aVR aVL aVF
aVL
V1
aVF
V2
V1
V3 V4
Fig. 60.2 (a) Clinical PVC with activation time 31 ms pre-QRS as recorded from the left coronary cusp (LCC). (b) Clinical PVC with pacemaps obtained from the RV outflow tract (RVOT), the right and the left coronary cusps (RCC, LCC)
aVR
V5 V6
V2 V3 V4
ABld
V5
ABlp
V6
(Fig. 60.3). Ablation in this location abolished the patient’s PVCs. In summary, the 12-lead ECG provides important information about the site of origin of outflow tract ventricular
ectopy. However, due to individual variability in the anatomical relationships between these complex structures, detailed activation and pacemapping are of critical importance when ablating outflow tract ectopy.
Case 60 Fig. 60.3 (a) Clinical PVC with activation time 31 ms pre-QRS as recorded from the proximal portion of the anterior interventricular vein (AIV). (b) Clinical PVC with pacemap obtained from the AIV. (c) Fluoroscopy obtained in the right and left anterior oblique projections demonstrating the proximity of two catheters placed in the LCC and proximal AIV
269
a
b
c
CS activation
GCV/AIV
I
I
II
II
III aVR aVL
III
aVL
V1
aVF
V2
V1
V4 V5 V6
RAO
aVR
aVF
V3
Clinical
V2 V3
LAO
V4
Abld
V5
Ablp
V6
References 1. Dixit S, Gerstenfeld EP, Callans DJ, Marchlinski FE. Electro cardiographic patterns of superior right ventricular outflow tract tachycardias: distinguishing septal and free-wall sites of origin. J Cardiovasc Electrophysiol. 2003;14(1):1-7. 2. Daniels DV, Lu YY, Morton JB, et al. Idiopathic epicardial left ventricular tachycardia originating remote from the sinus of Valsalva: electrophysiological characteristics, catheter ablation, and identification from the 12-lead electrocardiogram. Circulation. 2006; 113(13):1659-1666.
3. Bazan V, Gerstenfeld EP, Garcia FC, et al. Site-specific twelve-lead ECG features to identify an epicardial origin for left ventricular tachycardia in the absence of myocardial infarction. Heart Rhythm. 2007;4(11):1403-1410. 4. Tanner H, Hindricks G, Schirdewahn P, et al. Outflow tract tachycardia with R/S transition in lead V3: six different anatomic approaches for successful ablation. J Am Coll Cardiol. 2005;45(3): 418-423.
Case 61 Ronald Lo, Henry H. Hsia, and Amin Al-Ahmad
Case Summary
Case Discussion
A 55-year-old woman with no significant past medical history has been complaining of palpitations for the past 3 years. These symptoms are associated with fatigue, activity intolerance, chest discomfort, and lightheadness. A Holter monitor was ordered which demonstrated greater than 20,000 PVCs per day. These accounted for approximately 25% of her daily heart beats. Her electrocardiogram is shown in Fig. 61.1. Her echocardiogram demonstrated an ejection fraction of approximately 45% with normal valves and cardiac chamber sizes. She was tried on beta blockers without any effect with respect to symptoms. Where is the origin of her premature ventricular contractions?
Analysis of the electrocardiogram demonstrates ventricular bigeminy with left bundle left inferior axis morphology PVCs. This suggested a possible right ventricular outflow tract origin of the PVCs; however, the early transition raises the possibility of a left ventricular outflow tract or aortic cusp site of origin. Initial mapping of the right ventricle and the right ventricular outflow tract did not locate any points of earliest activation that were earlier than ventricular activation. Mapping was then performed in the left ventricle and the aortic root along with the aortic cusps. Careful mapping using CARTO demonstrated a site between the left and right aortic cusps that was −22 ms presystolic to the earliest ventricular electrogram. The unipolar electrogram also demonstrated a QS signal as shown in Fig. 61.2. Pace mapping was also performed in the areas around the aortic cusps, with the best pace mapped site being in between the right and the left aortic cusps. Further analysis of the electrocardiogram demonstrated a qrS pattern in leads V1–V3, which appear to be due to a site of activation in between the right and left coronary cusps. The activation pattern is due to direction of the propagating wave from the right and left coronary cusps as a q wave in lead V1, and the anterior activation pattern from the RVOT to the aortic root as an r wave in V1. The remainder of the ventricular activation produced a large S wave resulting in the qrS pattern seen on the electrocardiogram. Catheter ablation in this region (Fig. 61.3) produced successful termination of the spontaneous PVCs and repeat Holter monitors afterward did not demonstrate any PVCs consistent with the ablated morphology.
R. Lo (*) Riverside Electrophysiology, 4000 14th Street, Suite 209, Riverside, CA 92501, USA e-mail:
[email protected] H.H. Hsia Department of Cardiovascular Medicine, Stanford University, 300 Pasteur Drive, H2146, Stanford, CA 94305, USA e-mail:
[email protected] A. Al-Ahmad School of Medicine, Stanford University, 300 Pasteur Drive, H-2146, Stanford, CA 94305, USA e-mail:
[email protected] A. Natale et al. (eds.), Cardiac Electrophysiology, DOI: 10.1007/978-1-84996-390-9_61, © Springer-Verlag London Limited 2011
271
272
R. Lo et al.
Fig. 61.1 12-lead ECG with ventricular bigeminy with left bundle left inferior PVCs
Unipolar and bipolar activation mapping
43 ms
LAT 2-Map > 134 Points
AoV
−114 ms R
Fig. 61.2 Left ventricular unipolar and bipolar activation mapping demonstrating earliest activation in the region between the left and right coronary cusp and the earliest activation at that site −22 ms presystolic
L 0.70 cm
Case 61
273 Aortogram
Successful ablation site
Bibliography Yamada T, Yoshida N, Murakami Y, et al. Electrocardiographic characteristics of ventricular arrhythmias originating from the junction of the left and right coronary sinuses of Valsalva in the aorta: the activation pattern as a rationale for the electrocardiographic characteristics. Heart Rhythm. 2008;5:184-192.
LAO
Fig. 61.3 Successful ablation site of premature ventricular contraction in between the left and right coronary cusp
Case 62 Richard H. Hongo and Andrea Natale
Case Summary The patient is a 57-year-old woman with newly diagnosed nonischemic dilated cardiomyopathy (LVEF 25–30%) and >11,000 uniform VPCs during a 24-h Holter monitor. She presented for electrophysiology study for ablation of the VPC focus (Fig. 62.1) and to assess for inducible sustained ventricular tachycardia. 3-D electroanatomic activation mapping was performed with a 4-mm Navistar RMT catheter utilizing the CARTO and Stereotaxis remote magnetic navigation systems. Earliest activation (0 ms presystolic) of the recurring VPCs within the right heart was localized to the anteroseptal RVOT, just below the pulmonary valve. Pacemapping was also performed and an 11/12-lead morphology match was achieved at the same site as the earliest activation. Ablation was performed at this site with the 4-mm RMT catheter with temperature and power limited to 52°C and 50 W, respectively. Despite multiple ablations (Fig. 62.2) achieving adequate temperature and power, there were no flurries of ventricular beats during ablations and the VPCs continued to recur. What is the most appropriate next step? Should this patient receive an ICD?
Case Discussion Prominent R waves in precordial leads V1 and V2 is most consistent with an LVOT VPC focus. Right heart electroanatomic mapping reveals earliest activation at the anteroseptal aspect of the RVOT just below the pulmonary valve. The earliest site, however, is on time with, but does not precede, the
R.H. Hongo Sutter Pacific Medical Foundation, California Pacific Medical Center, 2100 Webster Street, Suite 521, San Francisco, CA 94115, USA e-mail:
[email protected] onset of the P wave. Pacemapping at this site is only an 11-out-of-12 leads match with the VPC morphology. Despite the less than ideal activation time and pacemapping match, attempting ablation with the 4-mm catheter is appropriate before proceeding with the higher risk left-sided mapping and ablation. Once, however, the lack of VPC flurries and the persistence of VPCs are apparent, further ablation with higher powered catheters is unlikely to be successful, and the next most appropriate step is to proceed with mapping of the LVOT; the LVOT is more approachable than the epicardium, and the VPC ECG morphology is more suggestive of an LVOT focus. Following transseptal puncture, 3-D activation mapping of the recurring VPCs was performed with remote magnetic navigation within the LVOT. A region of early activation was found just below the aortic valve, along the posteroseptal aspect of the LVOT, but was no earlier than the earliest site in the RVOT (0 ms presystolic). Within this region, however, there was a single site that demonstrated presystolic (−20 ms) activation (Fig. 62.3). The first ablation at this site resulted in a brief flurry of VPCs that was followed by complete cessation of VPCs (Fig. 62.4). Several ablations were performed adjacent to the first ablation, targeting fractioned electrogram signal. There were no recurrent VPCs observed overnight on telemetry monitoring, and the VPC focus has not recurred after 6 months. The LVEF has remained depressed (1.5 mV
Limited endocardial voltage abnormality
R
LAO
Lateral LV epicardium
PA
>1.0 mV
Sinus rhythm
a
I II
Base
R
III aVR
aVL
aVL
aVF
aVF
V3 Fractionated/late potentials
V4 220 bpm)
41 J × 5
A chest X-ray taken in the intensive care unit is shown (Fig. 92.2). What steps could have been taken during the implant to prevent this from occurring?
A. Al-Ahmad School of Medicine, Stanford University, 300 Pasteur Drive, H-2146, Stanford, CA 94305, USA e-mail:
[email protected] A. Natale et al. (eds.), Cardiac Electrophysiology, DOI: 10.1007/978-1-84996-390-9_92, © Springer-Verlag London Limited 2011
373
374
A. Al-Ahmad
Fig. 92.1 Episode of ventricular fibrillation with failed 41 J shock
Reference 1. Russo AM, Sauer W, Gerstenfeld EP, et al. Defibrillation threshold testing: is it really necessary at the time of implantable cardioverter-defibrillator insertion? Heart Rhythm. 2005; 2:456-461.
Fig. 92.2 Chest X-rays showing biventricular system; note lack of left ventricular lead due to failure to place lead during implant
Case 93 M. Eyman Mortada, Jasbir S. Sra, and Masood Akhtar
Case Summary
Case Discussion
A 62-year-old male had a history of nonischemic cardiomyopathy with a left ventricular ejection fraction of 30%, congestive heart failure (New York Heart Association class II-III), and atrial fibrillation. His rhythm was converted to sinus rhythm with DC cardioversion and maintained with Tikosyn therapy. The cardiomyopathy persisted after optimizing his medical therapy for 3–4 months. Therefore, he received a dual-chamber implantable cardioverter defibrillator, and the device was programmed to suppress the atrial fibrillation by pacing the atrium 10 beats faster than his sinus rate to a maximum of 110 bpm with an AF suppression cycle of 30 beats. He has a first-degree AV block with AV delay of 205 ms, so the sensed AV delay was programmed at 250 ms and the paced AV delay was programmed at 300 ms. On an office visit, he complained of multiple episodes of palpitations. During the interrogation of the device, an episode of tachycardia had occurred, similar to multiple episodes that were disclosed on the stored EGMs (Fig. 93.1). What is the mechanism of the tachycardia? What is the best course of action? Fig. 93.2 shows a recording after shortening the PostVentricular Atrial Refractory Period (PVARP), shortening the AV delay, and turning ON the VIP (Ventricular Intrinsic Preference) mode. What is the most likely mechanism of the tachycardia? What is the best course of action?
Current models of pacemakers have advanced programming capabilities. There are multiple features that can be adapted to the individual patient’s needs. However, these sophisticated programming options can create complications of their own. This case is one example. Atrial fibrillation (AF) suppression, by pacing the atrium faster than the sinus rate, is a great tool to reduce the frequency of AF. This patient had no episodes of AF since the device was implanted. Yet, he had multiple symptomatic episodes of other types of tachycardia. The tachycardias started with a programmed pacing in the atrium at faster rates (10 bpm faster) than the sinus rate to suppress AF, and, at the same time, he had two premature ventricular captures (PVC). The second PVC had a retrograde atrial activity, which landed in the PVARP. It was then followed by an atrial pacing at the programmed cycle length (10 bpm faster than the sensed sinus rate) which did not capture the atrium due to its refractory period status (functional non-capture). After 300 ms (paced AV delay), the ventricle was paced and subsequently gave a retrograde atrial activity that landed in the PVARP again, leading to continuous-circuit symptomatic tachycardia. This manifestation had been described by Barold and Levine as pacemaker-repetitive nonreentrant ventriculoatrial synchronous rhythm1. Currently, there is no programmable algorithm available to detect and terminate this condition. There are a few algorithms which may prevent this situation from happening:
M.E. Mortada (*), J.S. Sra, and M. Akhtar Department of Electrophysiology, Aurora Cardiovascular Services, Aurora Sinai/Aurora St. Luke’s Medical Centers, University of Wisconsin School of Medicine and Public Health, 2801 W. Kinnickinnic River Parkway, #777, Milwaukee, WI 53215, USA e-mail:
[email protected] 1. Prolongation of the atrial escape interval to allow atrial myocardium time to recover: This algorithm can be programmed in St. Jude and Medtronic devices by extending the atrial escape interval to 300–350 ms from the sensed atrial activity when it lands in PVARP (e.g. PVC with retrograde atrial conduction). This would prevent the functional non-capture condition. 2. A decrease in the lower base rate (increase lower rate interval): In this scenario, it would be feasible to lower the base rate only if one turned off the atrial fibrillation suppression
A. Natale et al. (eds.), Cardiac Electrophysiology, DOI: 10.1007/978-1-84996-390-9_93, © Springer-Verlag London Limited 2011
375
376
Fig. 93.1 During the interrogation of the device, an episode of tachycardia had occurred. It was similar to multiple episodes that were disclosed on the stored EGMs. From top to bottom: Leadless ECG,
M.E. Mortada et al.
Markers’ channel, atrial bipolar recording, right ventricular bipolar recording, and key parameters. Labels used in this figure are defined in the text
Fig. 93.2 After shortening the PVARP, shortening the AV delay, and turning ON the VIP (Ventricular Intrinsic Preference) mode; this recording was observed
Case 93
mode. But that would increase the risk of atrial fibrillation. Hence, using atrial pacing for AF suppression and terminating this condition if it occurs reduces the number of pacing cycles. That, in turn, increases the frequency of detections of intrinsic atrial activity, leading to interruption of the pacemaker-repetitive nonreentrant ventriculoatrial synchronous rhythm, if it is present. 3. Shortening of the AV delay: This feature is helpful in patients who need ventricular pacing due to atrioventricular block. It allows for longer VA duration in a fixed cycle length, as in this case. When there is longer VA duration, the next atrial pacing comes further out, allowing more time for the atrial tissue to recover during the PVARP if there is atrial activity, leading to capture of the atrium and preventing this condition from happening. After shortening the PVARP, shortening the AV delay, and switching the Ventricular Intrinsic Preference (VIP) mode to ON, the patient again had two PVCs, which induced another type of tachycardia. After the second PVC (fourth sensed ventricular activity), the patient had a retrograde atrial activity during the PVARP, leading to pacemaker-repetitive nonreentrant ventriculoatrial synchronous rhythm for two beats. In these two beats, after the PVC, the paced AV delay was long, with no escaped intrinsic ventricular activity. Due to the programmed VIP, the paced AV delay was shortened, leading to termination of the above condition, but causing the retrograde atrial activity to be out of the PVARP due to the decremental VA conduction, and, therefore, tracking it with ventricular pacing and creating pacemaker-mediated tachycardia (PMT). It is less likely to be an atrial arrhythmia (atrial tachycardia) due to the fixed VA conduction time and the fixed rate at max track rate (120 bpm “500 ms”). There was no far-field ventricular sensing, since the atrial pacing was not inhibited, and there was atrial activity present at the same cycle length of the sinus rate (the sinus cycle length was 630–640 ms as seen in the first interrogation). Additionally, there were extra markers on the atrial lead at the same time as the markers on the ventricular lead. Finally,
377
the rhythm recorded over the ventricle during the tachycardia is all paced beats. Therefore, it cannot be spontaneous _ventricular tachycardia. The best course of action is to increase the PVARP to prevent the PMT from happening. However, this increases the risk of pacemaker-repetitive nonreentrant ventriculoatrial synchronous rhythm. The following modifications of the algorithm prevented the two observed tachycardias: 1. PVARP duration same as it was in the beginning. 2. Detection of PMT algorithm turned ON to terminate the tachycardia when it happens. 3. Shortened AV delay with longer ventricular intrinsic preference. 4. Extended atrial escape interval when atrial activity is sensed in the PVARP. 5. Reduced number of pacing cycles during atrial pacing in the AF suppression algorithm. Each patient is unique in regards to the function of his conduction system and its response to the different pacing maneuvers. Thus, it is essential to try the above algorithms at multiple intervals, to fit the requirements of each patient, until the issue is solved. As a last resort, it may be necessary to turn OFF the AF suppression program to prevent pacemaker-repetitive nonreentrant ventriculoatrial synchronous rhythm. If that is the case, it is important to remember to increase the PVARP Neither decreasing PVARP nor changing atrial sensitivity solves the problem of PMT2.
References 1. Barold SS, Levine PA. Pacemaker repetitive nonreentrant ventriculoatrial synchronous rhythm. A review. J Interv Card Electrophysiol 2001;5:45-58. 2. Ellenbogen KA, Kay GN, Lau CP, Wilkoff BL, Lau CP. Clinical Cardiac Pacing, Defibrillation and Resynchronization Therapy. 3rd ed. Edinburgh, UK: Saunders/Elsevier Health Science, 2006, pp 101.
Case 94 Kenneth A. Ellenbogen and Rod Bolanos
Case Summary A 68-year-old man undergoes implantation of a dual chamber ICD for sustained monomorphic ventricular tachycardia (VT) induced during electrophysiology study. His past medical history is significant for an ischemic cardiomyopathy with an LVEF of 40% and a prior history of syncope. The patient receives a Guidant VitalityTM 2 EL T 167 ICD with a 4087 FlextendTM active fixation atrial lead and a dual coil active fixation RelianceTM G 0185 Gore ICD lead. The R waves at implant were 14 mV and the P waves were 2.5 mV. The following stored EGM was recorded just prior to DFT testing (Fig. 94.1). The first and sixth QRS complexes are preceded by a V sensed event and the third QRS complex has two closely coupled V sensed events. Do these sensed events on the ventricular channel correlate with any physiologic electrical activity? The patient undergoes successful DFT testing at 14 J twice, and the events seen on the ventricular channel are no longer seen. The pocket is closed and the patient is returned to his room uneventfully. The next morning during the post implant device check, the following EGM is elicited during pacing (Fig. 94.2). The atrial output was 3.5 V at 0.5 ms. There is intermittent sensing on the ventricular channel of an event that appears to reproducibly follow the paced P wave. What are possible explanations for why the P wave is being sensed on the ventricular channel (far-field P wave oversensing)? What diagnostic study may aid in explaining this finding? Can the choice of ventricular lead affect the prevalence of this problem? Review of the patient’s morning chest x-ray shows that the ventricular lead has “come back” and has lost most of its “heel” in the right atrium. The decision is made to bring the patient back to the EP lab to reposition the ventricular lead.
K.A. Ellenbogen () Cardiac Electrophysiology and Pacing, Department of Cardiology, VCU School of Medicine, P.O. Box 980053, Richmond, VA 23298-0053, USA e-mail:
[email protected] The ICD lead is positioned more distally out in the apex and a new true bipolar rate sense lead is inserted and positioned in the right ventricular septum. No further evidence of far-field P wave oversensing is seen. The patient underwent repeat DFT testing successfully at 14 J. While the pocket was being closed, the patient received two successive shocks while the cardiac monitor revealed sinus bradycardia in the 1940. Device interrogation shows surface lead I, Atrial egm, ventricular rate/sense egm, and marker channel (Fig. 94.3) just prior to delivery of the shocks. Does the rapid “sensed” ventricular event on the ventricular rate/sense EGM have a likely physiologic explanation?
Case Discussion The atrial, ventricular, and shock electrograms of one of the two episodes are shown (Fig. 94.4). The “rapid ventricular” event is seen only on the rate/sense lead. What should be the next step in diagnosing the cause of this problem? The pocket was opened and the set screws on the rate sense port examined. The screws were appropriately tightened but the artifact remained. A second generator was connected with resolution of the rapid, rhythmic activity previously seen only on the ventricular rate/sense lead. The original generator underwent evaluation by the manufacturer where a large tear in the rate/sense header sealing ring was seen. In Fig. 94.1, the VS event preceding the first and sixth QRS complexes have a sharp EGM and do not appear to correlate with any atrial activity or with any other part of the ventricular electrogram such as the T wave and should be considered “nonphysiologic.” The VS event preceding the third QRS complex could possibly be explained by far-field P wave oversensing. At that time no further evidence of inappropriate sensing on the ventricular channel was seen and DFTs were performed successfully. The next morning there is clear evidence of intermittent sensing of the P wave on the ventricular channel. In this case, lead position must be assessed, and this can be done with a post implant day chest x-ray. If the RV lead is dislodged, implanted
A. Natale et al. (eds.), Cardiac Electrophysiology, DOI: 10.1007/978-1-84996-390-9_94, © Springer-Verlag London Limited 2011
379
380
K.A. Ellenbogen and R. Bolanos
Fig. 94.1 Surface ECG, atrial and ventricular EGM prior to DFT testing
too proximally in the RV septum, or in the RV outflow tract then P wave oversensing can be seen. The P wave pacing output is not particularly high in this patient, but a high atrial pacing output would be more likely to result in far-field P wave oversensing. The ICD lead utilized in this case is an integrated bipolar lead and is more susceptible to this phenomenon. The chest film revealed that the ICD lead had pulled back some and this likely was the fundamental event that resulted in oversensing of the P wave on the ventricular rate/sense lead. The patient was appropriately returned to the lab and not only was the ICD lead positioned more distally, but a separate true bipolar rate sense lead was inserted to minimize the likelihood of future far-field P wave oversensing. Review of the EGMs associated with the two “inappropriate shocks” during pocket closure shows rapid, sharp, and rhythmic VS events that fell into the VF zone and resulted in two shocks. The VS EGMs do not correlate with any ventricular activity and there is no clear atrial activity that is being oversensed in this instance. When both the rate/sense EGM and Shock coil EGMs are compared, it is clear that only the rate/sense lead is being affected. Potential nonphysiologic causes of EMI typically effect both leads and EMI typically has a pulsed, high frequency appearance that is of varying duration seen
on all EGMs. Diaphragmatic myopotential oversensing is typically seen on the rate sense channel with integrated leads like the one used in this patient; but the sharp, well-demarcated mostly regular appearing EGMs are not typical of diaphragmatic myopotentials. The appearance of such nonphysiologic artifacts on only one lead particularly after a recent implant demands close inspection of that lead and the connection to the device header. The rate sense lead was inspected and was unremarkable. With insertion of the header torque wrench, the artifact could be reproduced and the generator was exchanged with elimination of the artifact. The original device underwent inspection by the manufacturer that revealed a large tear in the sealing ring of the rate/sense lead port. Compromise of the set-screw seal plug can lead to oversensing due to air escaping from the header connectivity cavity. This oversensing is typically seen on the ventricular rate sense lead and usually lasts only 1–2 days. This phenomenon can lead to inappropriate detection of ventricular tachycardia/ventricular fibrillation in approximately 20% of cases. To prevent damage to the seal plug during insertion of the torque wrench, the manufacturer recommended approaching the sealing ring at a 45° angle with the torque wrench prior to fully engaging the header screw.
Case 94
Fig. 94.2 Surface ECG, atrial and ventricular EGM on post-op day one
Fig. 94.3 Surface ECG, atrial and ventricular EGM after lead revision and DFT
381
382
K.A. Ellenbogen and R. Bolanos
Fig. 94.4 Atrial, ventricular and shock electrograms during episode that resulted in shock
Bibliography Cheung JW, Iwai S, Lerman BB, Mittal S. Shock-induced ventricular oversensing due to seal plug damage: a potential mechanism of inappropriate device therapies in implantable cardioverter-defibrillators. Heart Rhythm. 2005;2:1371-1375.
Lee BP, Wood MA, Ellenbogen KA. Oversensing in a newly implanted dual-chamber implantable cardioverter-defibrillator: what is the mechanism? Heart Rhythm. 2005;2:782-783. Weretka S, Michaelson J, Becker R, et al. Ventricular oversensing: A study of 101 patients implanted with dual chamber defibrillators and two different lead systems. PACE. 2003;26(Part 1):65-70.
Case 95 Byron K. Lee
Case Summary A 46-year-old man with a history of ventricular tachycardia and ICD implantation presented urgently to the Device Clinic 1 day after experiencing several shocks while having sex. After four painful and startling shocks, they suddenly stopped occurring. Several minutes later he was back to his baseline status and chose to come into the clinic the next day, rather than to go to the emergency department immediately. Interrogation of his Medtronic Marquis ICD showed that there were several episodes of tachycardia that crossed the lower rate cutoff of the VT zone and triggered therapies (Fig. 95.1). These episodes corresponded to when the patient felt his shocks the night before. The device settings are shown in Fig. 95.2. The intracardiac electrogram from one of the episodes of antitachycardic pacing (ATP) is shown in Fig. 95.3. Can we determine if the clinical arrhythmia is a supraventricular tachycardia or a ventricular tachycardia?
Case Discussion The beginning of the recording shows the clinical tachy cardia. There is AV association with one A electrogram for every V electrogram. This suggests that the clinical
arrhythmia is likely a supraventricular tachycardia but it does not rule out ventricular tachycardia. AV association can also occur with ventricular tachycardia when there is 1:1 retrograde conduction. Further examination of the intracardiac electrogram shows that during ATP there is clear capture of the ventricle. During ventricular capture, the atrial rate is unchanged and continues on at exactly the same rate as the clinical tachycardia. This indicates that the clinical tachycardia is driven by the atria, clinching the diagnosis of a supraventricular tachycardia rather than ventricular tachycardia. In this case, exertion preceded the shocks, strongly suggesting that the supraventricular tachycardia is simply sinus tachycardia. Therefore, the shocks were inappropriate. The shocks experienced by this patient could probably have been avoided if his therapy zones were programmed more appropriately for someone his age. 220 – age is a reasonable estimate of the maximum sinus rate for a patient. Therefore, you could expect this patient would reach sinus rates of around 174 bpm with heavy exertion. This rate is higher than the programmed lower rate cutoff for his VT zone which was set at 167 bpm. Therefore, it was not surprising that he had inappropriate shocks. The patient had the lower rate cutoff for his VT zone increased to 182 bpm and he has had no further inappropriate shocks since.
B.K. Lee Division of Cardiology, University of California, San Francisco, 500 Parnassus Avenue, San Francisco, CA 94143, USA e-mail:
[email protected] A. Natale et al. (eds.), Cardiac Electrophysiology, DOI: 10.1007/978-1-84996-390-9_95, © Springer-Verlag London Limited 2011
383
384
B.K. Lee
Fig. 95.1 Episode list ICD Model: Marquis DR 7274 Serial Number: PKC 138209H
Episode Lists Report
Jul 14, 2005 15:21:26 9966 Software Version 4.0 Copyright Medtronic, Inc. 2001
Last Interrogation: Jul 14, 2005 13:18:17 Episodes Last Cleared: Jun 27, 2005 12:30:27 VT/VF Episodes ID#
Date/Time
9 8 7
350 ms VT Rx 6 No Jun 30 15:36:04 VT Jun 30 15:35:07 VT 350 ms VT Rx 2 Yes Jun 30 15:23:02 VT 350 ms VT Rx 2 Yes Last Session (Jun 27, 2005) (Data prior to last session has not been interrogated.)
Type
V. Cycle
Last Rx
Success
Duration 2.8 min 47 s 39 s
SVT/NST Episodes ID#
Date/Time
A. Cycle
V. Cycle
Duration
Reason
(No data since last session.) Last Session (Jun 27, 2005) (Data prior to last session has not been interrogated.)
ICD Model: Marquis DR 7274 Serial Number: PKC 138209H Therapy 1 2 3 4 5 6
VT 167-188 bpm Burst Pacing Ramp Pacing CV 15 J CV 30 J CV 30 J CV 30 J
Parameter Summary Report
VF 188-500 bpm Defib 26 J Defib 30 J Defib 30 J Defib 30 J Defib 30 J Defib 30 J
Brady Pacing
Fig. 95.2 Device parameters
Mode Lower Rate Upper Tracking Rate Upper Sensor Rate
DDD 50 ppm 130 ppm 95 ppm
Jul 14, 2005 15:20:20 9966 Software Version 4.0 Copyright Medtronic, Inc. 2001
Case 95
Fig. 95.3 Intracardiac electrogram during ATP
385
Case 96 Amin Al-Ahmad and Paul J. Wang
Case Summary
Case Discussion
A 20-year-old man with a history of cardiomyopathy, long QT syndrome, ventricular tachycardia, and an implantable cardioverter defibrillator is admitted with multiple ICD shocks. The patient has a Medtronic Maximo DR 7278 that was implanted 2 years ago. Interrogation of the device reveals five episodes classified as nonsustained VF and three episodes classified as VF that are treated with shocks. A representative episode is seen in Fig. 96.1. Device parameters are as follows:
Examination of the episode electrogram reveals episodic bursts of high ventricular rates on the ventricular electrogram. The cycle length during these bursts is less than 200 ms and does not appear to be consistent with a physiologic signal. This electrogram is most consistent with noise related to lead fracture. Lead fracture is not uncommon in younger patients who are very active, and can be the cause of painful inappropriate shocks. An impedance rise is commonly seen with lead fracture, although it is worth noting that at times the lead impedance can be normal. Asking the patient to perform isometric contractions of the upper extremities while checking the lead impedance can unmask a high lead impedance when it is not immediately seen. Figure 96.2 illustrates the lead performance trends report; this shows the lead impedance fluctuating from a normal reading to a very high reading.
Mode
AAI
LRL/URL
70 bpm
VF zone
300 ms (200 bpm)
Ventricular sensitivity
0.3 mV
What is the cause of the shock? Should any programming changes be made?
Fig. 96.1 Episode electrogram prior to shock delivery
A. Al-Ahmad and P.J. Wang (*) School of Medicine, Stanford University, 300 Pasteur Drive, H-2146, Stanford, CA 94305, USA e-mail:
[email protected];
[email protected] A. Natale et al. (eds.), Cardiac Electrophysiology, DOI: 10.1007/978-1-84996-390-9_96, © Springer-Verlag London Limited 2011
387
388
A. Al-Ahmad and P.J. Wang
Ventricular pacing impendance At implant Last
584 Ω 584 Ω
Highest Lowest
>3,000 Ω 400 Ω
Ω >3,000 2,000 1,500 1,000 800 600 400 300 180 and VF >200 bpm. Interrogation of the device reveals several nonsustained episodes of rapid ventricular rates on the device arrhythmia logbook (Fig. 100.1) with one event categorized as VF with an associated diverted shock. The atrial, RV (nearfield), and shock (farfield) electrograms at the initiation of this event are shown (Fig. 100.2). At the top of the figure, one sees marked bradycardia with no ventricular pacing despite the programmed lower rate limit of 60 bpm. On the bottom of the figure, there are several ventricular sensed events falling within the VT and VF zones on the RV EGM with no correlating EGMs on the shock electrogram. The sharp, nearly constant activity being sensed on the RV channel is most likely due to what? What is the likely cause of the patient’s reported presyncope. What feature in this device guards against absolute inhibition of pacing in this case? As a result of the oversensing of the noise on the RV channel, an episode is declared by the device. At the top left hand side of Fig. 100.3, the capacitor begins to charge, but ultimately the shock is aborted. Why did this occur?
The nonsustained episodes seen on the arrhythmia logbook are all due to the same cause, device oversensing of noise on the ventricular channel. The noise is nearly constant across systole and diastole and is of regular amplitude. EMI can have this pattern, but its presence on only one channel makes EMI less likely the culprit. Diaphragmatic myopotentials are classically seen predominantly on the RV channel particularly if the lead is positioned in the inferior apex of the right ventricle. The EGMs due to oversensing of myopotentials tend to be more varied in amplitude and frequency with often respiratory variability. Further interrogation of the device revealed high impedance on the RV lead pointing to a lead fracture as the underlying problem. The patient’s symptoms were likely due to the profound bradycardia caused by inhibition of ventricular pacing from the oversensing of the noise on the RV lead. Pacing was intermittently seen as evidence by the VP-Ns seen on the marker channel due to a feature of the device where if there is “continuous noise” during the noise window of the device (VN markers on the marker channel), the device will pace at the lower rate limit to prevent asystole from oversensing noise. In this instance, the oversensing nearly resulted in delivery of a shock after the device declared a VF episode (bottom left, Fig. 100.2), but the therapy was diverted during redetection (Fig. 100.3). This particular device requires 6/10 beats in the tachycardia zone during redetection to proceed with therapy.
K.A. Ellenbogen Cardiac Electrophysiology and Pacing, Department of Cardiology, VCU School of Medicine, 980053, Richmond, VA 23298-0053, USA e-mail:
[email protected] A. Natale et al. (eds.), Cardiac Electrophysiology, DOI: 10.1007/978-1-84996-390-9_100, © Springer-Verlag London Limited 2011
399
400 Fig. 100.1 Device arrhythmia logbook
K.A. Ellenbogen Guidant
VENTAK PRIZM 2 DR Arrhythmia Logbook Report
Date/Time
Episode
Rate bpm
Type zone
34 33 32 31 30 29 28 27 26 25 24 23 22 21 20 19 18 17 16 15 14 13 12 11 10 9 8 7 6 5 4 3 2 1
05 - DEC - 03 04 - DEC - 03 04 - DEC - 03 04 - DEC - 03 27 - NOV - 03 10 - NOV - 03 01 - OCT - 03 09 - SEP - 03 27 - JUL - 03 15 - JUL - 03 14 - JUL - 03 13 - APR - 03 13 - JAN - 03 07 - JAN - 03 31 - DEC - 02 29 - DEC - 02 29 - DEC - 02 23 - DEC - 02 25 - NOV - 02 24 - NOV - 02 24 - NOV - 02 24 - NOV - 02 24 - NOV - 02 24 - NOV - 02 24 - NOV - 02 24 - NOV - 02 22 - NOV - 02 22 - NOV - 02 15 - OCT - 02 14 - MAY - 02 20 - JAN - 02 08 - AUG - 01 08 - AUG - 01 08 - AUG - 01
End of Report
09 : 58 19 : 45 19 : 45 16 : 51 20 : 11 19 : 32 20 : 19 15 : 03 19 : 05 21 : 25 20 : 02 13 : 39 20 : 54 19 : 54 19 : 44 20 : 34 08 : 18 22 : 44 00 : 22 23 : 20 22 : 28 22 : 18 20 : 46 07 : 07 06 : 45 03 : 15 04 : 35 04 : 01 19 : 45 11 : 06 14 : 58 10 : 49 10 : 41 10 : 35
ATR ATR ATR ATR Spont Spont Spont ATR Spont Spont Spont ATR Spont Spont Spont Spont ATR Spont PMT PMT PMT PMT PMT PMT PMT PMT PMT PMT Spont ATR ATR Induce Induce Induce
VF
VF VF VF
137 253 150 122 138 132 174 100 130 164 176 104 169 143 185 118 93 155 120 120 120 120 120 120 120 120 120 120 155 105 89 255 316 245
Therapy/ Duration 00 : 16 m : s 00 : 07 m : s 00 : 08 m : s 00 : 05 m : s Nonsustained Nonsustained Nonsustained 00 : 05 m : s Nonsustained Nonsustained Nonsustained 00 : 14 m : s Nonsustained Nonsustained Diverted Nonsustained 00 : 06 m : s Nonsustained
Nonsustained 00 : 06 m : s 00 : 07 m : s 17 J 11 J, 17J 31J
V > A
Stab ms
A F i b
Ons
F F F
N/R N/R N/R
O O O
50% 56% 59%
F F F
N/R N/R N/R
O O O
56% 59% 34%
F F T F
N/R N/R 219 N/R
O O O O
59% 66% 59% 47%
F
N/R
O
72%
F
N/R
O
72%
T T T
41 54 96
O O O
N/R N/R N/R
Case 100
Fig. 100.2 Atrial, ventricular and shock electrograms during onset of stored episode
401
402
Fig. 100.3 Atrial, ventricular and shock electrograms during stored episode. Note, therapy diverted
K.A. Ellenbogen
Case 101 Amin Al-Ahmad and Paul J. Wang
Case Summary A 68-year-old man with coronary artery disease, congestive heart failure, and ICD is admitted with multiple ICD shocks for an ablation procedure. The device, a Boston Scientific Vitality HE, is interrogated. Atrial and ventricular lead parameters are within acceptable limits and are unchanged from prior device testing. An example of a representative episode is shown in Fig. 101.1. His device is set with a VT zone at 165 bpm and a VF zone at 200 bpm. He is programmed to receive two ATP trains followed by shock in the VT zone, and to maximum energy (41 J) shocks in the VF zone. Would an atrial flutter ablation result in a reduction in the number of shocks?
Case Discussion Examination of the stored electrogram reveals atrial flutter with ventricular pacing followed by an acceleration of the ventricular rate. Is this acceleration of the ventricular rate
conducted atrial flutter, in which case an atrial flutter ablation would be potentially helpful. Or is this ventricular tachycardia? In this case we do not have an intrinsic electrogram in sinus rhythm (or atrial flutter) to compare the shock morphology with that of the rapid ventricular rate. While conducted atrial flutter is possible, it is very unlikely as we would not expect a patient who is in ventricular pacing during atrial flutter to suddenly begin to rapidly conduct. Indeed, further history reveals that the patient is pacemaker dependent. Thus, this episode represents VT. Programming the device to add more ATP or to change the ATP to be more aggressive may be helpful, although this may result in a higher risk of inducing VF. In addition, he has not been responding to ATP and had been receiving multiple shocks despite ATP. Antiarrhythmic medications may also play a role in the management of this condition. In this patient, a VT ablation resulted in a significant reduction of spontaneous VT. Atrial flutter ablation was also performed at that time.
A. Al-Ahmad (*) and P.J. Wang School of Medicine, Stanford University, 300 Pasteur Drive, H-2146, Stanford CA 94305, USA e-mail:
[email protected];
[email protected] A. Natale et al. (eds.), Cardiac Electrophysiology, DOI: 10.1007/978-1-84996-390-9_101, © Springer-Verlag London Limited 2011
403
404
Fig. 101.1 The shock electrogram showing a typical episode resulting in shock
A. Al-Ahmad and P.J. Wang
Case 102 Kenneth A. Ellenbogen
Case Summary A 72-year-old man with a history of congestive heart failure due to non-ischemic cardiomyopathy and an implantable defibrillator presents to clinic for evaluation of palpitations and presyncope. The patient denies having received any shocks since his device was implanted 9 months ago. Prior interrogation of his device showed normal function except for atrial lead “sensing issues.” The atrial sensitivity was programmed to 0.9 mV. Interrogation of his Medtronic GemTM DR 7271 reveals several VT/VF episodes and no shocks were delivered. Device settings are as follows: Mode
DDD
Lower rate limit/upper rate limit
60/120
VT zone (rate > 180)
ATP × 3 (burst), 21 J × 1, 31 J × 4
VF zone (rate > 210 bpm)
31 J × 5
The following atrial and ventricular EGMs with the corresponding marker channels are shown (Fig. 102.1) during detection of a tachyarrhythmia in the VT zone. What is the
differential diagnosis for the tachyarrhythmia falling in the VT zone? Why does the device characterize the arrhythmia as VT? Once the device commits to therapy, ATP is delivered with the third ATP resulting in termination of the tachycardia (Fig. 102.2).
Case Discussion In this case, the third ATP sequence resulted in termination of the tachyarrhythmia with return to normal sinus rhythm (NSR) with some PVCs. Review of the sensed atrial rate (marker channel) during detection (Fig. 102.1) shows no clear correlation with the ventricular rate with clearly more Vs than As. Yet on the atrial channel the atrial EGM shows an atrial rate concordant with the ventricular rate as sensed on the ventricular lead. Here there are rapid atrial EGMs that are at the same rate as the ventricular EGMs, but are not sensed by the device as evidenced by their absence on the marker channel along with slower atrial EGMS falling at the
Fig. 102.1 Stored EGM showing the atrial and ventricular EGM, the marker channel and intervals at the onset of tachycardia event
K.A. Ellenbogen Cardiac Electrophysiology and Pacing, Department of Cardiology, VCU School of Medicine, Richmond, VA 23298-0053, USA e-mail:
[email protected] A. Natale et al. (eds.), Cardiac Electrophysiology, DOI: 10.1007/978-1-84996-390-9_102, © Springer-Verlag London Limited 2011
405
406
K.A. Ellenbogen
Fig. 102.2 Stored EGM showing ATP terminating the tachycardia
sinus bradycardia rate which are sensed and evident on the marker channel. Based on these observations, the arrhythmia mechanism differential diagnoses include SVT with a rapid ventricular response with atrial undersensing, ventricular tachycardia with VA dissociation (appropriate A sensing), and less likely ventricular tachycardia with VA conduction and atrial undersensing of the retrograde As. VT with 1:1 VA conduction and atrial undersensing is less likely as the sensed atrial rate appears to march through at a regular rate independent of the ventricular rate. If there was 1:1 VA conduction, the atrial rate would be dictated solely by the ventricular rate. SVT with one to one conduction is less likely as the atrial sensed events march out regularly independent of the ventricular rate. This event is an example of VT with VA dissociation, appropriate atrial sensing during sinus rhythm,
and an atrial EGM displaying an atrial signal plus far-field R waves that are appropriately not sensed as evident on the marker channel (although far-field R waves are “confusingly” visible on the marker channel). The device correctly classified the arrhythmia as VT based on the ventricular EGM and V > A. Additionally, this is an example of “farfield” and “near-field” electrograms. On the atrial channel, the near-field EGM is the sharp signal and this is sensed correctly by the device as atrial activity. The other electrogram on the atrial channel is far field, and it represents ventricular activity sensed in the atrium. It is a “far-field” EGM as it has a low frequency, “non-sharp” appearance consistent with a sensed signal from a further away source. It is appropriately not sensed on the atrial channel, but certainly is confusing when one first looks at the recorded strip shown here.
Case 103 Amin Al-Ahmad and Paul J. Wang
Case Summary
in the shock? Why does the patient still receive a shock despite termination of the arrhythmia?
A 58-year-old male with a history of congestive heart failure and an ICD comes to clinic after receiving an ICD shock while mowing the lawn. The patient was not symptomatic during the episode. His left ventricular ejection fraction is 25% and he is being treated with appropriate medications including an angiotensin converting enzyme inhibitor and a beta-blocker. Evaluation of his Medtronic EnTrust ICD shows lead thresholds, sensing, and impedance in an acceptable range and similar to prior device testing. The device is programmed with a VT zone set to 400 ms (150 bpm) and a VF zone set to 320 ms (188 bpm). Interrogation of the episode that resulted in shock reveals a 1:1 tachycardia with a cycle length of 300 ms (200 bpm) that spontaneously terminates prior to shock (Figs. 103.1 and 103.2). What is the rhythm that results
Case Discussion Examination of the interval plot reveals that the patient is initially tachycardic with a heart rate of 133 bpm prior to detection (Fig. 103.1). The electrogram (Fig. 103.2) is consistent with sinus tachycardia. The heart rate then accelerates to approximately 300 ms (200 bpm). The electrogram during this 1:1 tachycardia is similar to that of the electrogram prior to rate acceleration. This suggests that this rhythm acceleration is more consistent with a supraventricular tachycardia (SVT) such as atrial tachycardia with 1:1 conduction, rather than ventricular tachycardia with retrograde 1:1 conduction.
ATP Shocks Success ID# Seq
Type VF
0
35J V−V
Yes
Date
Time Duration Avg bpm Max bpm Activity at hh:mm hh:mm:ss A/V A/V Onset
21 08-Dec-2006 23:53
:15 214/214
VF = 320 ms Detection
A−A
---/---
Rest
VT = 400 ms 34.3 J
Interval (ms) 1,500 1,200 900 600
Term.
400 200
Fig. 103.1 Interval plot showing a 1:1 tachycardia that results in shock
−20
−15
−10
−5
0 Time (s)
5
10
15
20
A. Al-Ahmad (*) and P.J. Wang School of Medicine, Stanford University, 300 Pasteur Drive, H-2146, Stanford, CA 94305, USA e-mail:
[email protected];
[email protected] A. Natale et al. (eds.), Cardiac Electrophysiology, DOI: 10.1007/978-1-84996-390-9_103, © Springer-Verlag London Limited 2011
407
408
Fig. 103.2 Episode electrograms and marker channels
A. Al-Ahmad and P.J. Wang
Case 103
Despite the fact that this SVT spontaneously terminates and the heart rate decreases to 133 bpm, the patient still receives a shock. To understand this we need to understand the reconfirmation algorithm in this device. As the device capacitor charge ends (CE on the marker channel), the device attempts to reconfirm prior to discharge. To “reconfirm” this device requires two intervals to be detected after the charge end that are faster than the VT interval + 60 ms, in this case 460 ms. In this case the intervals are 450 ms; thus although these intervals are lower than the VT rate cutoff, they still result in therapy delivery. Reprogramming of the device to increase the VT rate cutoff may eliminate this problem; however the risk of doing
409
this is possible under detection of VT. Another option may be to increase the number of intervals needed to detect VF from 12/16 to 18/24; however this may cause delay in therapy. In this case it was determined that the patient was not taking his beta-blockers. Resumption of beta-blocker therapy would potentially decrease the maximum rates during exertion and would decrease the likelihood of this type of inappropriate shock without any device programming changes. This case illustrates the importance of understanding the detection and reconfirmation algorithms of ICDs.
Case 104 Kenneth A. Ellenbogen
Case Summary A 65-year-old woman with a history of syncope and congestive heart failure due to nonischemic cardiomyopathy presents to the ER after reportedly receiving several shocks from her ICD. Telemetry in the ER shows normal sinus rhythm with some single premature ventricular contractions (PVCs). Interrogation of her Medtronic MaximoTM DR 7278 reveals that the device had delivered three shocks for a tachyarrhythmia in the VF zone. Device settings are as follows: Mode
DDD
Lower rate limit/upper rate limit
60/120
VT zone (rate > 166 bpm)
ATP × 1 (burst), 21 J × 1, 31 J × 4
VF zone (rate > 188 bpm)
31 J × 5
The atrial and ventricular EGMs along with the marker channel are shown (Fig. 104.1) leading up to the delivery of the first shock. The device categorizes the ventricular arrhythmia as VF and the atrial arrhythmia as an SVT (double tachycardia). Is this correct and why? The interval plot is shown (Fig. 104.2). What effect does the first 34.5-J shock have on the double tachycardia?
Case Discussion At the initiation of the event, a regular rapid arrhythmia of 190 ms is evident on the atrial channel with a coexisting irregular ventricular rhythm well below the VT zone cutoff of 166 bpm. The most likely initial diagnosis is atrial
K.A. Ellenbogen (*) Cardiac Electrophysiology and Pacing, Department of Cardiology, VCU School of Medicine, P.O. Box 980053, Richmond, VA, USA, 23298-0053 e-mail:
[email protected] fibrillation with a rapid and irregular ventricular rhythm. This relationship is evident at the beginning of the interval plot (Fig. 104.2). Subsequently, the ventricular rate abruptly drops into the VF zone twice as self-terminating rapid VT/VF episodes followed by a third VF episode that is sustained long enough to not only be detected, but to result in delivery of a 34.5-J shock. An alternative but less likely explanation is that these are brief episodes of a rapid and regular ventricular response to the atrial arrhythmia. This is less likely because the patient’s atrial rate is extremely rapid (although monomorphic in appearance), and thus more consistent with atrial fibrillation, than an organized atrial tachycardia. Additionally, there are subtle changes in the near field, or ventricular sensed electrogram suggesting this arrhythmia is more likely a ventricular tachyarrhythmia (a rapid SVT with aberration cannot be excluded). This first shock terminates the atrial arrhythmia (Fig. 104.3) and transiently terminates ventricular tachycardia, as well. The ventricular tachycardia quickly reinitiates, with the rate almost identical to the rate of the rapid and regular tachycardia seen prior to the first shock, and requiring a second 34.5-J shock that is also unsuccessful. In Fig. 104.4, we see the successful termination of the ventricular tachyarrhythmia with the third 34.5 J shock. It is worth noting here, that the atrial rhythm is now sinus and we clearly have a ventricular tachycardia with AV dissociation prior to shock delivery. The ventricular electrogram recorded from the rate sensing electrodes here is also different from what the rate sensing electrogram recorded during the atrial tachyarrhythmia prior to the first shock during what we thought were short bursts of ventricular tachycardia, even though the rates are similar. We have no simple explanation for this discrepancy. This case is most likely an example of appropriate classification by a device of a double tachycardia with termination of the atrial arrhythmia by the first shock followed by subsequent termination of the ventricular tachycardia by the third shock. In patients with VT/VF, dual tachycardias are quite common with a prevalence as high as 20%. Additionally, when therapy delivered for the VT/VF fails to convert the atrial arrhythmia, the time to the next VT/VF therapy is significantly shorter than
A. Natale et al. (eds.), Cardiac Electrophysiology, DOI: 10.1007/978-1-84996-390-9_104, © Springer-Verlag London Limited 2011
411
412
Fig. 104.1 Stored EGM showing the atrial and ventricular EGM, the marker channel and intervals at the onset of the episode
K.A. Ellenbogen
Case 104
413
Fig. 104.2 Episode interval plot
AFL terminates
VT continues
Fig. 104.3 Stored EGM showing the first shock
414
K.A. Ellenbogen
Fig. 104.4 Stored EGM showing final successful shock
when both arrhythmias are terminated by the initial shock. It has been shown that implanting dual chamber devices with atrial therapy capabilities in those with standard ICD indications does not reduce the incidence of VT/VF episodes.
Bibliography Gradaus R, Seidl K, Korte T, et al. Reduction of ventricular tachyarrhythmia by treatment of atrial fibrillation in ICD patients with
dual-chamber implantable cardioverter/defibrillators capable of atrial therapy delivery: the REVERT-AF Study. Europace. July 2007;9(7):534-539. Stein KM, Euler DE, Mehra R, et al. Do atrial tachyarrhythmias beget ventricular tachyarrhythmias in defibrillator recipients? J Am Coll Cardiol. July 2002;40(2):335-340.
Case 105 Kenneth A. Ellenbogen and Rod Bolanos
Case Summary A 59-year-old man presented to a local Emergency Room after receiving six shocks from his defibrillator. He reported feeling palpitations and lightheadedness followed by six consecutive shocks. He denied frank syncope. The patient had a single chamber Medtronic ICD and a MedtronicTM 6936 defibrillator lead implanted approximately 2 years ago.
Interrogation of his device (Fig. 105.1) reveals an episode lasting 2 min during which six episodes of “VF” (rate 2,500 W is not germane to the shock; as seen in the VT/VF report, the shock was delivered in the VF zone based solely on the rapid ventricular rates; detection enhancements did not play a role. In this case, the atrial impedance is high due to absence of an atrial lead; an atrial lead was not implanted due to chronic atrial fibrillation. The RV lead impedance is high and suggestive of loose set screw or fracture; typically, any value >2,000 W is abnormal. In the perioperative period, a loose set screw would be far more likely than lead fracture to give rise to these findings. The episode electrogram (Fig. 113.2, bottom panel) shows a near-field electrogram (Vtip-Vring) that is oversaturated with the type of noise characteristic of make-break contact artifact seen in lead fracture or loose set screw. The presence of V-V intervals in the nonphysiologic range below 150 ms also favor fracture/loose set-screw. These short intervals lead to detection of VF (“FD,” the fourth marker). Following detection, only “VS” (ventricular sensed event) markers are present, irrespective of the V-V interval. This is an idiosyncracy of the Medtronic system, which only displays “VS” markers once detection is met. The “CE” marker denotes charge end, and “CD” charge delivery after a brief confirmation interval. In Fig. 113.3, provocative maneuvers are performed with detection turned off to further identify the malfunctioning component. In each panel, electrograms using different sources are recorded during a hand shake. Note that the only electrogram that did not have artifact present during the handshake was the can-RV coil (top right panel). The RVtip-RVring (top left), RVtip-RV distal coil (bottom left), and RVtip-LVtip (bottom right panel) all contain artifact (sharp, nonphysiologic deflections). Thus, any electrogram that incorporates the RV tip has a noisy signal during handshaking, indicating a loose setscrew or lead fracture involving the RV tip conductor. At surgical revision, reinserting the RV distal electrode connector in the header and tightening the screw eliminated the malfunction.
A. Natale et al. (eds.), Cardiac Electrophysiology, DOI: 10.1007/978-1-84996-390-9_113, © Springer-Verlag London Limited 2011
443
444
P.A. Friedman and C.D. Swerdlow
May 02, 2006 09:50:41 9998 Software Version 2.0 Copyright Medtronic, Inc. 2004
ICD Model: InSync Sentry 7299 Serial Number: PRK 124359H
VT/VF Episode #12 Report
Page 1
ID#
Date/Time
Type
V. Cycle
Last Rx
Success
Duration
12
May 02 10:20:49
VF
140 ms
VF Rx 1
Yes
10 s
Interval (ms) 2,000 1,700 1,400 1,100 800
V−V
VF = 320 ms
A−A
FVT = 260 ms 34.8 J
600 400 200 −40
−35
−30
−25
−20
−15
−10
−5
0
5
10
15
Time (s) [0 = Detection]
Fig. 113.1 Top panel: surface telemetry recorded at the time of shock delivered during a handshake. Bottom panel: Episode report from the same episode Lead Performance EGM amplitude pacing impedance Defibrillation impedance SVC impedance
Atrial Not Taken >2,500 Ω
Note 1: 0−35 J charge time not available.
Atip Aring
Fig. 113.2 Interrogation following the shock. Top panel: lead impedance values. Bottom panel: stored electrograms from the event that triggered the shock
Vtip Vring
40 Ω
RV 6.8 mV 1,936 Ω 32 Ω
LV 324 Ω
Case 113
Fig. 113.3 Electrograms recorded during provactive maneuver (handshake) while detection is turned off
445
Case 114 Paul A. Friedman and Charles D. Swerdlow
Case Summary
interfence is characterized by normal lead impedance values, noise throughout the cardiac cycle, and a greater amplitude signal on the far-field than the near-field elecA 67-year-old man received a Guidant 1850 single chamber trogram. The latter occurs since the far-field electrogram defibrillator on an Endotak™ right ventricular lead for sudis recorded from larger electrodes with a greater interelecden death prevention in the setting of an ischemic cardiotorde spacing than the near-field electrogram, resulting in myopathy. He was using an auger when he received a shock a larger “antenna” that is more susceptible to external without antecedent symptoms. The ICD is interrogated and signals. Integrated bipolar leads (which includes the the episode electrograms shown in Fig. 114.1. Lead impedEndotak™) are more likely to record EMI than true bipoance values were normal. What is your diagnosis, and lar leads. See Case 108 for depiction of integrated and management plan? bipolar leads. Figure 114.2 shows examples of oversensing of extra-cardiac noise. The far-left panel shows a lead fracture, characterized by saturated high frequency elecCase Discussion trograms that account for less than 10% of the cardiac cycle. Artifact is seen on the near-field recording to a The tracing shows (from top to bottom) the near-field ven- greater extent than the far-field recording. With a lead tricular electrogram, far-field ventricular electrogram, and fracture, the lead impedance may be elevated; an impedmarkers and V-V intervals. Halfway through the top panel, ance >2,000 W is highly suggestive. The middle panel high frequency noise becomes apparent on both the near- shows myopotential oversensing. The electrogram affected field and far-field electrograms. The presence of multiple depends on the source; diaphragmatic oversensing affects high frequency events/cardiac cycle that are unrelated to the near-field channel and may lead to inappropriate the R-wave is characteristic of oversensing of extra- shocks. Pectoralis oversensing distorts morphology cardiac signals. Extra-cardiac signals may be due to lead templates in some systems, leading to SVT-VT misclasfracture/loose set screw, myopotential oversensing, or sification with use of morphology-based detection electromagnetic interference (EMI). Electormagnetic enhancements. The far right panel depicts EMI.
P.A. Friedman (*) Division of Cardiovascular Diseases and Department of Internal Medicine, Mayo Clinic, 200 First Street SW, Rochester, MN 55905, USA e-mail:
[email protected] C.D. Swerdlow Division of Cardiology, David Geffen School of Medicine, Cedars-Sinai Medical Center, Los Angeles, CA, USA e-mail:
[email protected] A. Natale et al. (eds.), Cardiac Electrophysiology, DOI: 10.1007/978-1-84996-390-9_114, © Springer-Verlag London Limited 2011
447
448
Fig. 114.1 Device interrogation following shock delivery
P.A. Friedman and C.D. Swerdlow
Case 114 Fig. 114.2 Characteristic electrograms recorded in the setting of lead fracture (left panel), myopotential oversensing (middle panel), and electromagnetic interference (EMI, right panel)
449
Case 115 Paul A. Friedman and Charles D. Swerdlow
Case Summary
What changes in bradycardia pacing parameters should be made?
A 78-year-old man with ischemic cardiomyopathy (NYHA Class IV, left-ventricular ejection fraction 14%) and renal failure underwent implantation of a prophylactic, cardiac-resynchronization ICD in 2005. He had sinus rhythm with first degree AV block and left bundle branch block. His heart failure improved to Class III and ejection fraction improved to 21%. In May 2007, he saw his cardiologist for a routine followup visit. ICD interrogation showed 96% ventricular pacing and no episodes of VT. ECG monitoring showed frequent ventricular tracking of atrial premature complexes, resulting in asymptomatic rhythm irregularities. To reduce this tracking, bradycardia programming was changed to the settings shown in left panel of Fig. 115.1. In August 2007, he returned for a routine visit, reporting an increase in exertional dyspnea and reduction in exercise capacity. ICD interrogation showed 41% ventricular pacing and asymptomatic, devicedetected VT, all terminated by antitachycardia pacing. The plasma natriuretic peptide concentration has increased from 165 pg/mL in May to 536 pg/mL. Figure 115.1 (right panel) shows a stored EGM of device-detected VT with onset identical to the seven other episodes. ICD programming for detection and therapy of VT/VF is shown below: Intervals to detect
Detection interval (ms)
Therapy
VF
18/24
320
35 J Shock × 6
Fast VT
18/24
240
ATP X1, 35 J Shock × 6
VT
16
400
ATP X3, 35 J Shock × 6
P.A. Friedman (*) Division of Cardiovascular Diseases and Department of Internal Medicine, Mayo Clinic, 200 First Street SW, Rochester, MN 55905, USA e-mail:
[email protected] C.D. Swerdlow Division of Cardiology, David Geffen School of Medicine, Cedars-Sinai Medical Center, Los Angeles, CA, USA e-mail:
[email protected] Case Discussion Figure 115.1 shows an episode of monomorphic VT at cycle length 340 ms terminated by antitachycardia pacing. The post-therapy rhythm is a SVT with 1:1 AV association. The pattern of atrial and ventricular timing at the end of the antitachycardia pacing sequence indicates that the likely diagnosis is atrial tachycardia with first degree AV block. The rhythm preceding VT is sinus tachycardia at cycle length 560 ms without cardiac-resynchronization pacing. The patient’s heart failure has worsened because of reduction in cardiac resynchronization pacing, which generally needs to be applied to more than 90% of QRS complexes for therapeutic effect. The root cause of insufficient cardiac- resynchronization pacing is the long post-ventricular atrial refractory period (PVARP) of 500 ms, which was increased in May to reduce tracking of atrial premature complexes. This prevents tracking of the sinus P waves that fall within the PVARP (VS–AR interval 200–270 ms) as the sinus cycle length decreases. This episode and the other seven episodes of VT were preceded by an atrial-paced event, setting up safety pacing, which initiated VT. Assuming the activity sensor at 0, the device will pace at the lower rate (70 bpm), and thus the VS–AP interval = lower rate interval – PAV ~ 850 − 130 ms = 720 ms. After the VS–VS interval shortens to 510 ms, probably because of a premature ventricular complex, the measured VS–AP interval is 710 ms, indicating that the sensor was driving the pacing rate slightly faster than the lower rate of 70 bpm. Note also that AR–AP interval is 300 ms, which means that the noncompetitive atrial pacing feature (NCAP), designed to prevent pacing-induced atrial fibrillation, withheld the Ap event until the NCAP timer expired. The conducted VS beat from the preceding AR P wave times in the cross-talk window after the AP event. This initiates “Safety Pacing” with an AP–VP interval of
A. Natale et al. (eds.), Cardiac Electrophysiology, DOI: 10.1007/978-1-84996-390-9_115, © Springer-Verlag London Limited 2011
451
452
P.A. Friedman and C.D. Swerdlow
Fig. 115.1 Bradycardia pacing parameters and stored electrogram of VT terminated by antitachycardia pacing. See text for details
110 ms, and the resulting fusion beat. The fusion beat is followed by a VS beat with V-V interval of 430 ms. This completes a short-long-short sequence that initiates VT. Without electrograms, we determine from the intervals alone if this last beat is a premature ventricular beat or if it is conducted from the NCAP Ap beat with a Ap-–VS interval of 480 ms. The latter explanation suggests proarrhythmia related to specific features of bradycardia pacing (long PVARP and NCAP) and first degree AV block.1 The correct action to increase percent of cardiac resynchronization pacing is to shorten the PVARP. This may also reduce
the incidence of VT by preventing pacing-related proarrhythmia.
Reference 1. Sweeney MO, Ruetz LL, Belk P, Mullen TJ, Johnson JW, Sheldon T. Bradycardia pacing-induced short-long-short sequences at the onset of ventricular tachyarrhythmias: a possible mechanism of proarrhythmia? J Am Coll Cardiol. 2007;50(7):614-622.
Case 116 Paul A. Friedman and Charles D. Swerdlow
Case Summary The 78-year-old man described in previous case 116 is followed. No programming was performed at an August 2007 visit. Instead, amiodarone therapy was initiated to prevent asymptomatic VT requiring termination by antitachycardia pacing. Six weeks later, on September 28, 2007, the patient developed rapid palpitations after receiving bad news. Within a few minutes, he experienced dyspnea and his typical angina. He used nitroglycerin spray twice without relief. Twenty minutes later he collapsed and died. Figure 116.1 shows the data retrieved postmortem from an episode of ICD-detected VT that was recorded about 20 min after he collapsed. What is the diagnosis? What steps could have been taken to prevent the patient’s death?
Case Discussion The Flashback™ intervals in the top left panel show RR intervals for approximately 11 min preceding ICD detection of “VT.” The extreme RR variability is characteristic of
undersensing during ventricular fibrillation.1 The interval plot at lower left shows two unsuccessful attempts at antitachycardia pacing denoted by arrowheads. The stored EGM at right shows, relatively slow, late-stage VF, probably with substantial undersensing. It is difficult to assess the magnitude of undersensing because the true-bipolar sensing EGM was not recorded. The most likely explanation is that amiodarone slowed the rate of VT so that it remained undetected. The patient became ischemic, and the rhythm degenerated into VF. Antiarrhythmic drugs that slow the rate of VT below the programmed VT rate cut-off can prevent initial detection of VT2 or divert appropriate therapy during reconfirmation. In this case, VT could likely have been prevented or reduced in frequency by reprogramming the PVARP. Even in the absence of proarrhythmia, infrequent asymptomatic VT terminated by antitachycardia pacing may not require pharmacological prophylaxis. Amiodarone often slows the VT cycle length by 100 ms or more, and patients with advanced heart failure may not tolerate even relatively slow VT. 2 When therapy with amiodarone is initiated either for therapy of atrial or ventricular arrhythmias in a patient with known monomorphic VT, the VT detection interval should usually be increased by 50 ms.
P.A. Friedman (*) Division of Cardiovascular Diseases and Department of Internal Medicine, Mayo Clinic, 200 First Street SW, Rochester, MN 55905, USA e-mail:
[email protected] C.D. Swerdlow Division of Cardiology, David Geffen School of Medicine, Cedars-Sinai Medical Center, Los Angeles, CA, USA e-mail:
[email protected] A. Natale et al. (eds.), Cardiac Electrophysiology, DOI: 10.1007/978-1-84996-390-9_116, © Springer-Verlag London Limited 2011
453
454
P.A. Friedman and C.D. Swerdlow
Stored electrogram VT 400 ms FVT 320 ms VF 240 ms
RR Interval (ms)
Flashback intervals
1,000 800 600 400 200 0
−600
−400
−200
0
Time (s) [0 = Detection]
Interval plot A−A
V−V Interval (ms) 2,000 1,700 1,400 1,100 800
VF = 320 ms FVT = 240 ms
VT = 400 ms
Burst
600 400 200 −40
−30
−20
−10
0 10 20 Time (s) [0 = Detection]
30
*
*
*
Septermber 28, 2007
Fig. 116.1 Flashback intervals, interval plot, and stored electrogram from ICD-detected VT. See text for details
References 1. Swerdlow CD, Friedman PA. Advanced ICD troubleshooting: part II. Pacing Clin Electrophysiol. 2006;29(1):70-96.
2. Bansch D, Castrucci M, Bocker D, Breithardt G, Block M. Ventricular tachycardias above the initially programmed tachycardia detection interval in patients with implantable cardioverter- defibrillators: incidence, prediction and significance. J Am Coll Cardiol. 2000;36(2):557-565.
Case 117 Paul A. Friedman and Charles D. Swerdlow
Case Summary A 24-year-old woman with complex congenital heart disease, depressed systemic ventricular function, and congestive heart failure previously received an InSync II Marquis™ 7289 CRT-D utilizing a Medtronic Sprint Fidelis 6949 right ventricular defibrillation lead, 4193 left ventricular lead, and
a Novus 5076 right atrial lead. While in hospital on telemetry, she develops recurrent episodes of pacing failure, ventricular arrhythmias, and shocks (Fig. 117.1). The device status report is shown in Fig. 117.2. In order to further troubleshoot the device malfunction, a real-time electrogram and surface ECG are simultaneously recorded (Fig. 117.3). What is the diagnosis? How is the problem corrected?
Fig. 117.1 Telemetry recordings obtained during hospitalization. See text for details
P.A. Friedman (*) Division of Cardiovascular Diseases and Department of Internal Medicine, Mayo Clinic, 200 First Street SW, Rochester, MN 55905, USA e-mail:
[email protected] C.D. Swerdlow Division of Cardiology, David Geffen School of Medicine, Cedars-Sinai Medical Center, Los Angeles, CA, USA e-mail:
[email protected] A. Natale et al. (eds.), Cardiac Electrophysiology, DOI: 10.1007/978-1-84996-390-9_117, © Springer-Verlag London Limited 2011
455
456
Fig. 117.2 ICD status report. Note impedance values. Discussion in text
P.A. Friedman and C.D. Swerdlow Aug 08, 2007 09:43:11 9989 Software Version 2.0 Copyright Medtronic, Inc. 2002
ICD Model: InSync II Marquis DR 7289 Serial Number: PRJ 620851S
Status report
Page 1
Last Interrogation: Aug 08, 2007 07:13:28 Battery voltage
Lead impedance
(ERI = 2.62 V) Aug 08, 2007 07:13:26 Voltage Last capacitor formation Mar 30, 2007 19:54:26 Charge time Energy Last charge
7.96 s 0.0 - 30.0 J
Aug 08, 2007 03:00:03 A. Pacing RV Pacing LV Pacing V. Defib SVC Defib EGM amplitude
Aug 08, 2007 05:27:42 Charge time Energy Sensing integrity counter
4.51 s 0.0 − 24.0 J
Aug 08, 2007 05:26:38 P wave amplitude RVR-wave amplitude Last high voltage rherapy
2.88 V
(if>300 counts, check for sensing issues) Since Aug 08, 2007 03:25:21 120−130 ms V−V intervals 17
Case Discussion The patient has striking episodes of failure of ventricular output (seen in Fig. 117.1 and 117.3) leading to short–long–short intervals, which are pro–arrythmic and induce polymorphic ventricular arrhythmias (Fig. 117.1). The tracing in Fig. 117.3 is duplicated in Fig. 117.4 with additional labels added. The arrows indicate the paced atrial beats not followed by a QRS, which lead to alternating V-V intervals. Note that the marker channel for these beats displays “VS” with two bars of differing heights (circled in Fig. 117.4). This double-bar marker indicates safety pacing. Safety pacing prevents “cross talk,” which occurs when an atrial pacing event is sensed on the ventricular channel, inhibiting ventricular output. As shown in Fig. 117.5 an atrial pacing event is followed by a ventricular blanking period, then a cross-talk window. Any event sensed during the cross talk window will result in delivery of a ventricular pacing stimulus with an abbreviated atrioventricular interval (AVI). Thus, if an atrial event is sensed on the ventricular channel during the cross-talk window, a pacing stimulus is delivered after an abbreviated AVI that may be suboptimal hemodynamically, but preferable to asystole. If an intrinsic
Aug 08, 2007 05:08:31 Measure impedance Delivered energy Vaveform Pathway
432 Ω 560 Ω 368 Ω 49 Ω None
1.4 mV 5.0 mV
42 Ω 24.1 J Biphasic B>AX
ventricular event is sensed during the cross-talk window, the ventricular pacing stimulus follows shortly after its onset (due to the abbreviated AVI), avoiding pacing during repolarization and proarrhythmia. In the present case, by its design, the 7289 CRT-ICD delivers safety pacing pulses only to the right ventricular lead. Due to failure of the Fidelis™ lead in this patient, output failure occurs, awnd the ventricle is not paced. Note that the other (captured) pacing pulses in Fig. 117.4 are associated with the “BV” marker for biventricular pacing. It is likely that only the left ventricular lead is capturing during pacing, so that during safety pacing, which is delivered only to the nonfunction right ventricular lead, ironically, no ventricular capture occurs. Figure 117.6 further corroborates this. Note that after an external shock is delivered to terminate ventricular tachycardia, biventricular pacing occurs that captures only the LV (first arrow). Due to the width of the LV-only paced QRS, the same ventricular complex is sensed on the RV lead, giving rise to an “FS” marker (second arrow), for “fib sense” event. Thus, while the Fidelis™ fails to pace, it remains capable of sensing. Also note that the impedance of the failed lead is not elevated (Fig. 117.2). Fidelis lead malfuncntion without significant increases in impedance have been described.
Case 117 Fig. 117.3 Simultaneous surface telemetry (top panel) and device markers and electrograms (bottom panel)
Fig. 117.4 Simultaneous surface and device recordings, as in 118.3. Large arrows indicated paced p waves without subsequent QRS. Each such event has a characteristic marker (The first one is circled). Details in text
457
458 Fig. 117.5 Venricular sensing following a paced atrial event. Sensing during the cross talk window will trigger a paced ventricular event with an abbreviated AV interval. Figure courtesy of Dr. David Hayes
Fig. 117.6 Episode from same patient treated with shock. Note the last complex is biventricularly paced, but sensed on the RV channel. See text for discussion
P.A. Friedman and C.D. Swerdlow
Case 118 Paul A. Friedman and Charles D. Swerdlow
Case Summary A 59-year-old man with ischemic cardiomyopathy had a single-chamber Medtronic Entrust™ ICD and Sprint Fide lis™ Model 6949 dual-coil true bipolar lead implanted over 2 years ago after out-of-hospital VF. His NYHA Class is III and left ventricular ejection fraction is 34%. After an episode of monomorphic VT storm a year ago, he has been treated with amiodarone 200 mg per day to decrease the frequency of VT. He now presents for a routine follow-up visit. The pacing threshold is 0.5 V at 0.4 ms, and the pacing lead impedance is stable at 560 W. One episode of device-detected VT is stored in the ICD. The patient had no awareness of this episode, which occurred 6 weeks ago. Figure 118.1 shows the corresponding stored electrogram. Figure 118.2 shows 80-week trend plots for R wave amplitude and impedance measurements corresponding to each-high voltage lead which are displayed with each interrogation. What is the diagnosis? What steps should be taken now?
Case Discussion Figure 118.1 an episode of monomorphic VT with AV dissociation that was detected immediately and terminated by antitachycardia pacing. The high-voltage impedance trends show intermittent high values over the last 5 weeks. The defibrillation impedance for
P.A. Friedman (*) Division of Cardiovascular Diseases and Department of Internal Medicine, Mayo Clinic, 200 First Street SW, Rochester, MN 55905, USA e-mail:
[email protected] the right ventricular coil is measured between that coil and the ICD can. The impedance for the superior vena cava electrode is measured between that electrode and the right ventricular coil, the high-voltage electrode of opposite polarity. Simultaneous fluctuations in both measurements indicate intermittent conduction failure in the common portion of the circuit that includes right ventricular coil. The differential diagnosis includes isolated high-voltage conductor failure in the lead or failure of conductive contact in the header (e.g., loose set screw). Recalled Fidelis leads have been implanted in more patients than any other lead family.1 About 95% of Fidelis lead failure involves one of the two pace-sense electrodes. Failures of pace-sense electrodes present with intermittent high pacing impedance measurements and evidence of oversensing, including inappropriate shocks.2 See Fig. 118.3. A modification of previous lead failure algorithm may assist in identifying lead failures before patients receive inappropriate shocks.3 Isolated high-voltage failures represent only about 5% of reported Fidelis failures. But the time course in this case is unusual for header-connector problems, which usually occur in the first year after implant. So the differential diagnosis includes two unlikely events, both of which require operative intervention despite the fact that the patient is asymptomatic and therapy of VT was successful; successful antitachycardia pacing provides no information about the present or future efficacy of defibrillation, and the likelihood of needing defibrillation is significant in a patient with a history of out-of-hospital VF. If no problem is identified within the header at surgery, the lead should be replaced regardless of intraoperative impedance measurements, which may be normal in cases of intermittent conductor failure. No other diagnostic testing is helpful. Chest radiography does not identify Fidelis lead failures, and noninvasive defibrillation testing may not identify intermittent high-voltage failures.
C.D. Swerdlow Division of Cardiology, David Geffen School of Medicine, Cedars-Sinai Medical Center, Los Angeles, CA, USA e-mail:
[email protected] A. Natale et al. (eds.), Cardiac Electrophysiology, DOI: 10.1007/978-1-84996-390-9_118, © Springer-Verlag London Limited 2011
459
460
Fig. 118.1 Stored electrogram of device-detected VT
P.A. Friedman and C.D. Swerdlow
Case 118
461
R wave amplitude At implant 15.8 mV Last 15.2 mV
Highest 16.6 mV Lowest 11.2 mV mV >20.0 10.0 5.0 3.0 2.0 1.0
09/08/06
11/17/06
01/26/07 04/06/07 06/15/07 Last 80 weeks (min/max per week)
08/24/07
11/02/07
0.0 200 150 100 80 60 40 30 09/08/06
11/17/06
01/26/07 04/06/07 06/15/07 Last 80 weeks (min/max per week)
08/24/07
11/02/07
200 150 100 80 60 40 30 09/08/06
11/17/06
01/26/07 04/06/07 06/15/07 Last 80 weeks (min/max per week)
08/24/07
11/02/07
3,000 Ω
CareAlert™ Highest Lowest
>3,000 Ω 552 Ω Ω >3,000
2,498 short V−V intervals (120 − 130 ms) in last week
2,000 1,500 1,000 800 600 400 300 207 bpm) : 30 Joule x 6 VT Zone (rate > 167 bpm): ATP x 3 (Burst), 15 Joule x 1 and 30 shocks may improve patient quality-of-life and extend the Joule x 4 longevity of the implanted device. In this case, ATP successWhat is the mechanism of the tachycardia and how did it fully terminated an atrial tachyarrhythmia and prevented an inappropriate shock. terminate?
Case Discussion Examination of the intracardiac electrograms demonstrates a 1:1 tachycardia that starts with an atrial beat that is in the post ventricular atrial refractory period. This beat appears to
R. Lo (*) Riverside Electrophysiology, 4000 14th Street, Suite 209, Riverside, CA 92501 e-mail:
[email protected] A. Al-Ahmad and P.J. Wang School of Medicine, Stanford University, 300 Pasteur Drive, H-2146, Stanford, CA 94305 e-mail:
[email protected];
[email protected] A. Natale et al. (eds.), Cardiac Electrophysiology, DOI: 10.1007/978-1-84996-390-9_124, © Springer-Verlag London Limited 2011
481
482
Fig. 124.1 Intracardiac electrogram of tachycardia with termination via ATP
R. Lo et al.
Case 125 Ronald Lo, Amin Al-Ahmad, and Paul J. Wang
Case Summary A 72 year old man with a history of nonischemic dilated cardiomyopathy with an ejection fraction of 20%, complete left bundle branch block and mechanical aortic and mitral valves. He has been having symptomatic Class III–IV heart failure symptoms and subsequently underwent implantation of a St Jude Medical Atlas + HF Model V-340 biventricular defibrillator. His symptoms of heart failure have improved significantly after implantation of his cardiac resynchronization device. However, he does note occasionally the sensation of palpitations with subjective worsening of dyspnea on exertion. His device interrogation revealed the event displayed in Fig. 125.1. What is the rhythm displayed? Why would this rhythm cause him to experience worsening heart failure symptoms?
Case Discussion Interrogation of his device revealed recording of two electrogram channels, the atrial and ventricular channels. Prior to the event, he is noted to be tracking his atrium with biventricular pacing. The fourth beat of the electrogram reveals
the start of an atrial arrhythmia, likely an atrial tachycardia with predominately 1:1 conduction to the ventricle. It is interesting to note that the PR interval during the atrial tachycardia is shorter than the PR interval during a paced rhythm of the first two beats. This may be due to the location of the atrial tachycardia being located closer to the AV node. Upon detection of an atrial arrhythmia, or atrial high rate episodes, the device mode switches from a tracking mode to a demand backup pacing mode, in this case DDI. In DDI mode, there is sensing in both the atrium and the ventricle, with the only action the device is taking is inhibition of pacing with a sensed complex. The benefit of DDI pacing is prevention of rapid atrial tracking, however with less optimal atrial and ventricular timings, especially in a cardiac resynchronization device. In this patient, there is loss of biventricular pacing as the ventricular complexes are due to the intrinsic conduction from the atrial tachycardia. The loss of beneficial biventricular pacing and atrial-ventricular synchrony contributes to worsening hemodynamic function and mechanical dyssynchrony. Treatment of the atrial tachycardia with beta blockers or antiarrhythmic medications may decrease the likelihood of this from recurring. This case illustrates the importance of maintaining biventricular pacing in patients who have atrial tachyarrhythmias.
R. Lo (*) Riverside Electrophysiology, 4000 14th Street, Suite 209, Riverside, CA 92501 e-mail:
[email protected] A. Al-Ahmad and P.J. Wang School of Medicine, Stanford University, 300 Pasteur Drive, H-2146, Stanford, CA 94305 e-mail:
[email protected];
[email protected] A. Natale et al. (eds.), Cardiac Electrophysiology, DOI: 10.1007/978-1-84996-390-9_125, © Springer-Verlag London Limited 2011
483
484 Fig. 125.1 Device interrogation
R. Lo et al.
Section Clinical Cases
III
Case 126 Mehmet K. Aktas, Abrar H. Shah, and James P. Daubert
Case Summary
Case Discussion
A 51-year-old man on chronic methadone therapy for a history of heroin abuse presented to the emergency room with cough and dyspnea. On exam he was found to be tachypneic and pulse oximetry showed a saturation of 88% on room air and a portable chest X-ray revealed a right lower lobe infiltrate. His serum potassium was 3.6 meq/L and his serum magnesium was 2.0 meq/L. An electrocardiogram performed upon arrival showed sinus arrhythmia with evident U waves and a prolonged QTc interval (Fig. 126.1) although this was not recognized at the time. He was given intravenous moxifloxacin while in the emergency room and was hospitalized for in-patient antibiotic therapy. Twelve hours later the patient reported feeling anxious and was found to be diaphoretic. Telemetry monitoring was initiated and showed sinus rhythm with frequent ventricular ectopy. An electrocardiogram showed ventricular bigeminy with significant QT interval prolongation (Fig. 126.2). Minutes later the patient became pulseless and apneic and was found to be in ventricular fibrillation (Fig. 126.3). Chest compressions were started and within a minute spontaneous return of sinus rhythm was noted. Serial electrocardiograms demonstrated progressive QT prolongation with rate corrected QT intervals (QTc) as high as 630 ms. What was the likely cause for this patient’s cardiac arrest?
The QT interval, the electrocardiographic gauge of ventricular repolarization, is often overlooked or misinterpreted. Alter ations to the timing and mechanism of ventricular repolari zation can lead to ventricular tachyarrhythmias particularly “short-long-short” sequences, which are often a trigger of torsade de pointes. The factors influencing the QT interval are complex and may include a variety of channelopathies, changes in autonomic innervation, and acquired factors such as drugs or electrolyte disturbances.1,2 The patient described was on methadone which is a known QT prolonging drug and hence prudence would require that the QT interval be closely monitored when other QT prolonging drugs are begun.3–5 Even with a prolonged baseline QTc (which was overlooked) he was started on a fluoroquinolone type antibiotic, moxifloxacin, which has been shown to consistently prolong the QT interval and has rarely been associated with torsade de pointes.6,7 The combination of methadone, and moxifloxacin in this patient led to significant QT prolongation and torsade de pointes. Once drug induced torsade de pointes (or even significant QT prolongation without torsade) is identified, immediate discontinuation of the offending drug or drugs is required. Temporary pacing may be considered to prevent pause related ventricular tachyarrhythmias and “short-long-short” sequences. Chronotropic agents, such as isoproterenol or atropine, may also be considered to increase the heart rate in attempt to shorten the QT interval and eliminate “short-longshort” sequences. Intravenous magnesium sulfate, a safe and effective adjunctive therapy, may also be given for the acute termination of torsade de pointes.8 Potassium should be maintained in the high normal range. The patient above continued to have salvos of ventricular fibrillation despite these measures and so low-dose intravenous dopamine for chronotropic support was started and the episodes of ventricular fibrillation stopped. Moxifloxacin was discontinued and over the ensuing days his QT interval returned to baseline. His methadone was gradually tapered with further normalization of his QTc. Since discharge has remained in sinus rhythm with no further arrhythmias or syncope.
M.K. Aktas (*) Department of Cardiology/Electrophysiology, University of Rochester Medical Center, 601 Elmwood Ave, 679C, Rochester, NY 14642 e-mail:
[email protected] A.H. Shah Department of Cardiology, University Cardiovascular Associates, 2365 South Clinton Ave., Suite 100, Rochester, NY 14618 e-mail:
[email protected] J.P. Daubert Cardiac Electrophysiology, Cardiology Division, Duke University Health System, DUMC Box 3174 Duke Hospital 7451H, Durham, NC 27710 e-mail:
[email protected] A. Natale et al. (eds.), Cardiac Electrophysiology, DOI: 10.1007/978-1-84996-390-9_126, © Springer-Verlag London Limited 2011
487
488
M.K. Aktas et al.
Fig. 126.1 Electrocardiogram at presentation showing sinus arrhythmia. The QTc interval is prolonged at 495 ms
Fig. 126.2 Electrocardiogram shows a ventricular bigeminal rhythm which confounds calculation of the QTc. Nevertheless, the QT interval is severely prolonged extending out to the succeeding QRS complex. The QTc is estimated at about 630 ms
Case 126
489
Fig. 126.3 Telemetry strip showing ventricular bigeminy with a “short-long-short” sequence followed by torsade de pointes
References 1. Roden DM. Drug-induced prolongation of the QT interval. N Engl J Med. 2004;350:1013-1022. 2. Moss AJ. Drug-induced QT prolongation: an update. Ann Noninvasive Electrocardiol. 2006;11:1-2. 3. Wedam EF, Bigelow GE, Johnson RE, Nuzzo PA, Haigney MC. QT-interval effects of methadone, levomethadyl, and buprenorphine in a randomized trial. Arch Intern Med. 2007;167:2469-2475. 4. Krantz MJ, Lewkowiez L, Hays H, Woodroffe MA, Robertson AD, Mehler PS. Torsade de pointes associated with very-high-dose methadone. Ann Intern Med. 2002;137:501-504.
5. Chugh SS, Socoteanu C, Reinier K, Waltz J, Jui J, Gunson K. A community-based evaluation of sudden death associated with therapeutic levels of methadone. Am J Med. 2008;121:66-71. 6. Dale KM, Lertsburapa K, Kluger J, White CM. Moxifloxacin and torsade de pointes. Ann Pharmacother. 2007;41:336-340. 7. Sherazi S, DiSalle M, et al. (2008). Moxifloxacin-induced torsades de pointes. Cardiol J 15(1): 71-73. 8. Tzivoni D, Banai S, Schuger C, et al. Treatment of torsade de pointes with magnesium sulfate. Circulation. 1988;77:392-397.
Case 127 Loren P. Budge and John P. DiMarco
Case Summary A 55-year-old man was brought to the emergency department after an episode of palpitations and syncope. He had no prior cardiac history, and his medical history was significant only for hypertension, which he has been trying to manage with a salt-restricted diet. He has been quite active and denied previous symptoms of angina or heart failure. He has not been taking any medications. His symptoms started abruptly while sitting at his desk at work, and consisted of rapid palpitations with chest pain, shortness of breath, lightheadedness, and diaphoresis. A nurse was present and reported a heart rate near 200 bpm. After a few minutes, he briefly lost consciousness, then quickly awoke and felt well. He has no
known history of arrhythmias, although he has had several prior episodes of palpitations in the past which have resolved spontaneously for which he had not sought evaluation. The rescue squad was called. During transport several short runs of a wide complex tachycardia were noted on monitor but no strips were saved. Upon arrival at the emergency room, his heart rate was 80, with a blood pressure of 108/72. His electrocardiogram is shown in Fig. 127.1. An echocardiogram was obtained later that day and is shown in Fig. 127.2. This led to a cardiac magnetic resonance study as shown in Fig. 127.3. What is this patient’s diagnosis? What arrhythmias is this patient likely to have? What would you do next?
Fig. 127.1 ECG on arrival to the emergency department
L.P. Budge (*) Cardiovascular Division, University of Virginia Health System, 1215 Lee Street, Box 800662, Charlottesville, VA, 22908 USA e-mail:
[email protected] J.P. DiMarco Cardiovascular Division, University of Virginia Health System, 1215 Lee Street, Charlottesville, VA, 22908 USA e-mail:
[email protected] A. Natale et al. (eds.), Cardiac Electrophysiology, DOI: 10.1007/978-1-84996-390-9_127, © Springer-Verlag London Limited 2011
491
492
Fig. 127.2 Transthoracic echocardiogram apical four chamber view at end-systole. This echocardiogram is diagnostic for left ventricular noncompaction. Pertinent findings include prominent left ventricular trabeculae with a ratio of non-compacted to compacted myocardium >2:1 (double-sided arrows demarcate compacted and non-compacted segments). Intertrabecular recesses are also seen (one-sided arrow). Color Doppler (not shown) demonstrates flow within these recesses with communication to the left ventricular cavity
L.P. Budge and J.P. DiMarco
Fig. 127.3 Cardiac magnetic resonance (CMR) apical short axis view in diastole. This CMR demonstrates the hallmark features of left ventricular noncompaction. There is a greater than 2.3:1 ratio of non-compacted to compacted myocardium (demarcated by the doublesided arrows shown) with prominent inferolateral trabeculae. Cine (not shown) reveals hypokinetic contraction of the noncompacted segments with evidence of blood flow within the trabecular space with communication to the left ventricular cavity
Case Discussion This gentleman presented with an episode of syncope with non-sustained wide complex tachycardia noted by the rescue squad prior to his admission. His admission ECG is nonspecific since it shows only normal sinus rhythm with lateral T wave inversions. However, his echocardiogram shows prominent left ventricular trabeculae and is diagnostic for isolated left ventricular non-compaction (LVNC). LVNC is an uncommon congenital cardiomyopathy. Patients with LVNC will manifest on echocardiography prominent left ventricular (LV) trabeculae with deep intertrabecular recesses. This pattern is caused by intrauterine arrest of compaction, resulting in two layers of myocardium: compacted and non-compacted. There is continuity between the LV cavity and the intertrabecular recesses, without any communication to the epicardial vessels. LVNC most commonly affects the inferolateral portion of the LV apex, although other areas, including the RV can also be affected. There is usually a corresponding decrease in LV ejection fraction. The most common clinical presentations of symptomatic LVNC are heart failure, arrhythmia (atrial or ventricular), chest pain or systemic embolism. The sinus rhythm ECG usually shows only non-specific ST or T wave abnormalities. Associated conduction abnormalities such as bundle branch or fascicular blocks, or Wolf-Parkinson-White syndrome may be seen. When LVNC occurs in families, it has been linked to mutations in a number of cytoskeletal proteins and the
inheritance pattern is usually autosomal dominant. There is some phenotypic and genotypic overlap with other cardiomyopathies, especially hypertrophic cardiomyopathy. LVNC may also occur with several neuromuscular disorders such as Barth Syndrome, Charcot-Marie-Tooth 1a, MelnickNeedles Syndrome and Nail-patella Syndrome. It may rarely be seen in conjunction with other forms of congenital heart disease, including Ebstein’s anomaly, bicuspid aortic valve, L-TGA, left atrial appendage isomerism, and ventricular septal defects. The prevalence of LVNC was originally thought to be extremely rare, but as imaging modalities and awareness foster recognition of milder forms of the disorder, more cases with less prominent clinical and imaging findings are being diagnosed. The diagnostic criteria of isolated LVNC differ depending on the imaging modality used. Echocardiographic criteria include an absence of coexisting cardiac abnormalities, a ³ 2:1 ratio of non-compacted to compacted myocardium (NC/C ratio) at end-systole with thickening of the myocardial wall (as opposed to apical thrombus, which would not thicken) and documentation of flow within the intertrabecular recesses. Cardiac magnetic resonance imaging criteria for LVNC require a NC/C ratio of >2.3:1 at end-diastole which has a sensitivity of 86% and a specificity of 99%. Prognosis in LVNC is controversial. Most studies report no more than 3–4 year follow-up durations and are biased towards patients with the more severe forms of the disease. Those who present in childhood have a reported 14% 3 year mortality. LV function often recovers transiently if heart
Case 127
failure therapy is initiated, only to again worsen in early adulthood. Of adults who present with symptoms, 41% of patients had documented ventricular tachycardia (VT), 53% were hospitalized for heart failure, and 24% had a thromboembolic event over a mean 44 month follow-up period. Transplant-free survival in symptomatic patients was 58% at 5 years, with a 35% mortality rate, half of whom died suddenly. However among asymptomatic patients with an incidental finding of LVNC, 5 year transplant-free survival was 97%, with less than a 10% thromboembolic rate. Further studies on the long-term prognosis of asymptomatic patients with findings of LVNC during cardiac imaging are clearly needed. Treatment recommendations include standard medical therapy according to ejection fraction, heart failure symptoms, and atrial arrhythmias. Due to the propensity for thrombus formation, warfarin is recommended for patients with an EF £ 40% or with atrial fibrillation even if other risk factors are absent. Given that sudden cardiac death is common in this population, and there is currently no reliable way to determine who is at risk for life-threatening arrhythmia, recent guidelines support consideration of ICD therapy as a Class 2B indication. This patient presented with heart failure and what was most likely ventricular tachycardia. He was started on therapy with an ACE inhibitor and a beta blocker. A dual chamber ICD was inserted. Three months after insertion, he had an episode of lightheadedness and an electrogram stored by the ICD showed a burst of monomorphic ventricular tachycardia that was broken with antitachycardia pacing. No atrial arrhythmias have been detected.
493
Bibliography Engberding R, Yelbuz TM, Breithardt G. Isolated noncompaction of the left ventricular myocardium – a review of the literature two decades after the initial case description. Clin Res Cardiol. 2007;96:481-488. Epstein AE, DiMarco JP, Ellenbogen KA, et al. ACC/HRS 2008 guidelines for device-based therapy of cardiac rhythm abnormalities: a report of the American College of Cardiology/American Heart Association Task Force on Practice Guidelines (Writing Committee to Revise the ACC/AHA/NASPE 2002 Guideline Update for Implantation of Cardiac Pacemakers and Antiarrhythmia Devices): developed in collaboration with the American Association for Thoracic Surgery and Society of Thoracic Surgeons. Circulation. 2008;117:e350-e408. Frischknecht BS, Attenhoffer Jost CH, et al. Validation of noncompaction criteria in dilated cardiomyopathy, and valvular and hypertensive heart disease. J Am Soc Echo. 2005;18:865-872. Jenni R, Oechslin E, Schneider J, Attenhofer Jost C, Kaufmann PA. Echocardiographic and pathoanatomical characteristics of isolated left ventricular non-compaction: a step towards classification as a distinct cardiomyopathy. Heart. 2001;86:666-671. Kobza R, Jenni R, Erne P, Oechslin E, Duru F. Implantable cardioverster-defibrillators in patients with left ventricular noncompaction. PACE. 2008;31:461-467. Murphy RT, Thaman R, Blanes JG, et al. Natural history and familial characteristics of isolated left ventricular non-compaction. Eur Heart J. 2005;26:187-192. Oechslin EN, Attenhofer Jost CH, Rojas JR, Kaufmann PA, Jenni R. Long-term follow-up of 34 adults with isolated left ventricular noncompaction: a distinct cardiomyopathy with poor prognosis. J Am Coll Cardiol. 2000;36:493-500. Petersen SE, Selvanayagam JB, Wiesmann F, et al. Left ventricular non-compaction: insights from cardiovascular magnetic resonance imaging. J Am Coll Cardiol. 2005;46:101-105.
Case 128 David J. Callans
Case Summary
What is the likely mechanism responsible for these findings?
A 62-year-old man without previous cardiac history presented to the hospital with complaints of near syncope and exercise intolerance. The presenting electrocardiogram (Fig. 128.1) demonstrates sinus rhythm with a normal PR interval and narrow QRS on half of the conducted beats, intermittent AV block, and alternating left and right bundle aberrancy on the other half of the conducted beats.
Case Discussion An electrophysiologic study was performed to investigate the pathophysiology of his AV block (Fig. 128.2). A split His potential was recorded and intermittent AV block was 500 ms
I II III aVR aVL aVF V1 V2 V3 V4 V5 V6 4:54:12 PM
4:54:13 PM
4:54:14 PM
4:54:15 PM
4:54:16 PM
Fig. 128.1 Presenting electrocardiogram with intermittent AV block and alternating left and right bundle aberration
D.J. Callans Department of Cardiology, University of Pennsylvania, 3400 Spruce Street, Philadelphia, PA 19104, USA e-mail:
[email protected] A. Natale et al. (eds.), Cardiac Electrophysiology, DOI: 10.1007/978-1-84996-390-9_128, © Springer-Verlag London Limited 2011
495
496
D.J. Callans 200 ms
I
aVF
V1
S
V6
hRA
CT
HIS d
C
HIS p
C 4:16:04 PM
4:16:05 PM
Fig. 128.2 Surface and intracardiac recordings during sinus rhythm and atrial pacing. A split His potential is seen, and atrial pacing results in intermittent intra-His conduction block
A proposed mechanism for this phenomenon is presented in Fig. 128.3. Despite normal infra-His conduction, alternating aberrancy is favored during 3:2 conduction because of alternating retrograde concealment, resulting in long-short input to the other bundle branch on the following portion of the sequence. Alternating bundle branch block is recognized as a high risk situation and is considered a Class 1 indication for permanent pacing. The patient was treated with dual chamber pacing, indicated because of the finding of intra His block, which resolved his symptoms in follow up.
Fig. 128.3 Concealment into the distal left bundle branch on the first aberrant beat “protects” the left bundle from long-short stimulation (by making the pause shorter with reference to the left bundle than the right) following the sinus complex that blocks at the infra His level. The pattern reverses itself in the next part of the sequence, shielding the right bundle branch in the same manner
induced with atrial pacing, with an intra-His level of conduction block. The alternating bundle branch pattern was not reproduced during the EP study, because a steady state pattern of 3:2 block could not be demonstrated, despite pacing autonomic manipulations.
Bibliography Bharati S, Lev M, Wu D, Denes P, Dhingra R, Rosen KM. Pathophysiologic correlations in two cases of split His bundle potentials. Circulation. 1974;49:615-623. Lerman BB, Marchlinski FE, Kempf FC, Buxton AE, Waxman HL, Josephson ME. Prognosis in patients with intra-Hisian conduction disturbances. Int J Cardiol. 1984;5:449-460. McAnulty JH, Murphy E, Rahimtoola SH. Prospective evaluation of intrahisian conduction delay. Circulation. 1979;59:1035-1039. Wu D, Denes P, Dhingra RC, et al. Electrophysiological and clinical observations in patients with alternating bundle branch block. Circulation. 1976;53:456-464.
Case 129 Andrew E. Darby and John P. DiMarco
Case Summary A 51 year-old man is referred for evaluation of heart failure with AV block. He had been in his usual state of health until 6 months earlier when he began to noted easy fatigue. He then developed gradually progressive dyspnea with exertion. Two weeks prior to presentation, the patient’s dyspnea with exertion progressed to the point of shortness of breath with walking only fifty feet. The onset of orthopnea and paroxysmal nocturnal dyspnea finally prompted him to seek care in the emergency department. His medical history includes hypertension, dyslipidemia, and obesity. He had donated his left kidney to a brother with polycystic kidney disease 13 years earlier. The patient does not smoke or use illicit substances. He lives with a wife and one child, and he works as a long-haul commercial truck driver. The patient was afebrile on presentation. He was relatively hypotensive with a blood pressure of 85/60, and he had a heart rate of 90 bpm. He was noted to clinically have heart failure with elevated jugular venous pressure, bibasilar crackles, and lower extremity edema. Studies obtained included a chest x-ray demonstrating cardiomegaly with pulmonary vascular congestion. His presenting electrocardiogram is shown (Fig. 129.1). A chest CT scan revealed pulmonary edema with prominent mediastinal and paratracheal lymphadenopathy. A transthoracic echocardiogram indicated severe left ventricular dysfunction with an approximate ejection fraction of 25%. A cardiac catheterization demonstrated no significant coronary disease. The patient’s dyspnea improved with diuresis, but he developed further changes in his electrocardiogram. Initially
A.E. Darby (*) Department of Cardiology, University of Virginia, 1215 Lee Street, Charlottesville, VA 22908, USA e-mail:
[email protected] J.P. DiMarco Cardiovascular Division, University of Virginia Health System, 1215 Lee Street, Charlottesville, VA 22908, USA e-mail:
[email protected] he was in sinus rhythm with first degree AV block and left bundle branch block (Fig. 129.1). He was later noted to have type II second degree AV block alternating with periods of complete heart block (Fig. 129.2). What disease process could account for this patient’s presentation of heart failure with advanced AV block?
Case Discussion The differential diagnosis of heart failure with advanced AV block should include infiltrative processes (sarcoidosis or myocarditis), lyme carditis, and certain genetic conditions which may present in the fourth or fifth decades of life (lamin A/C deficiency or alpha-myosin heavy chain gene mutation). This patient’s chest CT showed mediastinal lymphadenopathy which suggested sarcoidosis as the potential etiology. A cardiac MRI revealed patchy areas of delayed hyperenhancement in a non-coronary distribution suggestive of an infiltrative process (Fig. 129.3). He subsequently underwent a lymph node biopsy which revealed noncaseating granulomas diagnostic of sarcoidosis. The patient’s heart failure and advanced AV block were therefore deemed secondary to cardiac sarcoidosis. Sarcoidosis is a chronic, multisystem disorder of unknown etiology. It is characterized by the accumulation of T lymphocytes and macrophages in tissues leading to the formation of noncaseating granulomas which disrupt normal tissue architecture. The disorder may involve any organ system but most commonly affects the lungs, skin, eyes, liver, and lymphatics. Cardiac granulomas are found in nearly 25% of patients with sarcoidosis examined at autopsy. Importantly, it accounts for 13–25% of sarcoidosis-related deaths. Cardiac involvement may precede, follow, or occur concurrently with other organ involvement. The most common cardiac manifestations of sarcoidosis are conduction abnormalities, heart failure and ventricular arrhythmias. Sarcoid granulomas have an affinity for the conduction system. First degree AV block is common due to involvement of the AV node or bundle of His. Interventricular
A. Natale et al. (eds.), Cardiac Electrophysiology, DOI: 10.1007/978-1-84996-390-9_129, © Springer-Verlag London Limited 2011
497
498
A.E. Darby and J.P. DiMarco
Fig. 129.1 Presenting electrocardiogram demonstrating a first-degree AV block and left bundle branch block
Fig. 129.2 An electrocardiogram demonstrating high grade AV block. Since there is now a right bundle branch block pattern and the original ECG showed a left bundle branch block, this may represent either 2:1
AV block with alternating bundle branch aberration or complete AV block with a ventricular escape rate ½ the sinus rate
conduction defects (right or left bundle branch block) may also be apparent on the electrocardiogram. The most common rhythm abnormality among patients with clinicallyevident cardiac sarcoidosis is complete heart block, occurring in up to 30% of patients.
Ventricular dysrhythmias may also occur, and patients with cardiac sarcoidosis are at increased risk for sudden death. Infiltrating granulomas can cause inflammation with subsequent scar formation. This process may create a substrate for reentrant dysrhythmias. Ventricular tachyarrhythmias (VT)
Case 129
are the second most common mode of presentation of cardiac sarcoidosis. Sustained or nonsustained ventricular tachycardia are seen during Holter monitoring in about 23% of patients. Sudden death due to ventricular arrhythmias or complete heart block accounts for 25–65% of deaths due to cardiac sarcoidosis. Atrial arrhythmias are less common, occurring in approximately 19% of patients. Only limited data about the electrophysiologic findings in patients with sarcoid are available. Multiple VT morphologies are common in patients who present with VT. Low voltage areas of scar can be seen in either ventricle. Both epicardial and endocardial sites of VT origin are possible. Heart block and VT may occur separately or together. Heart failure is another common mode of presentation of cardiac sarcoidosis. Granulomatous infiltration and the subsequent inflammatory response may damage the myocardium resulting in systolic dysfunction. The infiltrative process may also cause abnormalities in diastolic function. Ventricular aneurysm formation has been noted to occur, and they may be a focus for ventricular dysrhythmias. Diagnostic criteria for cardiac sarcoidosis have been proposed (Table 129.1). The guidelines in the table do not incorporate updated imaging techniques, but they can serve as a reference point. Cardiac MRI has emerged as an extremely useful diagnostic tool as illustrated in Fig. 129.3. Published reports indicate an approximate 100% sensitivity for detecting myocardial infiltration suggestive of sarcoidosis. Therapy for cardiac sarcoidosis primarily consists of immunosuppression. Corticosteroids, such as prednisone dosed 1 mg/kg/day, are the standard of care. Steroid dosing should be gradually tapered based upon the clinical response. An inadequate response is managed by escalating
Table 129.1 Japanese Ministry of Health and Welfare Guidelines for the Diagnosis of Cardiac Sarcoidosis 1. Histologic Diagnosis Group: endomyocardial biopsy demonstrates epithelioid granulomata without caseating granulomata 2. Clinical Diagnosis Group: in patients with a histological diagnosis of extracardiac sarcoidosis, cardiac sarcoidosis is suspected when (a) and at least one of criteria (b) to (d) is present, and other etiologies such as hypertension and coronary artery disease have been excluded: a. Complete RBBB, LBBB, left axis deviation, AV block, VT, PVC, or pathological Q or ST-T change on resting or ambulatory electrocardiogram b. Abnormal wall motion, regional thinning, or dilatation of the left ventricle c. Perfusion defect by thallium-201 myocardial scintigraphy or abnormal accumulation of gallium-67 or technetium-99m myocardial scintigraphy d. Abnormal intracardiac pressure, low cardiac output, abnormal wall motion, or depressed ejection fraction of the left ventricle e. Interstitial fibrosis or cellular infiltration over moderate grade even if the findings are non-specific
499
Fig. 129.3 Cardiac magnetic resonance scan showing diffuse
immunosuppression with the addition of antimalarials, methotrexate, or azathioprine. Patients with left ventricular dysfunction should be treated with standard medications including ACE inhibitors, beta-blockers, and diuretics if needed. Patients with cardiac sarcoidosis often develop indications for permanent cardiac pacing. Strong consideration should be given to ICD placement given the high risk of ventricular arrhythmias and sudden cardiac death. Standard guidelines for primary and secondary prevention apply. ICD placement has been suggested for primary prevention, regardless of left ventricular function, for patients with frequent ventricular ectopy or nonsustained ventricular tachycardia. It is likely prudent to avoid amiodarone secondary to the potential confounding effects of pulmonary toxicity in patients with sarcoidosis. Our patient was started on 70 mg of prednisone daily. He underwent dual-chamber pacemaker implantation and was started on an ACE inhibitor and beta-blocker. The patient declined an ICD as implantation would preclude continuing his job as a commercial truck driver.
Bibliography Banba K, Kusano KF, Nakamura K, et al. Relationship between arrhythmogenesis and disease activity in cardiac sarcoidosis. Heart Rhythm. 2007;4:1292-1299. Dubrey SW, Bell A, Mittal T. Sarcoid heart disease. Postgrad Med J. 2007;83:618-623. Furushima H, Cinushi M, Sugiura H, Kasai H, Washizuka T, Aisawa Y. Ventricular tachyarrhythmia associated with cardiac sarcoidosis: its mechanisms and outcome. Clin Cardiol. 2004;27:217-222. Hiraga H, Yuwai K, Hiroe M. et al. Guideline for the diagnosis of cardiac sarcoidosis study report on diffuse pulmonary diseases. The Japanese Ministry of Health and Welfare 1993; 23–24.
500 Iannuzzi MC, Rybicki BA, Teirstein AS. Medical progress: sarcoidosis. N Engl J Med. 2007;357:2153-2165. Koplan BA, Soejima K, Baughman K, Epstein LM, Stevenson WG. Refractory ventricular tachycardia secondary to cardiac sarcoid: electrophysiologic characteristics, mapping and ablation. Heart Rhythm. 2006;3:924-929.
A.E. Darby and J.P. DiMarco Smedema JP, Snoep G, van Kroonenburgh MP, et al. Evaluation of the accuracy of gadolinium-enhanced cardiovascular magnetic resonance in the diagnosis of cardiac sarcoidosis. J Am Coll Cardiol. 2005;45:1683-1690. Syed J, Myers R. Sarcoid heart disease. Can J Cardiol. 2004;20: 89-93.
Case 130 Thomas J. Sawyer, Burr W. Hall, and James P. Daubert
Case Summary A 13-year-old boy is being evaluated for syncope. At the time of the syncopal event, he was a national, junior tennis champion, but had recently complained of increased fatigue and shortness of breath while playing tennis. One month
previously, his brother collapsed and died suddenly while running in his driveway at home. The patient’s paternal grandfather had also died suddenly just days before his brother. The 13-year-old boy’s ECG is shown below (Fig. 130.1). Representative echocardiographic images are also shown (Figs. 130.2 and 130.3). Of note, there was no LV outflow obstruction noted at rest or with provocative
Fig. 130.1 ECG at presentation
T.J. Sawyer (*) Cardiac Study Center, 1901 South Cedar St., Suite 301, Tacoma, WA 98405, USA e-mail:
[email protected] B.W. Hall Department of Cardiology, University of Rochester Medical Center, 601 Elmwood Avenue, Rochester, NY 14618, USA e-mail:
[email protected] J.P. Daubert Cardiology Division, Duke University Health System, DUMC Box 3174, Duke Hospital 7451H, Durham, NC 27710, USA e-mail:
[email protected] A. Natale et al. (eds.), Cardiac Electrophysiology, DOI: 10.1007/978-1-84996-390-9_130, © Springer-Verlag London Limited 2011
501
502
T.J. Sawyer et al.
at high risk for sudden death primarily as a result of ventricular arrhythmias. Treatment of the symptoms associated with HCM and identification of this high-risk subset of patients is the primary goal of the initial evaluation of these patients.
Findings in HCM
Fig. 130.2 Parasternal long axis image – late systolic frames. Note absence of SAM (systolic anterior motion of the mitral valve)
Classic physical findings include a systolic murmur that increases with maneuvers, which decrease either preload or afterload.2 Obstruction is not present in all patients (even with Valsalva or exercise) and thus a murmur may not be noted. Evidence of LVH is the hallmark of HCM. This is often suspected electrocardiographically and confirmed by 2D echocardiography. The hypertrophy is usually asymmetric with hypertrophy of the septum being greater than that of the free wall, but can be concentric or more predominant in other regions.1,2
Work Up for HCM Patients Once the diagnosis has been made, a detailed family history should be obtained. Special attention is given to a history of sudden death or unexplained syncope. All first degree relatives should undergo echocardiographic screening. Initial evaluation includes a 48 h Holter monitor and an exercise test. All patients should be told to avoid dehydration and strenuous exercise.
Fig. 130.3 Parasternal long axis image – early systolic frames. Note absence of SAM (systolic anterior motion of the mitral valve)
aneuvers What is your diagnosis? What is an appropriate m management strategy?
Case Discussion Hypertrophic cardiomyopathy (HCM) affects approximately 1 in 500 people (130.2). Thirty to fifty percent of patients with HCM have associated dynamic left ventricular outflow tract (LVOT) obstruction with even a higher percent having obstruction with exercise. Hemodynamically based symptoms consist primarily of shortness of breath and decreased exercise tolerance. Overall mortality in HCM is 30 mm
Case 130
• Abnormal blood pressure response to exercise (hypotension or failure to increase BP) • Non-sustained ventricular tachycardia Other less potent or agreed upon risk factors include LVOT obstruction, specific genotype, atrial fibrillation, and ischemia.
Treatment In the majority of patients, an implantable defibrillator is not necessary and therapy is focused on the relief of symptoms. Agents which block the effects of catecholamines and improve diastolic function are the mainstays of therapy. Increased diastolic filling time improves LVOT obstruction in patients with this variety of the disease. Beta-blockers are usually the initial therapeutic choice. Verapamil has also been used effectively and, like beta-blockers, has negative chronotropic and inotropic effects without significant alterations in afterload.1 If patients remain symptomatic despite optimal pharmacologic therapy, then several invasive options may be considered. Surgical septal myomectomy is the gold standard for the treatment of symptomatic, obstructive cardiomyopathy refractory to medical therapy. Successful operations can result in complete resolution of both mitral regurgitation and outflow gradient. Excellent long-term follow-up has been achieved with sustained improvement in exercise capacity and symptoms. Major complications typically occur in